Pediatric Aquifers

Lakukan tugas rumah & ujian kamu dengan baik sekarang menggunakan Quizwiz!

A 3-week-old infant is brought to his pediatrician with a chief complaint of light tan-colored stools and worsening jaundice. He is exclusively breastfed and has six to eight wet diapers per day. On exam, he appears to have scleral icterus and jaundice. Upon further workup, he is found to have an elevated direct bilirubin. What is his most likely diagnosis?

> The correct answer is A. ** A. Biliary atresia can present anytime between birth and 8 weeks of age, but usually presents after 2 weeks of age. Jaundice is usually the presenting finding, along with acholic stools, dark urine (from increased bilirubin excretion) and hepatosplenomegaly if the problem goes unrecognized. Laboratory values classically show conjugated hyperbilirubinemia, defined as direct bilirubin greater than 1.0 mg/dL if the total serum bilirubin (TSB) is less than 5.0 mg/dL, or more than 20% of TSB at any level. If biliary atresia is confirmed with further laboratory testing and imaging, surgical intervention must be pursued as soon as possible. B. Breastfeeding jaundice normally occurs within the first week of life, most often because of decreased intake leading to dehydration and increased enterohepatic circulation. This patient is exclusively breastfed, but his jaundice began at approximately day 16 of life. Also, breastfeeding jaundice causes increased unconjugated bilirubin levels, making this answer choice less likely. C. Caput succedaneum is caused by a serosanguinous fluid collection above the periosteum of an infant that crosses suture lines (as opposed to cephalohematoma which does not cross suture lines). This condition does not lead to significant hyperbilirubinemia. D. G6PD deficiency is an X-linked inherited disorder. The severity of this disorder is dependent upon the degree of deficiency of the enzyme, but may present with neonatal unconjugated hyperbilirubinemia. Laboratory findings show hemolytic anemia in the symptomatic state. Hemolysis is usually elicited by drugs, most notably primaquine and dapsone, as well as fava beans. This patient has a direct hyperbilirubinemia, making this answer choice unlikely. E. Physiologic jaundice peaks at three to four days of life and generally resolves within a day or two. This patient is well beyond that age, making this answer less likely. Physiologic jaundice would also only present with unconjugated hyperbilirubinemia.

A healthy male child brought into your office by his mother for a well-child examination. As part of your evaluation you assess his developmental milestones. He is able to run, make a tower of 2 cubes, has 6 words in his vocabulary, and can remove his own garments. What would you estimate this child's age to be based upon his developmental milestones?

A. 12 months B. 15 months C. 18 months D. 30 months E. 36 months A. This answer choice is incorrect. At age one year, gross motor skills include pulling to stand, standing alone, and perhaps first steps. Fine motor skills including putting a block in a cup and banging 2 cubes held in hands. At this age a child should be able to imitate vocalizations/sounds and babble. The majority of children this age will know 1 or 2 words in addition to "mama" and "dada." Social-emotional milestones at age one year are waving bye-bye and playing pat-a-cake. Running, building towers of blocks, removing clothing, and a 6-word vocabulary are more advanced skills than a 12-month-old would be expected to have. B. This answer choice is incorrect. At 15 months of age, a child should be able to stoop and recover, walk well, put a block in a cup, have a vocabulary of a few words, wave bye-bye, and drink from a cup. Running, building towers of blocks, removing clothing, and a 6-word vocabulary are more advanced skills than a 15-month-old would be expected to have. C. This answer choice is correct. At 18 months, a child should be able to walk backward, and 50-90% of children can run at this stage. An 18-month-old should be able to scribble, build a tower of 2 cubes, have 3-6 words in her or his vocabulary, and be able to help in the house and remove garments. D. This answer choice is incorrect. At 2 ½ years of age, kids can jump up and throw a ball overhand. They can build a tower of 6-8 cubes, point to 6 body parts, name 1 picture, put on clothing, and wash and dry their hands. This child is only able to build a tower of 2 cubes, can remove his clothing but does not yet put clothing on, and his vocabulary is limited to 6 words-leading us to believe he is younger than 2 ½ years old. E. This answer choice is incorrect. At age 3, children can balance on each foot for 1 second, wiggle their thumbs, name 4 pictures, name 1 color, name a friend, and brush their teeth with help. This child's vocabulary is only 6 words, he is not able to name a friend, he is only able to stack 2 cubes, and he has just started running, but is unable to balance on each foot for 1 second.

A 4-day-old infant is evaluated in the nursery for dysmorphic features. He was born at 40 weeks gestation by vaginal delivery to a 35-year-old G1P1 mother. Pregnancy was uncomplicated and mother declined prenatal screening. Vital signs and pulse oximetry are normal. Physical examination reveals mild hypotonia, epicanthal folds, upslanting palpebral fissures and a flat midface. Chromosomal studies are pending. Which of the following chromosomal abnormalities is most likely in this patient?

A. 45 X0 karyotype B. 47 XXY karyotype C. Trisomy 13 D. Trisomy 18 E. Trisomy 21 An extra chromosome 21 is indicative of Down syndrome. Patients with Down syndrome will present with the features described in the vignette as well as small ears, redundant nuchal skin, clinodactyly (the fifth digit is shorter and curved toward the radius). Additionally, cleft lip or palate, strabismus, and hypothyroidism may be seen. A. This answer is incorrect. This karyotype represents Turner syndrome. Patients with Turner syndrome are female (males demise) and present with neck lymphedema (can be seen as cystic hygroma in utero), low ears, hand/foot edema, hyperconvex nails, a wide chest with valgus at the elbows. Twenty percent of patients have coarctation of the aorta. Many do not present classically and may be diagnosed in their early teens when stature is short and no sexual maturation ensues. Most have a normal IQ. B. This answer is incorrect. This karyotype describes Klinefelter syndrome. These patients are males who have inherited an extra X chromosome. At birth they usually appear unremarkable without findings. Findings after puberty are variable but include testicular atrophy with resultant infertility, eunuchoid body (unusually long limbs compared to the body), and gynecomastia. C. This answer is incorrect. This syndrome with an extra chromosome 13 describes Patau syndrome. Presentation includes a general theme of shifting facial features toward the midline: microphthalmia (some present with a midline Cyclops eye), microcephaly with severe intellectual disability, polydactyly, cleft lip and palate, cardiac and renal defects, umbilical hernias, and cutis aplasia (particularly in the midline of the scalp). D. This answer is incorrect. Edwards' syndrome is caused by an extra chromosome 18. These infants present with severe intellectual disability, micrognathia (small jaw), low-set ears, short neck, overlapping fingers (digits 2 and 5 overlap medially upon closure of fist), heart defects, renal malformations, limited hip abduction, and "rocker-bottom" feet (the plantar side of foot has a u-shaped curve like a rocking chair).

A 10-month-old infant is brought to the emergency department by his very concerned and frantic grandmother. Earlier that day, she retrieved the child from his mother's new boyfriend, who had been watching him while his mother was at work. The grandmother makes it very clear she does not approve of this new boyfriend, and she is concerned that he is rough with her grandson. She demands that her grandson be worked up for injuries and that a restraining order be placed against the boyfriend. Which of the following finding does NOT indicate that a child is being physically abused?

A. A concaved, crescent-shaped mass on head CT B. A metaphyseal fracture of the wrist C. A spiral fracture of the tibia D. Retinal hemorrhages on fundoscopy E. Two posterior rib fractures A. This answer choice is incorrect. A concave, crescent-shaped lesion on head CT is indicative of a subdural hematoma. Subdural hematomas occur when bridging vessels are torn during the act of shaking a baby. Diffuse axonal injury occurs due to extreme rotational cranial acceleration force to the brain. CT scans and MRIs can detect the bleed. B. This answer choice is incorrect. Metaphyseal fractures, also called "bucket handle" or corner fractures, are caused by torsional force on the limb, or by violent shaking. These fractures are highly concerning for physical abuse. C. This answer choice is correct. Also called a "toddler's fracture," oblique, nondisplaced fracture of the distal tibia is a commonly occurring fracture in young, ambulatory kids ages 9 months to 3 years. It is not necessarily a sign of abuse. The child will usually present with acute onset of limp and refusal to bear weight on one leg. It usually occurs when a toddler falls while twisting, or gets a foot caught and falls while trying to free the foot. D. This answer choice is incorrect. Retinal hemorrhages are commonly found in victims of non-accidental head trauma. 65% to 90% of these most commonly infant patients present with diffuse, bilateral retinal hemorrhages. Note that certain types of retinal hemorrhages may also occur due to non-abusive diagnoses. Specialty consultation with a pediatric ophthalmologist is recommended. E. This answer choice is incorrect. Posterior rib fractures are highly associated with nonaccidental trauma.

A 12-year-old male with obesity comes to the clinic with a chief concern of right knee pain. On exam the right knee is neither swollen nor erythematous but he is noted to have limited ROM of the right hip. In addition, when he lifts his right leg, it externally rotates. The patient did not have a URI or any trauma preceding the onset of pain. The vital signs are normal at the time of the visit and he is well appearing and afebrile. What is/are the best next step(s) in management?

A. AP and lateral x-ray of the hip B. Aspiration of the knee C. MRI of the knee D. Observation and weight reduction counseling E. Ultrasound of the right hip joint A. This choice is correct because AP and lateral x-rays of the hip are needed to diagnose a slipped capital femoral epiphysis, which is considered an emergency. This patient's age group, his obesity, and the description of the external rotation of the right leg when the hip is flexed all suggest this diagnosis. B. This choice is incorrect because aspiration and drainage via arthroscopy is done for a septic joint. Based on the physical exam and vital signs, the patient is unlikely to have an infection of the bone or knee joint. C. This choice is incorrect because MRI is used to diagnose an acute hematogenous osteomyelitis. This child is not febrile and has no localized bone pain and is therefore less likely to have osteomyelitis. D. This choice is incorrect because observation is not optimal when the patient is likely to have an urgent condition that needs to be fixed. Weight reduction counseling might be a good option after his problem is addressed. E. This choice would be correct if septic arthritis was high on the differential. If effusion is detected, and suspicion for septic arthritis of the hip were high (for example: fever, non weight bearing, and elevated WBC and ESR/CRP), fluid should be aspirated for culture.

A 16-year-old homeless female presents with low-grade fever and abdominal pain. The patient reports recent unprotected sex. Abdominal examination reveals tenderness to palpation in the lower abdominal region, but no masses are felt. Pelvic examination reveals whitish cervical discharge and cervical motion tenderness. The discharge is sent for culture, and a pregnancy test is negative. What is the next best step in management?

A. Abdominal CT B. Arrange for hospitalization C. Begin oral antibiotics and IM antibiotics and treat her partner D. Pelvic ultrasound E. Surgical consult A. This answer choice is incorrect. Abdominal CT is not helpful in diagnosing PID. An argument could be made for an ultrasound if ovarian torsion were suspected, or if her symptoms were refractory to treatment, suggesting a tubo-ovarian abscess that could be detected by ultrasound. B. This answer choice is correct. This patient has signs and symptoms of PID. Cervical discharge should be tested for gonorrhea and chlamydia and sent for culture. As she is a homeless patient, she is at high risk for failure to complete her antibiotic course. Given the deleterious sequelae of incompletely treated PID, she should be hospitalized in order to ensure a full course of treatment. C. This answer choice is incorrect. Although most patients with PID can be discharged home with oral antibiotics, this patient is homeless and is therefore at higher risk for failure to complete a course of antibiotics. In addition, oral antibiotics alone, without an IM treatment, would not be the preferred course of therapy for the patient or her partner (unless her partner was unlikely to present for treatment). D. This answer choice is incorrect. Although pelvic ultrasound is helpful in identifying pelvic disease processes, this patient has clinical signs and symptoms of pelvic inflammatory disease (PID). Treatment of PID can be initiated without further diagnostic workup. E. This answer choice is incorrect. Although the patient exhibits abdominal pain, further workup is required before calling a surgical consult. As the pregnancy test is negative, a ruptured ectopic pregnancy is not an acute concern at this time.

A 16-year old female presents to the ED with abdominal pain. Upon questioning, the patient notes that the pain is consistently in the RLQ without radiation. She reports no dysuria, hematuria, diarrhea, or hematochezia. She has a history of multiple sexual partners and inconsistent condom use. She does not use any other contraceptive measures. She believes her last menstrual period was 3 weeks ago, but she is unsure. She has no history of abdominal or pelvic surgeries. Her temperature is 100.8 F, pulse is 85 bpm, respiratory rate is 12 bpm, and blood pressure is 110/70 mmHg. Her abdominal exam is notable for involuntary guarding, tenderness to palpation in the RLQ without rebound tenderness, and no CVA tenderness. Her pelvic exam is notable for cervical motion tenderness with some discharge. What is the best NEXT step in management?

A. Abdominal CT B. Cervical cultures C. Empiric antibiotics D. Pelvic ultrasound E. Pregnancy test A. This answer choice is incorrect. Abdominal CT may be necessary as a part of the future workup if initial workup is negative. However, the history and physical suggest pelvic inflammatory disease, and CT would not be a part of the initial workup. Importantly, given the history of multiple sexual partners, inconsistent birth control use, and uncertain LMP, pregnancy is a possibility, and a CT would be harmful to the fetus. B. This answer choice is incorrect. While cervical cultures are typically performed, looking specifically for N. gonorrhoeae or C. trachomatis, a pregnancy test is still performed first. This is done to exclude other possible diagnoses and guide inpatient versus outpatient management. C. This answer choice is incorrect. Clinicians will often begin empiric antibiotics immediately based on this presentation, as it is suggestive of PID. However, a pregnancy test is necessary to determine if admission for IV antibiotics is necessary rather than outpatient treatment. D. This answer choice is incorrect. Pelvic ultrasound is a useful technique to check for tubo-ovarian abscess or pregnancy (including ectopic), but a pregnancy test is still the best first choice. E. This answer choice is correct. A pregnancy test is the best first step in management. Pregnancy is one of the indications for inpatient management of PID, so this is very important information when determining whether to admit the patient from the ED or to provide outpatient treatment. In addition, given the patient's uncertainty around the timing of her LMP, this could be a presentation of an early ectopic pregnancy if her pregnancy test is positive, making this test important in directing further workup. While cervical cultures and empiric antibiotics are obviously a must when you suspect PID, pregnancy test is the first step, and the best answer.

A 9-year-old female is brought to clinic by her mother because of two days of abdominal pain and vomiting. She has vomited six times today and has had decreased appetite, but no diarrhea, fevers, sick contacts, or changes in her diet. Her mom states that she has been otherwise healthy apart from increased thirst and occasional bedwetting over the last few weeks. Of note, the patient's maternal grandmother suffers from celiac disease. On exam, the patient is afebrile, has a heart rate (HR) of 180 bpm, BP 90/60 mmHg, RR 50 bpm, and O2 saturation of 98%. She is lying in bed appearing slightly drowsy, taking rapid, deep breaths, and is slow to respond to questions. Her heart and lung exams are normal apart from being tachycardic, and her abdominal exam reveals mild diffuse tenderness to palpation with no rebound or guarding. Which of the following would be the most appropriate next step in management?

A. Abdominal ultrasound B. AP and lateral chest x-ray C. Fingerstick blood glucose D. Gastric lavage E. Midstream urine culture B. A chest x-ray would be appropriate if bacterial pneumonia were high on your differential. A patient with pneumonia generally presents with fever, cough, tachypnea, and will likely have characteristic lung findings, such as crackles, on exam. The patient may experience abdominal pain secondary to pleural inflammation; however, vomiting is not a common presentation. The absence of fever, cough, sick contacts, and lung findings on exam make pneumonia a less likely diagnosis. C. Obtaining a fingerstick blood glucose is the diagnostic step with the highest yield since the patient's clinical picture is strongly indicative of diabetic ketoacidosis (DKA). DKA is a condition more closely associated with type 1 (rather than type 2) diabetes, and is formally diagnosed if a random glucose is > 200 mg/dL, venous pH is < 7.3 and/or bicarbonate < 15 mEq/L, and there is ketonemia or ketonuria. Patients in DKA can present with abdominal pain and vomiting secondary to metabolic acidosis that stems from ketonemia and lactic acidosis. Furthermore, osmotic diuresis from hyperglycemia may contribute to dehydration, which can manifest as tachycardia, dry mucous membranes, delayed capillary refill, and altered mental status. In an attempt to compensate for the metabolic acidosis, the patient may also present as tachypneic with characteristic Kussmaul respirations (rapid, deep breaths). This patient's history of polydipsia, enuresis, and family history of autoimmune disease (including celiac disease and Hashimoto's thyroiditis) suggest that the patient has type 1 diabetes. Her current vital signs and general state of lethargy also point towards DKA and should be confirmed with a fingerstick glucose (in addition to other tests). D. Gastric lavage may be the indicated next step of management if toxin ingestion is the confirmed diagnosis. Patients who have ingested a toxic substance present with a variety of signs and symptoms, but are usually afebrile and obtunded. They may also present with signs of dehydration secondary to the toxin-induced vomiting. In a patient with salicylate toxicity, tachypnea is a common presentation. Assessing exposure to toxins is an important part of a pediatric history. If concern arises, the patient's urine should be sent for toxin-evaluation to insure and clarify the diagnosis. Given this patient's overall picture, DKA is the more likely diagnosis. Urine culture would be appropriate if pyelonephritis were high on your differential. A patient with pyelonephritis may present with a history of fever, dysuria, urinary frequency, CVA tenderness, and vomiting. However, this patient's overall clinical picture does not support the diagnosis, since the patient is afebrile without a history of dysuria or classic CVA tenderness.

A 3-year-old boy presents to the ED 20 minutes after his parents witness an episode of convulsions at home. His parents report that Charlie was in his usual state of good health until three days ago when he developed fever, cough, and rhinorrhea. This evening they found him in bed with his eyes rolled upward, jerking all four of his extremities uncontrollably. He was unarousable from this state, which self-resolved after about two minutes. This has never happened before. Currently, Charlie is sleepy but arousable and complains of nausea. His vitals include T 103.2 F, P 112 bpm, BP 100/60 mmHg, RR 22 bpm, O2 sat 99% on room air. Aside from rhinorrhea and erythematous mucous membranes, the remainder of his physical exam is unremarkable. What is the next best step in management?

A. Abdominal ultrasound B. Administration of valproic acid C. EEG D. MRI brain E. Workup for source of fever A. This answer is incorrect. Acute abdomen is low in the differential in the setting of the presenting symptoms of cough, rhinorrhea, and fever. B. This answer is incorrect, because the majority of patients with febrile seizures do not require treatment with anticonvulsants. This is especially true if this is Charlie's first febrile seizure. Anticonvulsants such as valproic acid may rarely be indicated, but generally these drugs are not recommended in the setting of febrile seizures because they are associated with serious side effects. C. This answer is incorrect because EEGs are indicated in recurrent, focal, or complex seizures. EEG is not indicated after one episode of a simple, febrile seizure. An EEG may eventually be useful to evaluate Charlie's situation, especially if he has another seizure in the ED or if his febrile seizures recur in the future; however, identification of the source of the fever should be the first priority. In addition, interictal EEGs are only positive in 60% of children with epilepsy. D. This answer is incorrect. In the differential diagnosis of a self-limited generalized seizure in the setting of fever in a child of this age group, a febrile seizure is higher on the differential than a brain mass. E. This is the correct answer because Charlie likely experienced a febrile seizure in the setting of an infection. These are relatively common occurrences in Charlie's age range (6 months to 5 years), and the first priority would be to identify the source of fever and treat it.

A 3-year-old boy presents to the ED 20 minutes after his parents witness an episode of convulsions at home. His parents report that Charlie was in his usual state of good health until three days ago when he developed fever, cough, and rhinorrhea. This evening they found him in bed with his eyes rolled upward, jerking all four of his extremities uncontrollably. He was unarousable from this state, which self-resolved after about two minutes. This has never happened before. Currently, Charlie is sleepy but arousable and complains of nausea. His vitals include T 103.2 F, P 112 bpm, BP 100/60 mmHg, RR 22 bpm, O2 sat 99% on room air. Aside from rhinorrhea and erythematous mucous membranes, the remainder of his physical exam is unremarkable. What is the next best step in management?

A. Abdominal ultrasound B. Administration of valproic acid C. EEG D. MRI brain E. Workup for source of fever Charlie likely experienced a febrile seizure in the setting of an infection. These are relatively common occurrences in Charlie's age range (6 months to 5 years), and the first priority would be to identify the source of fever and treat it. A. This answer is incorrect. Acute abdomen is low in the differential in the setting of the presenting symptoms of cough, rhinorrhea, and fever. B. This answer is incorrect, because the majority of patients with febrile seizures do not require treatment with anticonvulsants. This is especially true if this is Charlie's first febrile seizure. Anticonvulsants such as valproic acid may rarely be indicated, but generally these drugs are not recommended in the setting of febrile seizures because they are associated with serious side effects. C. This answer is incorrect because EEGs are indicated in recurrent, focal, or complex seizures. EEG is not indicated after one episode of a simple, febrile seizure. An EEG may eventually be useful to evaluate Charlie's situation, especially if he has another seizure in the ED or if his febrile seizures recur in the future; however, identification of the source of the fever should be the first priority. In addition, interictal EEGs are only positive in 60% of children with epilepsy. D. This answer is incorrect. In the differential diagnosis of a self-limited generalized seizure in the setting of fever in a child of this age group, a febrile seizure is higher on the differential than a brain mass.

A woman brings her 8-year-old son to the pediatrician after witnessing him stare blankly into the distance at dinner the previous week. He was unresponsive to her calling his name or any other stimuli, and it lasted for about 10 or 20 seconds. His teacher reports he does seem to daydream often in class but is able to keep up with schoolwork and excels in his studies. She doesn't note him being disruptive or impulsive in class. His mother is concerned about these blank stares and unresponsive episodes. Which of the following is the most likely diagnosis?

A. Absence seizure B. Atonic seizure C. Complex partial seizure D. Generalized tonic-clonic seizur E. Simple partial seizure A. The patient is having absence seizures. Absence seizures are characterized by loss of awareness of surroundings ("blank stare" or "in another world") and automatisms (e.g., eye-fluttering or lip-smacking). These patients do not lose consciousness or have loss of tone. Absence seizures should also be differentiated from ADHD, since children with ADHD also can be inattentive or seem to be daydreaming. However, since the patient in the vignette still does well in school and does not have other signs of ADHD, it is most likely an absence seizure. An EEG will confirm the diagnosis. B. Atonic seizure, also called drop seizures or drop attacks, consists of loss of motor tone, where patients have a brief lapse in muscle tone. These seizures can occur while walking, standing, or sitting, and have a characteristic head drop (the neck muscles releasing). These seizures last about fifteen seconds. The patient in the vignette did not have symptoms of an atonic seizure. C. Complex partial seizures have all the characteristics of simple complex seizures with the addition of loss of consciousness. D. Generalized tonic-clonic seizure is the most common type of seizure seen in children. These seizures begin acutely with tonic (rigid) stiffening of all extremities and upward deviation of the eyes. The tonic phase is followed by clonic jerks of all the extremities. The patient then loses consciousness and may have urinary incontinence. The patient in this vignette did not have movement of his extremities and did not have urinary incontinence. E. Simple partial seizure is a seizure localized to a small region of the brain. Patients do not lose consciousness. Based on the region of the brain the seizure is occurring in, the patient will exhibit focal symptoms (e.g., altered hearing or smelling, labored speech, etc.). If patients have motor symptoms, it is usually localized to one extremity.

CASE 19-- A 16-month-old male is brought to the urgent care clinic by his father because of a seizure 30 minutes ago. The child dropped to the floor with loss of consciousness and had sporadic twitchy movements of his legs and arms that lasted for five minutes. He had URI symptoms for the past two days, with a fever to 39.4 C (103 F) today. He is previously healthy and had normal developmental screening at his last visit. Neither parent has a seizure disorder, but the child's mother had a single seizure as a young girl after developing a high fever with a cold. Which of the following is the most likely diagnosis in this patient?

A. Absence seizure B. Complex febrile seizure C. Cyanotic breath-holding spell D. Epilepsy E. Simple febrile seizure A. Choice A is incorrect because epilepsy is a recurring condition, while this seizure is this infant's first episode. In addition, the infant has had URI symptoms with a high fever, likely leading to a febrile seizure. Epilepsy seizures are usually classified as unprovoked. If this is indeed a simple febrile seizure, the child's risk of developing epilepsy in the future is about 0.5-1% above the normal risk. If this child continues to have febrile seizures or starts experiencing complex febrile seizures, the risk for epilepsy in the future is even greater. Choice D is incorrect because absence seizures are generalized seizures, but children recover consciousness much more quickly, usually within 30 seconds, compared to tonic-clonic seizures, which take much longer. Absence seizures are most often seen in children starting from the age of 3 and are not associated with loss of tone. Potential causes include hyperventilation or photic stimulation. Given that this child's seizure occurred after URI symptoms and high fever and also was characterized by tonic-clonic jerks lasting about five minutes, this is not the most likely diagnosis. B. Choice B is incorrect because a complex febrile seizure is characterized by > 15 minute duration, occurring more than once during a 24-hour period, or being focal. Given that this child's seizure was associated with loss of consciousness and tonic-clonic movements lasting five minutes, and occurred only once during a 24-hour period, complex febrile seizure is not the most likely diagnosis. C. Choice C is incorrect. Breath-holding spells are most commonly seen in infants from 6 months to 6 years of age. The pathogenesis is not entirely understood but may involve autonomic nervous system dysfunction. In a cyanotic breath-holding spell the infant becomes angry or upset and there is a period of crying, followed by breath-holding in a forced expiration state that may lead the child to lose consciousness and become cyanotic. The child usually recovers soon thereafter with no residual side effects. Given that this child was not reported to have been angry or crying prior to losing consciousness and did not become cyanotic, and because he demonstrated tonic-clonic jerking movements for five minutes, a breath-holding spell would not be a likely diagnosis. D. Choice D is incorrect because epilepsy is a recurring condition, while this seizure is this infant's first episode. In addition, the infant has had URI symptoms with a high fever, likely leading to a febrile seizure. Epilepsy seizures are usually classified as unprovoked. If this is indeed a simple febrile seizure, the child's risk of developing epilepsy in the future is about 0.5-1% above the normal risk. If this child continues to have febrile seizures or starts experiencing complex febrile seizures, the risk for epilepsy in the future is even greater. E. Choice E is correct. Febrile seizures are one of the most common causes of seizures in children. Simple febrile seizures are more common than complex febrile seizures and are characterized by < 15 minutes duration, occurring only once in a 24-hour period, and are generalized (in this patient's case, generalized tonic-clonic). Febrile seizures are usually self-limited events triggered by an acute febrile illness. A positive family history for febrile seizures in the parents makes it a more likely diagnosis in their children. This particular diagnosis fits this patient the best.

Case 19-- A 16-month-old male is brought to the urgent care clinic by his father because of a seizure 30 minutes ago. The child dropped to the floor with loss of consciousness and had sporadic twitchy movements of his legs and arms that lasted for five minutes. He had URI symptoms for the past two days, with a fever to 39.4 C (103 F) today. He is previously healthy and had normal developmental screening at his last visit. Neither parent has a seizure disorder, but the child's mother had a single seizure as a young girl after developing a high fever with a cold. Which of the following is the most likely diagnosis in this patient?

A. Absence seizure B. Complex febrile seizure C. Cyanotic breath-holding spell D. Epilepsy E. Simple febrile seizure Febrile seizures are one of the most common causes of seizures in children. Simple febrile seizures are more common than complex febrile seizures and are characterized by < 15 minutes duration, occurring only once in a 24-hour period, and are generalized (in this patient's case, generalized tonic-clonic). Febrile seizures are usually self-limited events triggered by an acute febrile illness. A positive family history for febrile seizures in the parents makes it a more likely diagnosis in their children. This particular diagnosis fits this patient the best. A. Choice A is incorrect because epilepsy is a recurring condition, while this seizure is this infant's first episode. In addition, the infant has had URI symptoms with a high fever, likely leading to a febrile seizure. Epilepsy seizures are usually classified as unprovoked. If this is indeed a simple febrile seizure, the child's risk of developing epilepsy in the future is about 0.5-1% above the normal risk. If this child continues to have febrile seizures or starts experiencing complex febrile seizures, the risk for epilepsy in the future is even greater. B. Choice B is incorrect because a complex febrile seizure is characterized by > 15 minute duration, occurring more than once during a 24-hour period, or being focal. Given that this child's seizure was associated with loss of consciousness and tonic-clonic movements lasting five minutes, and occurred only once during a 24-hour period, complex febrile seizure is not the most likely diagnosis. C. Choice C is incorrect. Breath-holding spells are most commonly seen in infants from 6 months to 6 years of age. The pathogenesis is not entirely understood but may involve autonomic nervous system dysfunction. In a cyanotic breath-holding spell the infant becomes angry or upset and there is a period of crying, followed by breath-holding in a forced expiration state that may lead the child to lose consciousness and become cyanotic. The child usually recovers soon thereafter with no residual side effects. Given that this child was not reported to have been angry or crying prior to losing consciousness and did not become cyanotic, and because he demonstrated tonic-clonic jerking movements for five minutes, a breath-holding spell would not be a likely diagnosis. D. Choice D is incorrect because epilepsy is a recurring condition, while this seizure is this infant's first episode. In addition, the infant has had URI symptoms with a high fever, likely leading to a febrile seizure. Epilepsy seizures are usually classified as unprovoked. If this is indeed a simple febrile seizure, the child's risk of developing epilepsy in the future is about 0.5-1% above the normal risk. If this child continues to have febrile seizures or starts experiencing complex febrile seizures, the risk for epilepsy in the future is even greater.

Case 1-- A 33-year-old G1P0 female with a history of medically controlled seizures gives birth vaginally to a boy with IUGR at 38 weeks' gestation. The newborn is noted to have dysmorphic cranial features and his head circumference is 28.5 cm (< 5th percentile). What is another associated abnormality you might expect to see in this newborn?

A. Absent red reflex B. Cardiac defects C. Chorioretinitis D.Hepatosplenomegaly E. Tremors The mother was on an anticonvulsant for her seizures. Taking anticonvulsants during pregnancy may lead to cardiac defects, dysmorphic craniofacial features, hypoplastic nails and distal phalanges, IUGR, and microcephaly. Mental retardation may be seen. A rare neonatal side effect is methemoglobinuria. A. This answer choice is incorrect. The red reflex is the normal reddish-orange reflection of light from the eye's retina that is observed when using an ophthalmoscope. An absent red reflex in a newborn can be due to congenital cataracts or a retinoblastoma, it is not caused by maternal anticonvulsant use. C. This answer choice is incorrect. Chorioretinitis in a newborn may be due to congenital toxoplasmosis and CMV infections. It is not caused by maternal anticonvulsant use. D. This answer choice is incorrect. Hepatosplenomegaly in newborns is seen in metabolic diseases, storage diseases, HIV vertical transmission, intrinsic liver disease, and in congenital infections. Maternal anticonvulsant use does not cause hepatosplenomegaly. E. This answer choice is incorrect. Tremors in a newborn can be due to maternal substance use. They would not result from maternal use of anticonvulsants.

A 15-year-old female is brought to the ER with a chief concern of abdominal pain for three hours. She rates her pain at 8/10 and describes it as constant and located mainly in the middle of her abdomen. It is worse with coughing and moving. She had two episodes of nonbilious, non-bloody vomiting. She is sexually active with her boyfriend of three months and always uses condoms. Her last menstrual period was two weeks ago. Vital signs are normal. On exam, she exhibits involuntary guarding, mild rebound tenderness, and tenderness to palpation between her right anterior superior iliac spine and umbilicus. On pelvic exam, she reports tenderness when attempting to palpate her right adnexa, but no masses are appreciated and there is no cervical motion tenderness. Laboratory studies reveal normal complete blood count and c reactive protein. Which of the following is the most likely diagnosis in this patient?

A. Appendicitis B. Ectopic pregnancy C. Ovarian torsion D. Pelvic inflammatory disease A. Appendicitis is the most common condition in pediatric patients requiring immediate surgical intervention, but often (especially in infants) presents differently than in adults. Aspects of their atypical presentation include lack of migration of pain to the RLQ, negative Rovsing's sign, and involuntary guarding and fever without perforation. School-age children who can articulate the pain often describe pain with movement or coughing. Also, rebound tenderness was found to be neither sensitive nor specific in the pediatric population, while in the adult population it is one of the most accurate PE findings (86%). This patient's sudden onset of intense pain at the umbilicus with vomiting, anorexia, and tenderness at McBurney's point are all classic findings of appendicitis. The more atypical signs include diffuse pain centered below the umbilicus, and rebound tenderness that might point to a perforation (more likely, it is part of the atypical pediatric presentation given her normal WBC). Another atypical aspect of her exam is her adnexal pain during the pelvic exam, which could be due to the degree of inflammation and the positioning of her appendix. The key take-away point is to have a high index of suspicion for appendicitis in pediatric patients with abdominal pain given their atypical presentation. B. Patients with an ectopic pregnancy typically present with painless vaginal bleeding six to eight weeks after their last menstrual period. Associated abdominal pain is described as crampy pelvic pain, and it is often associated with nausea. Diffuse abdominal pain is also present if rupture and intraperitoneal bleeding occurs. Given that the patient is two weeks from her last menstrual period, she reports no vaginal bleeding, and her pain is intense and located in the middle of her abdomen, an ectopic pregnancy is less likely. C. Ovarian torsion is more common in the post-menopausal population, though it can present in any age group. It is described as intermittent stabbing pain in the lower abdomen or pelvis. Torsion is often secondary to an ovarian mass—such as a neoplasm or a corpus luteal cyst—which may occasionally be appreciated on exam. Nausea and vomiting are very common findings as well. Ultrasound is essential to initial workup. Given the patient's pain localized around her umbilicus, her tenderness at McBurney's point, and lack of palpable masses on pelvic exam, ovarian torsion is a less likely diagnosis. D. Pelvic inflammatory disease is on the differential given the patient's sexual history, and adnexal pain. However, this pain is often post-coital and also often occurs during or immediately following menstruation. Another key finding is mucopurulent discharge and cervical motion tenderness, both of which are absent in this patient. RUQ pain and a fever (present in 50% of patients with PID) are other signs that are not reported in this case.

Case 9-- A two-month-old infant is brought to clinic by her mother for a well-baby checkup. Mom says that her daughter is easy to care for because "she rarely cries and sleeps most of the time." On exam, the patient has a yellow tint to the skin, decreased muscle tone, and a large anterior fontanel. What is the most likely diagnosis in this patient?

A. Abusive head trauma B. Congenital adrenal hyperplasia C. Congenital hypothyroidism D. Neonatal lupus E. Sepsis Congenital hypothyroidism may not be clinically evident until 6 weeks of age due to circulating maternal thyroid hormone transmitted from the placenta. Signs and symptoms of congenital hypothyroidism include feeding problems, large fontanelles, hypotonia, large tongue, coarse cry, hyperbilirubinemia, and umbilical hernia. Congenital hypothyroidism should be identified on routine neonatal screening. A. This answer choice is incorrect. Abusive head trauma may result in significant head injury, including subdural hemorrhage, hypoxic/ischemic brain injury, and retinal hemorrhage. B. This answer choice is incorrect. Infants with congenital adrenal hyperplasia (CAH) often have abnormal genitalia (females), poor feeding, vomiting, dehydration, and electrolyte changes. D. This answer choice is incorrect. Neonatal lupus should be considered if the infant's mother has lupus. Cutaneous findings may be present at birth or may develop within the first 2 to 5 months of life. These findings include erythematous plaques, telangiectasias, or atrophic lesions. Affected infants may have cardiac abnormalities or conduction deficits (heart block). Hematologic disturbances may occur within the first 2 weeks of life. E. This answer choice is incorrect. Sepsis indicates significant infection of the infant. Signs and symptoms may include body temperature change, changes in respiration, increased or decreased heart rate, reduced movement, reduced feeding, low blood sugar, seizures, and jaundice.

A 12-month-old previously healthy girl presents with cough and mild subcostal retractions. She is afebrile, and physical exam reveals asymmetric wheezing. Chest x-ray demonstrates unilateral air trapping. What is the most likely diagnosis?

A. Acute bronchiolitis B. Asthma C. Croup D. Foreign body aspiration E. Pneumonia This patient displays the characteristics of foreign body aspiration including unilateral wheeze and air trapping in one lung indicating unilateral airway obstruction. The right main bronchus is more commonly obstructed due to anatomy (it is wider and more vertical than the left). The most commonly aspirated foods are hot dogs, nuts, hard candy, grapes, and popcorn. Foreign body aspiration also commonly presents in children younger than age four. A. This answer choice is incorrect. Acute bronchiolitis is a good differential, especially as this is the most common cause of wheezing in infant; however, if this were the diagnosis, the patient would most likely be febrile with bilateral wheezes, and chest x-ray would demonstrate scattered atelectasis and/or diffuse opacities from bronchial obstruction. B. This answer choice is incorrect. The physical exam finding of wheezing is consistent with asthma, but this patient has wheezing only on one side. Along those lines, chest x-ray in an asthmatic patient would demonstrate global air trapping with hyperinflated lungs, rather than unilateral findings. C. This answer choice is incorrect. Croup involves subglottic inflammation, typically presenting with inspiratory stridor and a "barky cough" (i.e., like a seal). This patient does not have either, and also presents with asymmetric wheezing and air trapping that would be unexpected in an individual with croup. E. This answer choice is incorrect. While this patient presents with cough and increased work of breathing, she is afebrile, and auscultation of the chest does not reveal crackles or focally decreased breath sounds/area of consolidation, which would be consistent with pneumonia. Additionally, chest x-ray findings are not consistent with lobar or multifocal pneumonia.

The father of a 15 year-old-girl calls the on-call provider with concerns of fever, headache and mental status changes over the past 12 hours. She has a fever of 39.4 C (103 F) and is acting dazed and difficult to arouse. On further questioning, the father states that she is breathing heavily, has decreased urine output, and decreased oral intake. He wants advice about whether she should be taken to the Emergency Department. Given the limited history, which of the following is highest on the differential?

A. Acute cystitis B. CNS tumor C. Diabetic ketoacidosis D. Hypoglycemia E. Meningitis A. Choice A is incorrect because acute cystitis is an infection that is usually limited to the lower urinary tract. While acute cystitis can present with fevers and dysuria, it would likely not present with more systemic systems, like tachypnea, altered mental status, and decreased PO intake. Such a systemic presentation would be more characteristic of bacteremia secondary to pyelonephritis. B. Choice B is incorrect because a CNS tumor would likely not result in such an acute presentation of altered mental status. While a CNS tumor may result in tachypnea and lethargy depending upon the location of the tumor, the presentation would likely be a more slow and gradual progression. Furthermore, a patient with a CNS tumor would likely not present with such a high-grade fever, which places a CNS tumor lower on our differential. C. Choice C is incorrect because diabetic ketoacidosis is usually associated with increased (not decreased) urine output secondary to the osmotic effect of ketones and glucose in the urine. Thus, while diabetic ketoacidosis should certainly be on the differential for causes of altered mental status and lethargy, it is not high on the differential given the presentation. D. Choice D is incorrect because hypoglycemia is unlikely to be source of such a high-grade fever (unless the patient had sepsis which resulted in high fever and in hypoglycemia). Furthermore, hypoglycemia would likely not affect urine output over such a significant period of time. While hypoglycemia should certainly be on the differential for causes of altered mental status, the presentation of fever and decreased urine output makes places it lower on our differential. E. Choice E is correct - meningitis is the most likely etiology in our differential given the fever, altered mental status, decreased PO intake, and decreased urine output. The incidence of meningitis has decreased in this patient's age range due to increased vaccinations against the most common causative organisms of meningitis, however, it still remains high on our differential given the presentation of this patient. Further management in the ED should consist of obtaining a more thorough history and physical exam as well as blood cultures and lumbar puncture to establish the diagnosis of meningitis.

The father of a 15 year-old-girl calls the on-call provider with concerns of fever, headache and mental status changes over the past 12 hours. She has a fever of 39.4 C (103 F) and is acting dazed and difficult to arouse. On further questioning, the father states that she is breathing heavily, has decreased urine output, and decreased oral intake. He wants advice about whether she should be taken to the Emergency Department. Given the limited history, which of the following is highest on the differential?

A. Acute cystitis B. CNS tumor C. Diabetic ketoacidosis D. Hypoglycemia E. Meningitis meningitis is the most likely etiology in our differential given the fever, altered mental status, decreased PO intake, and decreased urine output. The incidence of meningitis has decreased in this patient's age range due to increased vaccinations against the most common causative organisms of meningitis, however, it still remains high on our differential given the presentation of this patient. Further management in the ED should consist of obtaining a more thorough history and physical exam as well as blood cultures and lumbar puncture to establish the diagnosis of meningitis. A. Choice A is incorrect because acute cystitis is an infection that is usually limited to the lower urinary tract. While acute cystitis can present with fevers and dysuria, it would likely not present with more systemic systems, like tachypnea, altered mental status, and decreased PO intake. Such a systemic presentation would be more characteristic of bacteremia secondary to pyelonephritis. B. Choice B is incorrect because a CNS tumor would likely not result in such an acute presentation of altered mental status. While a CNS tumor may result in tachypnea and lethargy depending upon the location of the tumor, the presentation would likely be a more slow and gradual progression. Furthermore, a patient with a CNS tumor would likely not present with such a high-grade fever, which places a CNS tumor lower on our differential. C. Choice C is incorrect because diabetic ketoacidosis is usually associated with increased (not decreased) urine output secondary to the osmotic effect of ketones and glucose in the urine. Thus, while diabetic ketoacidosis should certainly be on the differential for causes of altered mental status and lethargy, it is not high on the differential given the presentation. D. Choice D is incorrect because hypoglycemia is unlikely to be source of such a high-grade fever (unless the patient had sepsis which resulted in high fever and in hypoglycemia). Furthermore, hypoglycemia would likely not affect urine output over such a significant period of time. While hypoglycemia should certainly be on the differential for causes of altered mental status, the presentation of fever and decreased urine output makes places it lower on our differential.

A 6-month-old female is brought into the pediatrician's office for three days of high fever, fussiness, and decreased appetite. The patient has not had any upper respiratory tract symptoms, vomiting, diarrhea, or rash. On physical exam the patient is fussy, has a RR of 28 bpm and a pulse of 160 bpm. She is febrile to 102.8 F (rectal). The patient is alert, not toxic appearing, and fully moving all extremities. Apart from her vital signs, no other significant exam findings are noted. A CBC demonstrates leukocytosis of 17.0 cells x 103 / µL with elevated bands. Of the following, which diagnosis is most likely?

A. Acute otitis media B. Bacterial meningitis C. Measles D. Roseola E. Urinary tract infection A. This answer choice is incorrect. Fever and fussiness can be possible signs of AOM in infants, however, there is no evidence of infection on this patient's physical exam such as inflamed, bulging erythematous tympanic membranes. B. This answer choice is incorrect. Although meningitis should always be on the differential for an infant with fever and fussiness, this patient is non toxic appearing, and does not have other exam findings suggestive of meningitis (such as bulging fontanelle, lethargy, extreme irritability). Definitive testing such as lumbar puncture may be ordered if there is increasing suspicion for meningitis, and caretakers should be counseled on the signs and symptoms of meningitis. At this time, however, the infant does not demonstrate signs that meningitis is the most likely explanation for the infant's presentation. C. This answer choice is incorrect. Measles typically begins with a "prodrome" period featuring the "3 Cs" (cough, coryza, conjunctivitis) along with high fever, often > 104 F, and general malaise and anorexia. On the 2nd to 4th day a maculopapular erythematous rash appears starting on the face/upper neck and spreading downward. Although infants receive their first vaccination against measles (the MMR) at 1 year of age, infants are generally protected unless they are exposed to older, unimmunized children who have the disease. D. This answer choice is incorrect. Roseola often presents with high fever for several days. Fever then abates as the classic rash appears. It is a diagnosis of exclusion at this point and should not preclude obtaining a urine sample in this child. E. This answer choice is correct. UTI, the most common bacterial illness in a female infant, is consistent with her high fever, fussiness, and decreased appetite. Her CBC suggests that she has a bacterial infection (leukocytosis and elevated bands). A sample of her urine should be obtained by catheterization and sent for urinalysis and culture.

An 18-month-old presents with yellow and poorly mobile tympanic membranes. Four months prior he presented then with several days of nasal congestion, cough, decreased eating, and ear tugging. His exam then revealed a red, nonmobile tympanic membrane and he was treated with amoxicillin. Based on the history and physical exam, what is the most likely diagnosis now?

A. Acute otitis media B. Mastoiditis C. Otitis externa D. Otitis media with effusion E. Viral encephalitis The earlier diagnosis of acute otitis media together with current findings of bilateral yellow and poorly mobile tympanic membranes on physical exam make OME the most likely diagnosis. A. This answer choice is incorrect. The time course of development of this infection over four months makes this diagnosis unlikely. In addition, with AOM, there is usually a sign of inflammation such as erythema of the tympanic membrane. B. This answer choice is incorrect. Although mastoiditis presents with tympanic membrane changes, the signs of bilateral yellow and poorly mobile tympanic membranes are more suggestive of otitis media with effusion. C. This answer choice is incorrect. Although otitis externa presents with otalgia, the sign of bilateral yellow and poorly mobile tympanic membranes are more suggestive of otitis media with effusion. E. This answer choice is incorrect. Viral encephalitis typically causes neurologic dysfunction with signs that include fever, headache, nausea, vomiting, photophobia, altered level of consciousness, seizures, and possibly focal neurologic signs. This child's presentation is much more consistent with otitis media with effusion.

A 5-week-old infant presents to clinic with 4 days of repeated, non-bilious, non-bloody vomiting with non-bloody diarrhea. He has 8 to 9 episodes of vomiting per day immediately following breastfeeding. The episodes started 4 days after the entire family suffered from severe viral gastroenteritis. His birth history is uncomplicated (full term, NSVD, unremarkable 30-week ultrasound) and birth weight was 3.6 kg (50th percentile). On exam, his vitals are: T 36.7°C, HR 185, BP 85/45, RR 36, Wt 4.1 kg (25th percentile). On exam, his eyes are moderately sunken without production of tears, his lips are cracked, and his throat is without erythema. His capillary refill is ~3 seconds, and his pulse is thready. He appears lethargic. What is your first step in management?

A. Administer 75 mL/kg of oral rehydration solution over 3-4 hours and 60-120 mL of oral rehydration solution for every episode of vomiting. B. Close observation in the office for 6 hours and encourage PO intake until vitals normalize. C. Intravenous 20 mL/kg boluses of ¼ normal saline solution until baseline clinical status is achieved, then closely monitor vitals for 6 hours while encouraging PO formula intake. D. Intravenous lactated Ringer's solution of 20mL/kg boluses until baseline clinical status is achieved, then either oral hydration or IV fluid hydration if not tolerating PO. E. Observe for 6 hours with normal PO intake and administer 60-120 mL of oral rehydration solution for every episode of vomiting. E. Observe for 6 hours with normal PO intake and administer 60-120 mL of oral rehydration solution for every episode of vomiting. A. This answer choice is incorrect. This infant is severely dehydrated and IV rehydration is key to initial fluid resuscitation in the severely dehydrated child, oral rehydration would not be sufficient. However, this plan would be appropriate for mild to moderately dehydrated patients. B. This answer choice is incorrect. This infant has severe dehydration as evidenced by lethargy, poor PO intake, tachycardia, weak pulses, and deeply sunken eyes. This child needs immediate IV fluids as opposed to observation and encouragement of PO intake. C. This answer choice is incorrect. One quarter normal saline (1/4 NS) is a hypotonic solution and is not used as fluid bolus therapy to achieve normal perfusion. The recommended therapy to correct severe dehydration is to give 20 mL/kg boluses of isotonic solution and to reassess for clinical improvement following each administration. Once the patient is stable and back to baseline, then continue IV hydration with 5% dextrose 0.9% normal saline OR give 100 mL/kg/day of oral rehydration solution. Providers should also factor in replacement of ongoing losses. D. This answer choice is correct. This patient has severe dehydration and. should be treated with Lactated Ringer's solution or normal saline in up to three 20ml/kg boluses until clinical baseline is achieved (normal pulse, perfusion and mental status). Once stabilized, oral rehydration may be attempted. If the patient still cannot take PO, then IV hydration with 5% dextrose and normal saline may be administered. Providers should also factor in replacement of ongoing losses. E. This answer choice is incorrect. Observation alone is not appropriate for a severely dehydrated child. Without abrupt intervention with IV fluid therapy, the child is at risk of developing hypovolemic shock.

A 16-year-old previously healthy girl comes to the Pediatrics Urgent Care Clinic having "almost fainted" at soccer practice. She says that she had not eaten much earlier in the day and it was very hot and muggy outside. She felt light-headed and sick to her stomach. She denies losing consciousness and did not fall to the ground. She denies any chest pain. When you examine her, her eyes are sunken and he is tachycardic. What would be your next step in her management?

A. Administer fluids and fluids and recheck vital signs B. Blood glucose management C. Echocardiogram D. Electrocardiogram E. Stress test The patient is likely dehydrated given dizziness without loss of consciousness in the setting of poor PO intake, hot weather, and exercise. As the symptoms occurred while she was upright, the likely mechanism is vasovagal. Her sunken eyes and tachycardia are signs of moderate to severe dehydration. Since this is a clinical diagnosis, fluids should be given with subsequent rechecking of heart rate and blood pressure to confirm the diagnosis. B. This answer choice is incorrect. While the patient has had poor PO intake, hypoglycemia is relatively uncommon in an otherwise healthy young female. In addition, we would expect symptoms including diaphoresis, anxiety, and tremulousness, as opposed to dizziness. C. This answer choice is incorrect. Obstructive outlet pathology is less likely given the presentation, as no syncopal episode occurred. An echocardiogram may be indicated, but it would not be the next step in the evaluation. D. This answer choice is incorrect. While an EKG would be useful to assess for possible arrhythmia (long QT syndrome, WPW), the patient did not have a syncopal event or chest pain. As a result, an EKG would not be the most appropriate next step in diagnosis. E. This choice is incorrect. A cardiac etiology like hypertrophic cardiomyopathy is less likely given the presentation, and a stress test would not be the next step in the evaluation.

A 10-day-old boy is brought to the ED by his mother because of fever. Mother describes that the baby has been "sleepy" and feeding less vigorously than in the previous two days. She believes his urine output has also decreased. His birth history is notable for prolonged membrane rupture (about 32 hours), and maternal fever at the time of delivery. Prenatal and neonatal ultrasound revealed bilateral hydronephrosis. On exam, the infant is sleepy with a temperature of 38.5 C. A blood sample is sent for CBC, BMP, and culture. Attempts are made to obtain CSF and urine for analysis and culture, but only very small volumes of these fluids are obtained. Volume resuscitation is started. Chest x-ray is performed with indeterminate results. What is the most appropriate next step?

A. Admit for observation and continue supportive care B. Attempt to obtain larger samples. Antibiotics should not be started until all needed results are pending. C. Send samples for culture and begin parenteral antibiotic treatment D. Send samples for gram stains and begin parenteral empiric antibiotic treatment E. Send the urine for urinalysis and the CSF for cell count, glucose and protein and begin parenteral antibiotic A. This answer choice is incorrect. Delay of therapy and observation alone would not be indicated. In an infant younger than one month, fever with any suspicion of sepsis, irrespective of source, requires immediate evaluation and initiation of empiric antibiotic treatment. B. This answer choice is incorrect. Given the consequences of significant bacterial infection in an infant this age, delaying therapy to obtain additional laboratory specimens is not appropriate. C. This answer choice is correct. A neonate who presents with fever along with sleepiness and poor feeding should have a full sepsis evaluation with specimens sent for culture. The infant should also be started on empiric antibiotic therapy immediately after specimen collection. This infant is likely to have a urinary tract infection, and possibly urosepsis especially given his known urinary tract anomalies. Meningitis cannot be ruled out at this time due to the absence of CSF cell count collection, therefore empiric antibiotics should be initiated at meningitic dosing. Urosepsis in this age may lead to bacteremia and in turn meningitis due to the immature blood brain barrier. Only cultures will give us the information required to determine the appropriate type and length of antibiotic therapy. D. This answer choice is incorrect. Although sending samples for gram stain may give an indication of whether an infection is present, it will not give the gold standard of information as would a culture with sensitivities. E. This answer choice is incorrect. Urinalysis and CSF profiles may help us confirm the diagnosis, but if positive in the absence of cultures, may commit the patient to a prolonged course of broad-spectrum and non-specific therapy.

A 5-year-old male comes to the clinic with a chief complaint of four days of progressively worsening fever that has been minimally responsive to acetaminophen. The patient complains of sore throat and decreased appetite. His sister had a positive rapid strep test and is now being treated with amoxicillin. Your concern is for Group A strep. What is the next best step in management?

A. Advise parents to give patient acetaminophen with return precautions B. Chest x-ray C. Rapid strep test with back-up culture if negative D. Send blood cultures E. Start antibiotic treatment A rapid strep test would provide confirmation of your clinical suspicion and allow for correct diagnosis prior to empiric antibiotic treatment. A. This answer choice is incorrect. Since the patient has a history of being exposed to a sick contact with Group A strep, being sent home with acetaminophen is not sufficient. B. This answer choice is incorrect. CXR should not be done for this patient in the absence of respiratory symptoms. D. This answer choice is incorrect. There is no indication for a blood culture at this time. E. This answer choice is incorrect. Although this child may be empirically treat this for infection with Group A strep, a test to diagnose infection should be done prior to initiation of antibiotics.

A 16-year-old female with a history of irregular menses presents to the ED with severe abdominal pain. She has regular unprotected sexual intercourse with multiple male sexual partners. She has experienced fevers, nausea, vomiting, right shoulder pain and reports no vaginal bleeding or discharge. Vitals are T 38.0 C, BP 90/60 mmHg, P 120 bpm, R 20 bpm. Qualitative ß-hCG is positive, and hemoglobin is 7 g/dL. Physical exam reveals that she has difficulty answering questions due to pain. Auscultation of the chest is notable for tachycardia, a murmur, and clear lungs bilaterally. The abdomen is rigid with tenderness throughout, but more so in the right lower quadrant. There is guarding and rebound tenderness. On pelvic exam, there is cervical motion tenderness but no bleeding or masses noted. What is the most likely diagnosis?

A. Appendicitis B. Fitz-Hugh-Curtis syndrome C. Hepatitis D. Pelvic inflammatory disease E. Ruptured ectopic pregnancy A. This answer choice is incorrect. Appendicitis classically presents as right lower quadrant pain. Symptoms include anorexia, nausea, vomiting, and fever. On physical examination, the patient may have peritoneal signs. This patient has low hemoglobin and hemodynamic instability which is unlikely in acute appendicitis. B. This answer choice is incorrect. Fitz-Hugh-Curtis syndrome is a possible complication of pelvic inflammatory disease, a possibility given the patient's sexual history. The abdominal pain would be felt in the right upper quadrant and may be referred to the right shoulder if the peritoneum becomes irritated. Although Fitz-Hugh-Curtis syndrome may occur during pregnancy, Fitz-Hugh-Curtis syndrome would not account for the low hemoglobin, peritonitis, or vital signs suggesting fluid loss/bleeding. C. This answer choice is incorrect. With acute hepatitis, patients may present with anorexia, nausea, vomiting, fatigue, malaise, fever, among other symptoms. Laboratory studies would reveal elevated AST and ALT. History may be positive for intravenous drug use, maternal hepatitis, or travel. The pain would be felt in the right upper quadrant and may be referred to the right shoulder if the peritoneum becomes irritated. It is unlikely the patient would be hemodynamically unstable and severely anemic. D. This answer choice is incorrect. Pelvic inflammatory disease (PID) is a possibility, as indicated by the patient's sexual history and cervical motion tenderness. Symptoms of PID include lower abdominal or pelvic pain, vaginal discharge, fever, fatigue, nausea, vomiting, diarrhea, dysuria, and dyspareunia, among other symptoms. Although PID may occur during pregnancy, it would not account for the low hemoglobin, peritonitis, or vital signs suggesting fluid loss/bleeding. E. This answer choice is correct. The patient likely has a ruptured right-sided ectopic pregnancy, as indicated by the positive pregnancy test, hemodynamic instability, and peritonitis. Hemorrhage into the peritoneum may irritate the peritoneum and cause referred pain to the right shoulder. Cervical motion tenderness may also be found. Patients may experience nausea, vomiting, and fever.

A 4-year-old boy with Down syndrome is brought by his mother to the office for increasing fatigue, intermittent fever, and decreasing appetite for three weeks. He is previously healthy. Physical examination reveals pallor and hepatosplenomegaly. What is the best next step in the management of this child?

A. Advise the mother to use acetaminophen as needed for fever B. Send CBC and peripheral smear C. Send patient for chest x-ray D. Send TSH E. Start the patient on a 7-day course of amoxicillin Fatigue, decreased appetite, anemia, and hepatosplenomegaly are all clinical signs and symptoms that may be associated with acute leukemia, for which patients with Down syndrome are at increased risk. A CBC would be helpful both for assessing the white blood cell count as well as the degree of anemia. A peripheral smear would be indicated to further evaluate for leukemia. A. This choice is incorrect. This patient is likely presenting with signs and symptoms of acute leukemia. Children with Down syndrome are at increased risk for childhood malignancy (specifically ALL). Treating the fevers with acetaminophen is appropriate, but would miss the underlying diagnosis. C. This choice is incorrect. This patient is not presenting with any acute respiratory symptoms to warrant immediate chest x-ray. D. This choice is incorrect. Although Down syndrome may be associated with hypothyroidism, the patient's fever and physical exam findings are not suggestive of hypothyroidism. E. This choice is incorrect. This patient is presenting with signs and symptoms concerning for acute leukemia. Although infection may complicate leukemia, an outpatient course of amoxicillin would not be appropriate therapy at this time.

A 19-year-old female in her 38th week of pregnancy goes into active labor. Within the first few days following birth, her baby is noted to have a high-pitched cry, tremulousness, hypertonicity, and feeding difficulties. The baby is otherwise developmentally normal and the remainder of the physical exam also is normal. Which drug was likely abused by mother during her pregnancy?

A. Alcohol B. Cocaine C. Heroin D. Marijuana E. Tobacco Maternal opiate use during pregnancy places the newborn at risk for neonatal abstinence syndrome. Affected newborns may exhibit CNS findings (irritability, hyperactivity, hypertonicity, incessant high-pitched cry, tremors, seizures), GI symptoms (vomiting, diarrhea, weight loss, poor feeding, incessant hunger, excessive salivation), and respiratory findings (including nasal stuffiness, sneezing, and yawning). A. This answer choice is incorrect. Fetal alcohol syndrome has a distinct pattern of facial abnormalities, growth deficiency, and CNS dysfunction. These infants may also exhibit other neurobehavioral deficits such as poor motor skills and hand-eye coordination and learning problems, such as difficulties with memory, attention, and judgment. B. This answer choice is incorrect. Cocaine use during pregnancy is not typically associated with withdrawal symptoms. Cocaine has been linked to subtle deficits appreciated later in childhood, including deficits in cognitive performance, information processing, and attention to tasks. D. This answer is incorrect. There is limited evidence for a neonatal withdrawal syndrome associated with maternal marijuana use. E. This answer is incorrect. Smoking is not associated with the withdrawal syndrome described above. Smoking has been linked in a dose-dependent manner with low weight newborns at birth. There is a two-fold increase in low birth weight even in light smokers (< 10 cigarettes per day). Smoking during pregnancy also has been associated with subtle neurodevelopmental deficits in some exposed children.

A 19-year-old G1P0 female presents in labor to the ED at 38 gestational weeks. When taking the history, it is discovered that she has had irregular prenatal care, drank a couple of beers every weekend, and smoked 4 cigarettes a day during her pregnancy. She delivers a baby boy who is small for gestational age. On exam, it is noted the baby has microcephaly, a smooth philtrum, and a thin upper lip. What do you suspect caused these features in the baby?

A. Alcohol exposure B. Congenital CMV infection C. Congenital rubella D. Tobacco exposure E. Vertically transmitted HIV Fetal alcohol syndrome has very characteristic facial features, including a smooth philtrum, thinning of the upper lip, and small palpebral fissures. B. This answer choice is incorrect. Symptomatic congenital CMV infection presents with microcephaly, jaundice, hepatosplenomegaly, low birth weight, and petechiae at birth. C. This choice is incorrect. Congenital rubella presents with intrauterine growth restriction, sensorineural deafness, eye abnormalities (retinopathy, cataracts), meningoencephalitis, cardiac defects, interstitial pneumonitis, petechiae, hepatosplenomegaly and jaundice. D. This answer choice is incorrect. While tobacco exposure can cause infants to be small for gestational age they typically do not have any characteristic facial features. E. This answer choice is incorrect. Typically, vertically transmitted HIV does not lead to recognizable symptoms at birth. This diagnosis cannot be completely ruled out without lab testing.

An 8-year-old girl comes to the clinic with two weeks of nasal congestion. Three days ago, she developed a fever of 38.7°C, purulent nasal secretions, malodorous breath, and a nocturnal cough. Examination of the nose reveals bilateral purulent nasal drainage and mild facial tenderness. Which of the following is the most likely diagnosis?

A. Allergic rhinitis B. Asthma C. URI D. Sinusitis E. Middle ear infection The maxillary and ethmoid sinuses are large enough to harbor infection in infancy. The sphenoid sinuses do not become large enough until the third to fifth year of life, and the frontal sinuses are rarely large enough until the sixth to tenth year of life. Sinusitis is characterized by the findings in the question stem and is often preceded by a URI. A. This answer choice is incorrect. Allergic rhinitis is a very common diagnosis in pediatrics, and nocturnal cough is a frequently associated symptom. However, allergies usually cause clear, thin nasal secretions and are usually not associated with fever and malodorous breath. Other physical exam findings associated with allergies may include allergic shiners, cobblestoning of the posterior pharynx, and edematous turbinates. B. This answer choice is incorrect. Asthma, another common diagnosis in pediatrics, is often triggered by a URI and is frequently associated with a nocturnal cough. Fever, purulent nasal secretions, and malodorous breath are not caused by asthma. End-expiratory wheezing, atopic diathesis, and a positive family history would support a diagnosis of asthma. C. This answer choice is incorrect. Viral URIs are very common in childhood. URI will present acutely with sneezing, runny nose, and congestion. Fever may or may not be present. This patient's prolonged symptoms along with fever and purulent nasal drainage make URI less likely. E. This answer choice is incorrect. Acute otitis media is a common diagnosis in pediatrics. It can follow a URI or occur simultaneously with sinusitis. However, this patient does not report ear pain.

A 5-month-old infant presents to urgent care with his mother who states that she witnessed her son stop breathing and turn blue for about 25 seconds. Upon physical stimulation by the mother, the patient began to breathe again. This is the first time she has ever witnessed this happening. The patient's birth and past medical history are unremarkable. Physical exam is unremarkable, vital signs are stable and normal, and lab studies are all within normal limits. Which of the following is LEAST likely to be on the differential diagnosis as a cause for this infant's BRUE (brief resolved unexplained event)?

A. Arrhythmia B. Congenital heart disease C. Gastroesophageal reflux D. Infection E. Seizures A. This answer choice is incorrect. Up to 5% of BRUEs have been shown to be caused by cardiac arrhythmias. Particular arrhythmias may include prolonged QT syndrome, Wolff-Parkinson-White, or bradycardia secondary to congenital heart block. In patients born to mothers with lupus, the suspicion for congenital heart block as a cause of the BRUE should be high. An ECG would be very useful in evaluation of a BRUE patient in which arrhythmia was suspected. B. This answer choice is correct. It is unlikely for a patient with congenital heart disease to first present with a BRUE. This patient has no past medical history and his birth history was unremarkable. Typically, a patient with undiagnosed congenital heart disease will suffer from acute decompensation within the first few weeks of life. These patients will also have growth problems, difficulty with feeding and a murmur is often appreciated on physical exam. Children with undiagnosed Tetralogy of Fallot may have intermittent episodes of cyanosis while blood supply is diverted from the pulmonary vasculature. C. This answer choice is incorrect. Gastroesophageal reflux is diagnosed in up to 30% of patient's presenting with an BRUE. However, there has never actually been a demonstrated link between reflux and apnea. It is postulated that reflux triggers laryngospasm, but treatment of the reflux does not necessarily prevent reoccurrence of the BRUE. Reflux as a cause of BRUE is more likely when the infant has gross emesis or regurgitation during the BRUE. If this patient does have gastroesophageal reflux, he may benefit from a therapeutic trial of H2 blocker. He may also benefit from instituting reflux precautions after meals such as leaving the patient upright for 15 to 30 minutes after eating, and making sure the baby burps after eating. D. This answer choice is incorrect. Infectious etiologies are a common cause of BRUE, particularly infection with RSV or B. pertussis. Patients with infection or sepsis may present with apnea, pallor, tachycardia, fever or hypothermia. They often have poor oral intake and hypotonia. In children less than 1 month of age, common etiologies of infection include Group B strep, E. coli, Streptococcus pneumoniae, listeria, and HSV. A CBC, respiratory virus PCR panel, CXR, and blood cultures would be helpful in evaluation of this patient's BRUE. E. This answer choice is incorrect. Seizures are a relatively common cause of BRUE. A patient with suspected seizure may display some or all of the following associated symptoms: foaming at the mouth, loss of bowel or bladder incontinence, abnormal posturing, abnormal ocular eye movements, and/or biting of the tongue. However, some seizures may not display any of these symptoms. An EEG may be useful in the evaluation of seizures in a patient presenting with an BRUE. The causes of seizures are vast and include the following: intracranial bleed, CNS infection, metabolic abnormality, electrolyte abnormality, fever, genetic syndromes and/or structural abnormalities.

A mother brings her 20-day-old male infant to your clinic for the child's first visit. You learn that the infant was born at home to a 28-year-old G1P1, and the infant has not yet received newborn screening. During your history, you learn that the infant has been vomiting 2 to 3 times per day, and the mother reports that her son seems fussier than her friends' infants. On exam, you note an eczematous rash and a musty odor to the infant's skin and urine. Which enzyme deficiency would you expect the infant to display?

A. Alpha-L-iduronidase B. Cystathionine synthase C. Glucose-6-phosphatase D. Phenylalanine hydroxylase E. Sphingomyelinase This infant likely has phenylketonuria (PKU), an autosomal recessive disorder of amino acid metabolism caused by a deficiency in the enzyme phenylalanine hydroxylase. Affected infants are normally detected by newborn screening, but can present with vomiting, hypotonia, musty odor, developmental delay, and decreased pigmentation of the hair and eyes. The best developmental outcomes occur if a phenylalanine-restricted diet is initiated in infancy. A. This answer choice is incorrect. A defect in alpha-L-iduronidase occurs in Hurler syndrome, a type of autosomal recessive lysosomal storage disease. Children typically do not display symptoms until one year of age. Symptoms include hepatosplenomegaly, coarse facial features, frontal bossing, corneal clouding, and developmental delay. Affected individuals typically do not live past fifteen years old. B. This answer choice is incorrect. A defect in cystathionine synthase occurs in homocystinuria, a disorder of amino acid metabolism. Homocystinuria is inherited in an autosomal recessive pattern. Individuals display Marfanoid body habitus, a hypercoagulable state, and possible developmental delay. The condition can be diagnosed by testing for increased methionine in a patient's urine or blood. C. This answer choice is incorrect. A defect in glucose-6-phosphatase occurs in Von Gierke's disease, a glycogen storage disease. Von Gierke's disease is inherited in an autosomal recessive pattern. Individuals present with hypoglycemia, hepatomegaly, and metabolic acidosis. E. This answer choice is incorrect. A defect in sphingomyelinase occurs in Niemann-Pick disease, a lysosomal storage disease. Children present by six months of age with hepatomegaly, ataxia, seizures, and progressive neurologic degeneration. Fundoscopic exam reveals a "cherry-red" macula.

A 14-month-old female presents to the clinic with a fever of 39.2 C and irritability. According to mom, the patient was initially sick one week ago with a runny nose and cough, but these symptoms had resolved. She started pulling at her ear and becoming increasingly irritable last night. She has had several prior ear infections and was most recently treated one month ago with amoxicillin. She is up to date on immunizations. Physical examination reveals a red, opaque, bulging tympanic membrane with bubbles and limited mobility of her left ear. The exam of the right ear is normal. Which of the following is the next step in the management of this patient?

A. Amoxicillin/clavulanate B. Decongestants C. High-dose amoxicillin D. Observation E. Tympanocentesis Amoxicillin/clavulanate is the treatment of choice for patients with moderate to severe otalgia or high fever, and is used for additional beta-lactamase coverage for Haemophilus influenzae and Moraxella catarrhalis, and when failure with amoxicillin is suspected. B. This answer choice is incorrect. The FDA has discouraged the use of over-the-counter cough and cold products in children younger than 2 years due to the increased risk/benefit ratio. C. This answer choice is incorrect. High-dose amoxicillin is the most common first-line treatment for acute otitis media due to its general effectiveness against susceptible and partially resistant S. pneumo, in addition to being low cost and having a high safety profile. However, this antibiotic was recently administered, raising concerns for a resistant organism. D. This choice is incorrect. Fifty to eighty percent of acute otitis media cases will resolve spontaneously without antibiotics; however, the decision to defer treatment with the "observation option" is based on the child's age and illness severity. This option is limited to healthy children between the ages of 6 months to 2 years with non-severe symptoms. Our patient is presenting with severe symptoms and a high-grade fever. E. This choice is incorrect. The patient has not had recurrent episodes of otitis media and has not started antibiotic therapy. Tympanocentesis is recommended as a diagnostic measure to confirm a bacterial etiology after a patient has failed repeated courses of antibiotics or if an unusually resistant organism is suspected.

A 2-year-old female is brought to the ED by her mother because of increasingly frequent abdominal pain. She has been experiencing this pain on and off for the past year, along with increasing abdominal distention, vomiting, and diarrhea. You chart her height and weight, and find that she is below the 5th percentile for both. IgA tissue transglutaminase (TTG) antibody returns positive. What is the best treatment for this child?

A. Antibiotic treatment B. Corticosteroids C. Gluten-free diet D. Metronidazole E. Pain management A. Antibiotics would be the treatment for some types of bacterial gastroenteritis. The chronic nature of this patient's abdominal pain strongly points away from a diagnosis of bacterial gastroenteritis, which would present acutely. Decreasing growth velocity also suggests a process more chronic than you would expect from bacterial gastroenteritis. Bacterial gastroenteritis typically presents with bloody diarrhea, with or without abdominal pain, and is associated with fecal leukocytes on Wright stain. B. Corticosteroids may be used to treat a flare of inflammatory bowel disease (IBD). Inflammatory bowel disease can present with mild/subacute or severe/acute abdominal pain, along with diarrhea (often bloody). Decreasing growth velocity is also common in patients with IBD. The positive IgA tissue transglutaminase antibody, however, suggests that a diagnosis of celiac is more likely in this patient. C. A gluten-free diet is the best way to manage celiac disease. Celiac disease can present with chronic abdominal pain, vomiting, abdominal distention, and diarrhea. Growth failure can result from malabsorption or anorexia. Anemia may also result from occult GI bleeding, although frank blood in the stool is rare. The IgA tissue transglutaminase antibody titer is a very sensitive and specific test for this disease. D. Metronidazole is the treatment of choice for giardiasis. Infections with giardia may be acute or chronic, but do not usually lead to growth failure. This patient's clinical picture may fit with a giardia infection, except that the positive IgA TTG antibody titer suggests a diagnosis of celiac disease. E. Pain management alone is not sufficient for a patient with abdominal pain from Celiac disease. Pain management alone may be appropriate in patients with Henoch-Schonlein purpura (HSP), if they have primarily joint and abdominal pain. The abdominal pain experienced by patients with HSP is usually acute in onset and preceded by the characteristic palpable purpuric rash. Because of the acute onset of HSP, growth failure is not associated with this disease. In this patient with chronic abdominal pain, no rash, and growth failure, HSP is unlikely.

Case 25- An 18-year-old mother with her 3-month-old son arrives at urgent care clinic reporting "my baby stopped breathing!" She and her baby are rushed into a triage room, where her son is noted to be very lethargic with increased work of breathing. As vital signs are being obtained, the mother reports "my baby stopped breathing in the car coming here, and didn't start again until I reached over and jostled his car seat!" Mom then shared that "my boyfriend said he rolled off the couch last night. Could it be related?" Mom also stated that her son hasn't been as active as usual, and has been vomiting occasionally. Physical exam is notable for a respiratory rate of 70 breaths per minute with intercostal retractions and crackles in the right lower lung fields posteriorly. You admit this patient with the diagnosis of pneumonia for empiric antibiotics and support, pending additional evaluation. CXR subsequently demonstrates a RLL infiltrate and faint, vertical lines on several posterior ribs bilaterally. What is the best next step in management?

A. Anticipatory guidance about appropriate car seat usage B. Head ultrasound C. Obtain a PTH level D. Skeletal survey (more x-rays) E. Sweat chloride testing A.This answer choice is incorrect. It is true that this mother needs to be better informed about car seat use, because the infant car seat should never be in the front seat of a car. This needs to occur prior to discharge, but does not have the same urgency as the correct answer. B. This answer choice is incorrect. If there are concerns for a head bleed, the best test for such a bleed and/or skull fracture would be a CT scan, not ultrasound. C. This answer choice is incorrect. An elevated PTH level suggestive of primary hyperparathyroidism can cause increased risk of fracture in infants, but this is an extremely rare disorder. In this patient, the pattern of posterior rib fractures, which can be caused by squeezing the infant's thorax, should lead the clinician to suspect non-accidental trauma. D. This answer choice is correct. In this case a skeletal survey is essential. Posterior rib fractures are always concerning findings that should raise suspicion for non-accidental trauma. While treatment of the patient's pneumonia has been initiated, a complete skeletal survey will screen for other worrisome findings, including multiple fractures in different stages of healing, fracture of the femur or tibia in a non-walking child, and skull fractures. If posterior rib fractures are identified, further evaluation for other injuries and mandated reporting are also indicated. E. This answer choice is incorrect. Sweat chloride testing for the diagnosis of cystic fibrosis should be done in an infant with pneumonia and failure to thrive. This patient's only episode of pneumonia with no evidence of FTT does not warrant a workup for CF at this time.

A 7-year-old male frequently gets in trouble at school for being disruptive and talking inappropriately in class. He does not follow directions and does not work well with classmates during group exercises. His mother relates that at home, he is always on the go, sleeping only six to seven hours each night, and he does not follow the rules at home. He often skips his homework and sometimes puts himself in danger, such as by running away unaccompanied. Which of the following is the most likely diagnosis?

A. Antisocial personality disorder B. Attention deficit hyperactivity disorder C. Bipolar mood disorder D. Functional neurologic symptom E. Rett syndrome ADHD is characterized by the triad of impulsivity, hyperactivity, and inattention- all of which are present in the child described above. Other symptoms include motor impairment and emotional lability. ADHD is typically diagnosed before the age of 7 but persists into adulthood. Intelligence is usually normal, but individuals with ADHD commonly perform more poorly academically than would be expected for their IQ. A. This answer choice is incorrect. Symptoms of antisocial personality disorder include inability to conform to societal norms, disregard of the rights of others, and often criminality. These individuals often exhibit impulsiveness due to their lack of consideration of the consequences of their actions. Males are affected more than females. However, a diagnosis of antisocial personality disorder can only be made in individuals older than 15 years. If symptoms appear earlier than that age, a diagnosis of conduct disorder is appropriate. The patient described does not display these characteristics. C. This answer choice is incorrect. Depression may be responsible for inattention in school for some children, and it is not uncommon for childhood depression to lead to bipolar disorder, in which hyperactivity and impulsivity comprise the manic phase of the disorder leading to a decreased need for sleep. However, the symptoms of depression and mania present in separate phases, not concurrently, as is described in this child, which makes bipolar disorder less likely in this patient. The mnemonic commonly used for mania is DIGFAST (distractibility, irresponsibility, grandiosity, flight of ideas, agitation, sleep decrease, talkativeness). The mnemonic for depression is SIGECAPS (sleep disturbance, loss of interest, guilt, energy loss, concentration impairment, appetite changes, psychomotor retardation, suicidal ideation). D. This answer choice is incorrect. Functional neurologic disorder (formerly called Conversion disorder) would be suspected if no cause could be identified for reported physical symptoms. The drive is unconscious on the part of the patient, and symptoms are not intentionally produced. Functional neurologic disorder is more common in adolescents and involves a sudden loss of sensory or motor functioning. When the patient consciously creates physical symptoms this is referred to as factitious disorder. E. This answer choice is incorrect. Rett syndrome is an X-linked pervasive developmental disorder seen only in females; affected males die in utero or at birth. The characteristic symptoms involve regression of language and development, intellectual disability, ataxia, and hand-wringing. This disorder is typically diagnosed earlier, at about age 1-4. This patient is male and does not have symptoms of regression.

After delivery, an infant experiences fetal distress. After a vaginal delivery with vacuum assist, the infant cries spontaneously but remains acrocyanotic, despite supplemental oxygen delivered by mask. The neonate is hypotonic and moves his extremities only in response to noxious stimuli. Physical exam reveals an open mouth with a protruding tongue, upslanting palpebral fissures, low-set ears, and a single palmar crease on both hands. What is the most common cardiac defect in the patient's suspected syndrome?

A. Aortic insufficiency B. Coarctation of the aorta C. Conduction pathway defects D. Endocardial cushion defects E. Patent ductus arteriosus This patient has physical signs of Down syndrome. Approximately 50% of children with Down syndrome are born with endocardial cushion defects, such as ventricular septal defect, atrial septal defect, or complete atrioventricular canal defect. A. This answer choice is incorrect. Aortic insufficiency is associated with Marfan syndrome. Aortic dissections are also seen in this condition and can be lethal. B. This answer choice is incorrect. Coarctation of the aorta is seen in 35% of patients with Turner syndrome. C. This answer choice is incorrect. Conduction pathway defects are not characteristic of patients with Down syndrome but may be seen in infants born to mothers with lupus (heart block). E. This answer choice is incorrect. Patent ductus arteriosus is usually seen in infants with prematurity, sepsis, metabolic acidosis, or pulmonary defects not associated with congenital syndromes.

A 6-week-old infant presents to your office for a check-up. The baby was born full-term by NSVD to a 29-year-old G1P0 mother with no complications. Mother states the baby was feeding well until a week ago, when he developed increased sleepiness, prolonged feeding, and greater duration between feeds. His mother notes he stops to take breaks during feeds because he seems to be trying to catch his breath. He has four to six wet diapers per day and stools three or four times per day. Vital signs: Temperature is 37.6 C (99.7F), respiratory rate is 68 breaths/minute, pulse is 138 beats/minute, blood pressure is 88/58 mmHg, and oxygen saturation is 98%. The physical examination is notable for increased respiratory effort and retractions, and, upon cardiac examination, a murmur with a hyperactive precordium and no cyanosis. Abdominal exam reveals a liver edge palpable to 4 cm below the right costal margin. Which condition would be LEAST LIKELY to be the cause of the infant's symptoms?

A. Aortic stenosis B. Atrial septal defect C. Coarctation of the aorta D. Patent ductus arteriosus E. Ventricular septal defect A. Choice A is incorrect because aortic stenosis is one of the heart defects that can present with a murmur and signs of congestive heart failure in infancy. An estimated 10% to 15% of patients with aortic valve stenosis present when they are younger than one year of age. Neonates with critical stenosis are typically symptomatic and present with symptoms of congestive heart failure — including poor feeding, rapid breathing, poor urine output, and fussiness — as the ductus arteriosus closes and systemic blood flow decreases. B. Choice B is correct because atrial septal defects (ASDs) do not cause CHF. ASDs often go undiagnosed for decades due to subtle physical examination findings and/or a lack of appreciable symptoms. Children with ASD's are generally asymptomatic. C. Choice C is incorrect because coarctation of the aorta is one of the heart defects that can present with a murmur and signs of congestive heart failure in infancy. Pediatric patients may present in the first few weeks of life with poor feeding, tachypnea, and lethargy and progress to overt CHF and shock if the narrowing is significant. Symptoms may be subtle at first, and patients may make repeated trips to the physician before being diagnosed. Diagnosis after the neonatal period is usually made during the evaluation of hypertension or a murmur. Older patients usually have not developed overt CHF because of the presence of arterial collateral vessels. D. Choice D is incorrect because a large patent ductus arteriosus (PDA) is one of the heart defects that can present with a murmur and signs of congestive heart failure in infancy. A PDA is more common in premature infants and those with neonatal respiratory distress syndrome. A smaller PDA may not cause any symptoms, but infants can present with tachypnea, poor feeding, tachycardia, shortness of breath, fatigue, diaphoresis, and poor growth. E. Choice E is incorrect because ventricular septal defect is one of the heart defects that can present with a murmur and signs of congestive heart failure in infancy. Signs and symptoms often appear during the first few days, weeks, or months of a child's life. VSD is an acyanotic congenital heart defect, manifesting as a left-to-right shunt. A holosystolic murmur is often appreciated, with larger VSDs causing a parasternal heave. An infant with a large VSD will fail to thrive and become diaphoretic and tachypneic, especially with feeding.

A 15-year-old female presents with three hours of abdominal pain and two episodes of nonbilious, nonbloody vomiting. She rates her pain at 8/10 and describes it as constant, diffuse, but most severe in her periumbilical region. It is worse with coughing and moving. She has never had pain like this before and has had no appetite since the pain started. She is sexually active with her boyfriend of three months, always uses condoms, and has not been tested for STIs. She is due to start her menstrual cycle next week. Vitals: 37.9 C, HR 100 bpm, BP 120/85 mmHg, RR 14 bpm. On exam, she exhibits involuntary guarding, mild rebound tenderness, and tenderness to palpation between her right anterior superior iliac spine and umbilicus. On pelvic exam, she reports tenderness when attempting to palpate her right adnexa, but there are no masses and no cervical motion tenderness. Her WBC and CRP are both mildly elevated. Based on the information above, what is the most likely diagnosis?

A. Appendicitis B. Cholecystitis C. Ectopic pregnancy D. Ovarian torsion E. Pelvic inflammatory disease A. This answer choice is correct. Appendicitis is the most common condition in children requiring immediate surgical intervention, but often presents differently than in adults (especially in infants). Aspects of their atypical presentation can include lack of migration of pain to the RLQ, negative Rovsing's sign, and involuntary guarding and fever without perforation. In school-age children who can articulate the pain, they often describe pain with movement or coughing. Also, rebound tenderness was found to be neither sensitive nor specific in the pediatric population, while in the adult population it is one of the most accurate PE findings (86%). Adolescents commonly present with a more typical clinical picture for appendicitis. Her sudden onset of intense pain at the umbilicus with vomiting, anorexia, and tenderness at McBurney's point are all classic findings. The more atypical signs include diffuse pain centered around the umbilicus and rebound tenderness that might point to a perforation. Another atypical aspect of her exam is her adnexal pain during the pelvic exam, which could be due to the degree of inflammation and the positioning of her appendix. The key take-away point is to have a high index of suspicion for appendicitis for pediatric patients with abdominal pain given their atypical presentation. B. This answer choice is incorrect. The pain of cholecystitis is steady, most often present in the RUQ, and can radiate to the right shoulder. It can worsen with ingestion of fat-rich foods, and often causes anorexia with episodes of nausea and vomiting. It is much less common in children than adults. Given this patient's presentation, age, and exam, cholecystitis is very low on our differential. C. This answer choice is incorrect. Patients with ectopic pregnancy typically present with vaginal bleeding six to eight weeks after their last menstrual period. Pain is described as crampy pelvic pain, and it is often associated with nausea. Diffuse abdominal pain is also present if rupture and intraperitoneal bleeding occurs. Given that this patient is three weeks from her last menstrual period, reports no vaginal bleeding, and her pain is intense and located in the middle of her abdomen, an ectopic pregnancy is less likely. D. This answer choice is incorrect. The primary risk factor for ovarian torsion is an ovarian mass (e.g. a cyst or neoplasm). While ovarian torsion can present at any age, frequency of cyst development, and hence torsion, is increased in women of reproductive age, during pregnancy, and in women undergoing ovulation induction for treatment of infertility. The pain of ovarian torsion is described as intermittent stabbing pain in the lower abdomen or pelvis. Nausea and vomiting are very common findings as well. Ultrasound is essential to initial workup. Given that the patient has periumbilical pain, tenderness at McBurney's point, and no palpable masses on pelvic exam, ovarian torsion is a less likely diagnosis. E. This answer choice is incorrect. Pelvic inflammatory disease is a possibility given the patient's sexual history, lack of STI screening, and adnexal pain. However, this pain is often post-coital and also first occurs during or immediately following menstruation. Other key findings are mucopurulent discharge and cervical motion tenderness, both of which are absent in this patient. RUQ pain and a fever (present in 50% of patients) are other signs not seen in this case.

A 9-year-old male presents to the emergency department in an ambulance after he was found unconscious at a local playground. In the ED he is lethargic and difficult to arouse. He is able to minimally verbalize that his head and stomach hurt. He vomits clear liquid twice over the course of 30 minutes. Vital signs are as follows: T 37.6 C, HR 66 bpm, BP 155/80 mm Hg, RR 18 bpm. His respirations are irregular with brief episodes of apnea. On physical exam you are unable to reproduce the abdominal pain and there is no rebound tenderness or guarding. The rest of the physical exam is unremarkable. Which of the following is the most likely diagnosis?

A. Appendicitis B. Diabetic ketoacidosis C. Intracranial hemorrhage D. Small bowel obstruction E. Viral gastroenteritis A. Choice A is incorrect because the lack of fever, inability to reproduce the abdominal pain in the RLQ, and the severely altered mental status argue against the diagnosis. In appendicitis, children will often complain of a migratory pain that begins around the periumbilical region and migrates to the RLQ. Patients often complain of rebound tenderness and demonstrate guarding as well. B. Choice B is incorrect because in a patient with DKA one would expect increased adrenergic tone leading to tachycardia, not inappropriate slowing of the HR. Secondly, the patient's breathing pattern is more consistent with Cheyne-Stokes respirations, not Kussmaul breathing. Kussmaul breathing is typically characterized by deep breaths that may be rapid, normal, or slow in rate without periods of apnea, often associated with metabolic acidosis. Lastly, one would expect signs or symptoms consistent with dehydration such as polyuria, polydipsia, decreased skin turgor, or skin tenting. Although altered mental status, vomiting, headache, and abdominal pain could be seen in DKA, this patient's presentation is most consistent from increased ICP from trauma (due to lack of dehydration, tachycardia). C. This is the correct choice. Increased ICP can be secondary to epidural or subdural hemorrhage. It is possible the patient may have fallen while playing in the playground. Increased ICP can present as the classic Cushing's triad: hypertension, inappropriate slowing of the heart rate, and irregular respirations (Cheyne-Stokes respiration). Lastly, the patient's headache and non-bilious vomiting can also be attributed to the increased ICP. D. Choice D is incorrect because a small bowel obstruction usually presents with bilious vomiting, abdominal distention, inability to pass flatulence, and moderate-to-severe abdominal pain. The pain is often paroxysmal, coming and going in four to five minute intervals. The patient's lack of a fever, however, could be consistent with a GI obstruction. The most common causes of a small bowel obstruction are adhesions from a previous surgery or a hernia. E. Choice E is incorrect because gastroenteritis usually presents with fever, colicky abdominal pain, and diarrhea. It would also be atypical for the patient's mental status to be so adversely affected by gastroenteritis. More than 95% of gastroenteritis hospitalizations occur in children younger than five years, with the peak incidence between 3 and 24 months of age. The incidence tends to peak in winter. There can be both viral and bacterial causes for gastroenteritis.

A 7-year-old male is brought by ambulance to the ED because of decreased mental status for two hours. His parents say that he has been tired for the past three days. Six hours ago, he began vomiting, but has not had diarrhea. He is difficult to arouse and moans with stimulus. Temperature is 98.6, pulse is 124 beats/minute, respirations are 28 breaths/minute, blood pressure is 96/68 and oxygen saturations are 99% on room air. Physical exam findings include dry mucous membranes and moaning on palpation of the abdomen. The remainder of the exam is normal. What is the most likely cause of his condition?

A. Appendicitis B. Diabetic ketoacidosis C. Non-accidental trauma D. Opioid overdose E. Sepsis A. Appendicitis would rarely present with altered consciousness. The usual history with appendicitis is onset of periumbilical pain that persists over hours, migrating to the right lower quadrant. Vomiting could be present, and tachycardia may be present due to pain or dehydration, but altered mental status would be unusual. On physical exam, peritoneal signs may be present as well as psoas, obturator, or Rovsing's signs. B. DKA typically presents with altered mentation, vomiting, dehydration, and abdominal pain. The history will yield polydipsia and polyuria during the days preceding DKA. Metabolic acidosis causes tachypnea as the body tries to blow off CO2 through a compensatory respiratory alkalosis. C. Lack of fractures, bruises, or burns argues against this choice. Trauma resulting in increased intracranial pressure may result in hypertension, bradycardia, and disordered breathing. D. Although he may have ingested opioids, his presentation does not fit well with this choice. Signs of opioid overdose include pinpoint pupils, depressed respiratory rate, and altered consciousness. His vital signs, tachypnea, and lack of pinpoint pupils argue against this choice. E. Sepsis can present with altered mental status. This child's presentation is less consistent with sepsis given that he doesn't have fever or other vital sign changes consistent with sepsis syndrome (temperature > 38.5°C or < 36°C, hypotension, along with warm, dry extremities).

A 16-year-old female presents with acute onset of diffuse abdominal pain with intermittent sharpness in the epigastrium that radiates to her back. She has had some episodes of vomiting and has a low grade fever. She is sexually active and has used alcohol in the past. Which of the following is most likely to present with epigastric abdominal pain?

A. Appendicitis B. Ectopic pregnancy C. Ovarian torsion D. Pancreatitis E. Urinary tract infection A. This answer choice is incorrect. The pain in appendicitis classically starts as periumbilical pain that migrates to McBurney's point (1-2" from the anterior superior iliac spine toward the umbilicus). However, it is important to always consider appendicitis in a teenager presenting with abdominal pain with a presentation like this, as acute appendicitis is the most common pediatric condition requiring emergency surgery. Of children presenting with acute abdominal pain, 1-4% have appendicitis. In this case, the location of the pain best supports a diagnosis of pancreatitis. B. This answer choice is incorrect. Although ectopic pregnancy is always a concern in a sexually active female presenting with abdominal pain, the pain is typically located in the lower abdomen. Fever and diffuse abdominal pain are not likely in uncomplicated ectopic pregnancy. C. This answer choice is incorrect. Although ovarian torsion is possible at any age, the abdominal pain is typically described as stabbing and is usually localized to the lower abdomen and pelvis. Nausea and vomiting can also be present. D. This answer choice is correct. Pancreatitis commonly causes continuous abdominal pain that can localize to the epigastrium ("band-like pain") as well as radiate to the back. Nausea and vomiting are nearly always present. Lipase is the most sensitive and specific laboratory test to diagnose pancreatitis. Although alcohol use can be associated with pancreatitis, the most common cause of pancreatitis in this age group is viral and idiopathic. E. This answer choice is incorrect. Urinary tract infections classically cause painful urination with frequency and urgency. Patients may present with suprapubic tenderness, and flank pain is common when the infection has spread to the kidney. UTIs are common in sexually active females, and fever and vomiting are often seen with pyelonephritis, but sharp right upper quadrant pain would not be expected.

A previously healthy 4-year-old girl is brought to her clinician because her parents have noticed that she has been less active than usual for the past three weeks. Her father explains that it is difficult to get his daughter out of bed in the mornings and that she no longer plays outside with her older brother. Physical examination is notable for a temperature of 38.4 C, heart rate of 125 bpm, pallor, truncal bruising, and diffuse lymphadenopathy. The remainder of the exam, including a thorough neurologic assessment, is unremarkable. Which of the following is the most likely diagnosis?

A. Aseptic meningitis B. Kawasaki disease C. Non-accidental trauma D. Acute lymphoblastic leukemia E. Infections mononucleosis A. Answer choice A is incorrect. Aseptic meningitis does not explain this child's pallor, bruising, or lymphadenopathy. While aseptic meningitis can present with lethargy and low-grade fevers, the remainder of her symptoms are not consistent with meningitis. B. Answer choice B is incorrect. This child does not meet criteria for the diagnosis of Kawasaki disease (KD).She has not had fevers for at least five days and has no other symptoms consistent with KD such as bilateral nonsuppurative conjunctivitis, changes in oral mucosa, extremity erythema or swelling, and polymorphous rash. In addition, this patient has diffuse lymphadenopathy as opposed to the unilateral cervical LAN > 1.5cm that is seen in KD. C. Answer choice C is incorrect. Non-accidental trauma does not account for the systemic inflammatory signs found on this patient's physical exam: fever and lymphadenopathy. Unexplained bruising, however, should always raise suspicion of child abuse. D. Answer choice D is correct. Acute lymphoblastic leukemia (ALL) is consistent with this child's evidence of anemia (fatigue, tachycardia, pallor) and thrombocytopenia (unexplained bruising). Failure of two or more hematologic cell lines should always raise suspicion for malignant invasion of the marrow. Furthermore, the child's chief complaint, fatigue, is the most common presenting symptom of acute leukemia. In addition, the incidence of ALL peaks at age 4 years. E. Answer choice E is incorrect. While this child's subacute presentation with marked fatigue, low-grade fevers, and generalized lymphadenopathy is consistent with infectious of mononucleosis, in this patient, apparent bone marrow failure evidenced by suspected anemia (fatigue, tachycardia, pallor) and thrombocytopenia (bruising) would be more suggestive of acute leukemia.

A previously healthy 11-month-old male with 5-day history of a "cold," is brought to the ED by his mother for one day of acute worsening cough and intermittent wheezing. Per the mother, the cough was initially dry but has become more "phlegmy," making it difficult for the infant to breathe, particularly when he is feeding or more active. His immunizations are up to date, and he has no known allergies. His family history is significant for a 6-year old sister who was diagnosed with asthma four years ago. On exam, the infant is afebrile, mildly tachypneic with normal O2 saturation. He has prominent nasal flaring and mild subcostal retractions. He has clear rhinorrhea but no evidence of oropharyngeal erythema. Lung exam reveals decreased breath sounds and wheezes on the right. What is the most likely diagnosis?

A. Asthma B. Epiglottitis C. Foreign body aspiration D. RSV bronchiolitis E. Viral URI Given the patient's age, foreign body aspiration should always be included in the differential diagnosis for acute onset wheezing. The lung findings of asymmetric breath sounds and wheezing support this diagnosis. Foreign body in the airway can be confirmed by bilateral decubitus or inspiratory/expiratory chest films, characterized by hyperinflation and decreased deflation on the affected side. If there is complete obstruction of the airway, x-ray will generally reveal atelectasis (whiting out) and signs of volume loss (mediastinal shift towards affected side to compensate for loss of volume). A. This answer choice is incorrect. Asthma is caused by inflammation of airway mucosa, mucus hypersecretion, mucosal edema and reversible bronchoconstriction. It generally presents as cough, wheezing, tachypnea and dyspnea worsened by cold air, exercise, allergies and URIs. The mainstay of treatment involves bronchodilators (beta-2 agonists) and inhaled steroids. Asthma is a possible diagnosis in this patient given the family history of asthma; however, it is less likely since he was previously healthy with no history of recurrent cough or wheezing. Furthermore, asthma does not generally present with focal wheezing as heard on his lung exam. B. This answer choice is incorrect. Epiglottitis was commonly caused by Haemophilus influenzae type B, but can also be caused by Staph and Strep species. Patients may present with fever, dysphagia, drooling, stridor and significant respiratory distress. Patients are generally seen sitting, leaning forward with the neck hyperextended. Epiglottitis has become less common due to immunization with Hib. This diagnosis is less likely in this patient, since his immunizations are up to date, he is afebrile and not in severe respiratory distress. D. This answer choice is incorrect. Bronchiolitis is a lower respiratory tract infection most commonly caused by RSV, which is characterized by bronchiolar obstruction secondary to mucus plugging, cellular debris, and edema. Patients generally present with fever and URI symptoms which progress to a worsening cough, wheezing and shortness of breath. Although this patient does have wheezing, the unilateral wheezing with decreased breath sounds is not consistent with bronchiolitis. E. This answer choice is incorrect. This patient probably developed a viral URI five days ago. An upper respiratory tract infection in children can manifest as fever, rhinorrhea, cough, sore throat and myalgias, and may be accompanied by wheezing. However, this patient's auscultation findings cannot be explained solely by a viral URI.

A 10-month-old infant is brought to the pediatrics emergency department by her parents, who say she has been coughing persistently for the past three hours. The parents were watching a movie at home when they first noticed their daughter coughing. Patient is a vaccinated, well-nourished infant in moderate distress with retractions, nasal flaring, and grunting. Physical examination reveals diminished breath sounds in the right lung with normal breath sounds on the left. What other associated physical exam findings are expected on auscultation?

A. Asymmetric wheeze B. Bronchial breath sounds C. Crackles D. Rhonchi E. Stridor This infant is in respiratory distress from foreign body aspiration, consistent with the history of acute onset of distress and asymmetric breath sounds. Common foreign bodies include peanuts, popcorn, grapes, hard candy and hot dogs. Respiratory distress from foreign body aspiration is usually accompanied by asymmetric breath sounds and wheezes on auscultation of the affected side. B. This answer choice is incorrect. Bronchial breath sounds are hollow-sounding and caused by air moving through areas of consolidated lung, such as in the setting of pneumonia. C. This answer choice is incorrect. Crackles are due to fluid in alveoli or opening and closing of stiff alveoli, not consistent with a foreign body aspiration. You would expect this in either pneumonia or congestive heart failure from pulmonary edema, both of which are unlikely in this patient given the abrupt onset and lack of history of cardiac problems. D. This answer choice is incorrect. Rhonchi are coarse, low-pitched, rattling sounds due to secretions and airway narrowing and are typically heard in the setting of bronchiolitis or pneumonia. E. This answer choice is incorrect. Stridor is due to airway narrowing above the thoracic inlet and can be seen in respiratory infections such as epiglottitis and laryngotracheobronchitis (croup). Stridor is also an expected finding in laryngomalacia, but this would not acutely appear at 10 months of age. Also, epiglottitis is more rare now in a vaccinated child, and it is not expected to hear unilaterally diminished breath sounds.

Case 28-- A 20-month-old girl is brought to the primary care clinic for follow-up of cerebral palsy. She was born via home birth at 38 weeks gestation to a primigravida mother. She developed severe jaundice requiring admission at 5 days of life and required phototherapy and exchange transfusion. She has subsequently developed slow and uncontrolled movements throughout her body. Her brain MRI shows atrophy of the basal ganglia. Which type of cerebral palsy is most consistent with this patient's presentation?

A. Ataxic cerebral palsy B. Dyskinetic cerebral palsy C. Spastic diplegia D. Spastic hemiplegia E. Spastic quadriplegia This choice is correct because dyskinetic CP is associated with kernicterus, due to hyperbilirubinemia, with findings of basal ganglia pathology on imaging. Patients typically have motor abnormalities throughout the body. Dyskinetic CP is also associated with perinatal asphyxia and can involve the thalamus and cerebellum on imaging. A. This choice is incorrect because ataxic cerebral palsy should show cerebellar abnormalities on imaging, while Amy's MRI shows only basal ganglia involvement. C. This choice is incorrect because spastic diplegia is classically associated with premature birth and specific MRI findings of periventricular white matter abnormalities. Patients present with motor involvement that is more prominent in the legs than the arms. D. This choice is incorrect because spastic hemiplegia — associated with a stroke damaging a unilateral upper motor neuron tract — should present with spasticity of the contralateral arm and leg, not the entire body. E. This choice is incorrect because patients with spastic quadriplegia have spasticity, clonus, and exaggerated tendon jerks throughout their bodies. Imaging would show global brain abnormalities.

A young couple presents to the ED with their 2-month-old son reporting excessive sleepiness and difficulty arousing him after his nap. Per the parents, the PMH and prenatal course are unremarkable, except that the patient has always been very fussy and would often cry despite being held and cradled. He is cared for during the day by his babysitter. Today he had been in his usual state of fussiness when the babysitter arrived, and they returned to find him napping quietly in his cradle but could not arouse him from sleep when it came time for his feeds. He finally opened his eyes after several minutes of gentle nudging but seemed to quickly fall asleep again. On exam, the infant is afebrile with poor tone and is only mildly responsive to painful stimuli. Eye exam shows dilated pupils and an ophthalmology consult reveals extensive retinal hemorrhages in multiple layers across all four quadrants. What is the most likely diagnosis?

A. Bacterial meningitis B. Closed head injury C. Infant botulism D. Intoxication E. Metabolic disorder A. This answer choice is incorrect. Bacterial meningitis is more common during the first month of life than at any other point and is usually secondary to GBS infection. While meningitis could account for some of the symptoms seen in our patient, such as lethargy and abnormal tone, fever is the most common symptom and is notably lacking in our patient. The extensive retinal hemorrhages noted are also inconsistent with this diagnosis and point more towards a closed head injury secondary to non-accidental trauma. Retinal hemorrhages may occur with severe bacterial meningitis, but are in a different pattern than seen in abusive head injury. Retinal hemorrhages associated with meningitis are usually intraretinal and limited to the posterior pole, whereas retinal hemorrhages due to non-accidental trauma are usually located in multiple quadrants, multiple layers, and may have associated retinoschisis. B. This answer choice is correct. Closed head injuries and extensive retinal hemorrhages in infants and children are secondary to violent shaking or throwing, resulting in tearing of the bridging vessels. The retinal hemorrhages, as seen in our patient, are most consistent with non- accidental head trauma. Other signs and symptoms include stiffness, constant crying, seizures, difficulty to arouse, decreased appetite and excessive sleeping. Infants often have no other signs of physical abuse (e.g., bruises). C. This answer choice is incorrect. Infant botulism results from ingestion of C. botulinum spores that germinate in the infant's gastrointestinal tract leading to constipation, symmetric weakness, feeding difficulties, drooling, irritability and weak cry. It is commonly believed to be associated with ingestion of honey but can also be caused by ingestion of environmental dust containing C. botulinum spores. This diagnosis, while on the differential, is unlikely given the lack of symmetric weakness noted in the history and presence of other physical exam findings more consistent with non-accidental trauma. D. This answer choice is incorrect. Intoxication caused by accidental ingestion of medications or toxins could result in symptoms similar to those seen in our patient such as lethargy and poor tone but is unlikely given the absence of any medication exposure noted in the history. Accidental ingestion is also more likely in children who are crawling or walking as opposed to this infant. Inappropriate medication administration is a potential cause of altered mental status in an infant, such as opiates. Infants will present with pinpoint pupils and prolonged apnea. Our patient had dilated pupils and extensive retinal hemorrhages, which is inconsistent with opiates. E. This answer choice is incorrect. Most inborn errors of metabolism are rare and caused by single enzyme deficiencies leading to disruption of the metabolic pathway. While errors of metabolism could account for some of the symptoms seen in our patient, such as lethargy and abnormal tone, a negative newborn screen makes this diagnosis less likely. The retinal hemorrhages noted are also inconsistent with this diagnosis and point more towards non- accidental trauma.

Case 12-- A 12-year-old boy presents to the ED with reports of anorexia, weight loss, persistent cough, with nocturnal coughing fits that have been waking him from sleep for the past three weeks. He reports no fever, chills, myalgia, sore throat, or rhinorrhea. The patient presented to his primary care clinician one week prior with the same report, and was treated with amoxicillin and bronchodilator therapy. His chest x-ray was negative for infiltrates at that visit. The patient's symptoms did not improve with this regimen. The cough became more frequent, sometimes causing emesis. Which of the following is the most likely diagnosis?

A. Atypical pneumonia due to Mycoplasma pneumoniae B. Foreign body aspiration C. Infection with Bordetella pertussis in the paroxysmal D. Laryngotracheobronchitis E. Reactive airway disease The paroxysmal stage of pertussis lasts four to six weeks and is characterized by repetitive, forceful coughing episodes, followed by massive inspiratory effort. This massive inspiratory effort is what results in the characteristic "whoop"-sounding cough. This is consistent with the patient's presentation and duration of illness. The forceful coughing fits in pertussis can even lead to conjunctival hemorrhages and pneumothoraces from the increased intrathoracic and intracranial pressures from Valsalva. The antimicrobial agents of choice for treatment of pertussis are azithromycin, clarithromycin, and erythromycin. Antibiotics given in the paroxysmal phase will reduce communicability but will not alter the clinical course. A. This answer choice is incorrect. Mycoplasma pneumonia would be associated with fevers and findings on chest x-ray and lung exam. B. This answer choice is incorrect. Foreign body aspiration presents more acutely, with a preceding history of choking, especially in an older child. Also, CXR findings may be negative on the initial PA film, but should be abnormal if inspiratory and expiratory films are obtained. D. This answer choice is incorrect. Croup (laryngotracheobronchitis) presents with difficulty breathing and a "seal bark" cough and usually lasts a week or less. Croup also is usually associated with fever. E. This answer choice is incorrect. Reactive airway disease would most likely have improved with bronchodilator therapy. In addition, you would not expect to see anorexia or weight loss with reactive airway disease.

An 18-month-old boy is brought to the primary care office for a well child visit. His family recently moved into town, and this is the first visit to the clinic. He was born at 32 weeks gestation and spent four weeks in the nursery due to breathing problems. Developmental screening shows normal social language and social development, but delayed gross motor development. He is able to stand, but not walk. Physical exam reveals increased tone, exaggerated deep tendon reflexes, and clonus in both of his lower extremities. Which of the following is the most likely diagnosis in this patient?

A. Autistic spectrum disorder B. Cerebral palsy C. Chromosomal disorder D. Metabolic disorder E. Myopathy A. Autistic spectrum disorders are identified by delayed language and social development. Gross motor delays are not a feature of autism. B. Spastic diplegia is a form of cerebral palsy, a nonprogressive static encephalopathy characterized by delays in motor development. It may be associated with periventricular white matter abnormalities that are thought to be due to ischemia. These changes can be visualized on MRI. In spastic diplegia, the motor abnormalities are often greater in the legs than in the arms. C. Chromosomal disorders usually present with dysmorphic features. Examples are trisomy 21 which has typical facial and body abnormalities. D. Metabolic disorders include many rare diagnoses. Some present in infancy and others are degenerative. Metabolic disorders tend to cause global delays isolated to the lower extremities. Although a child with a metabolic disorder could present with spastic diplegia, the higher prevalence of cerebral palsy makes this diagnosis more likely. E. Myopathies can be caused by metabolic, mitochondrial, and genetic causes. They present with hypotonia rather than hypertonia.

A 15-year-old female comes to the clinic with a chief complaint of feeling tired for one month. She has also been complaining of frequent nosebleeds while at school and bruising easily. She had menarche at the age of 9 and her periods have always been heavy and irregular. Her mother and grandmother also have heavy periods and easy bruising. What is the pattern inheritance of the bleeding disorder in this patient?

A. Autosomal dominant B. Autosomal recessive C. Mitochondrial D. X-linked recessive A. Autosomal dominant (AD) inheritance is the correct choice. In AD disorders males and females are equally affected within each generation. These include conditions such as von Willebrand disease, Marfan syndrome, neurofibromatosis, and Huntington's disease. B. Autosomal recessive (AR) is incorrect. In AR diseases female and male offspring of heterozygote carriers have a ¼ chance of being affected. These include Tay-Sachs disease and cystic fibrosis. C. Mitochondrial inheritance is incorrect. In mitochondrial disorders the disease is inherited only from the mother, and usually all children are affected. Affected males cannot pass the disorder on to their offspring. These include mitochondrial myopathies such as Leber's hereditary optic neuropathy and mitochondrial encephalopathy. D. X-linked recessive (XLR) is incorrect. In XLR disorders males are more commonly affected, while females are the carriers and pass the disorder on to their sons. Males cannot pass the disease on to other males. These include conditions such as Duchenne's muscular dystrophy, hemophilia, and Fragile X syndrome.

An 11-year-old male comes to the clinic with a chief complaint of abdominal pain for three months. The pain is not associated with eating. Sometimes he feels full and nauseated along with the pain, but then it resolves on its own. He denies diarrhea, vomiting, and bloody stools. He started a new school four months ago. His mother is primarily concerned because his abdominal pain causes him to miss school quite often now. He is otherwise healthy. When you evaluate his growth curves, he is progressing at the 60th percentile for height and weight and you do not notice a change since birth. He is an honors student. Vital signs are normal. Physical exam, including genital and rectal is normal. Stool is negative for occult blood. What is the most likely diagnosis in this child?

A. Bacterial gastroenteritis B. Functional abdominal pain C. Inflammatory bowel disease D. Meckel's diverticulum E. Peptic ulcer disease A. This is incorrect because bacterial gastroenteritis is more of an acute problem that would not persist for months. The absence of diarrhea also suggests bacterial gastroenteritis is not the best answer option. B. Functional abdominal pain would be the most likely diagnosis in this setting at this time. History in this setting is not suggestive of any other diagnosis directly causing his abdominal pain, except a change in his social setting. For better understanding of the nature of this child's pain, it would be best to talk to him alone, without his mother present, to determine if he is having trouble adjusting to school and to assess whether he has a stable home environment. His pain is chronic, with no other symptoms (diarrhea, bloody stools, growth failure), making a functional pain most likely. C. This answer is incorrect. Although inflammatory bowel disease (Crohn's disease or UC) could be the cause of this child's pain, but IBD usually presents with growth failure, diarrhea, bloody stools, and sometimes fever. Although this could be an early presentation, the absence of arthritis, uveitis, and other constitutional symptoms, make IBD less likely. D. This answer is incorrect because Meckel's diverticulum usually presents as painless blood in the stool. E. This answer is incorrect because peptic ulcer disease (PUD) is relatively uncommon in children and the stool sample was negative for occult blood. PUD would be chronic and recurrent, but should also be associated with eating and would not spontaneously remit every couple of days.

A 3-year-old female is at the pediatrician's office for continued right knee pain after a ground-level fall six weeks ago. The patient is UTD on all immunizations, has no significant PMH, and no recent illnesses. The patient's mother reports the patient complains of pain mostly in the morning when going to daycare but doesn't seem to be bothered by it while playing outside in the afternoon. On exam, the patient's vitals are all within normal limits. Her physical exam reveals a well-appearing toddler who walks stiffly and avoids bending her right knee. The knee has a mild effusion but no obvious erythema. There is pain with passive flexion and extension of the right knee. During the exam she tells you her left ankle also hurts, which her mother had forgotten about but says started hurting the same time as the right knee. Her CBC is normal, while her ESR and CRP are mildly elevated. Which of the following is the most likely cause of this child's condition?

A. Bacterial osteomyelitis B. Juvenile idiopathic arthritis C. Leukemia D. Septic arthritis E. Transient synovitis of the hip A. Bacterial osteomyelitis in children, usually hematogenous in origin, presents with acute onset of bone pain and fever and may involve a contiguous joint. This patient is well appearing and has no leg tenderness. B. Juvenile idiopathic arthritis (JIA) is correct for several reasons. Oligoarticular juvenile arthritis is the most common type of JIA (60% of JIA) and causes pain in four or fewer joints for six or more weeks. This patient is generally well even after six weeks of pain, which would be unlikely if this patient had septic arthritis. Her pain improves with activity, and the ESR/CRP are only mildly elevated. On exam, she has a mild effusion but no obvious erythema. In cases of systemic JIA, patients may have a rash which lasts only a few hours (evanescent) that is also macular and salmon-colored, and high-spiking and appears periodically (once or twice a day); however, this form of JIA is not consistent with this patient's history. C. Leukemia is incorrect because the patient does not present with other systemic symptoms such as fever, weight loss, lymphadenopathy, hepatosplenomegaly, petechiae, or bruising. This patient also has pain that improves with activity, as opposed to a more chronic pain that does not improve with position or movement that would be expected with leukemia. Additionally, a normal CBC without thrombocytopenia or anemia makes this answer unlikely. D. Septic arthritis is incorrect because the patient has been generally well for the past six weeks and presents with mild swelling but no erythema of her joint. She is afebrile with no leukocytosis on CBC. While each of these alone would not rule out septic arthritis, the combination of all these factors together make septic arthritis unlikely. Patients with septic arthritis are often febrile. Joint fluid analysis often reveals elevated WBCs and predominant neutrophils (although a joint tap in this scenario is not necessary based on the presenting symptoms and exam). Bacteria may also be cultured from the joint fluid. Additionally, her CRP and ESR are only mildly elevated. One would expect to see a greater elevation of these markers of inflammation in septic arthritis. E. Transient synovitis of the hip is incorrect because this condition, while acute, generally resolves in three to four days and this patient has had pain for six weeks. Transient synovitis typically affects the hip, but patients may also report knee or inner thigh pain. Commonly, patients may have a history of a recent URI. Additionally, patients with transient synovitis do not typically present with a joint effusion as is seen in this patient.

A 12-day-old baby is brought to the ED by her mother due to fussiness and tactile fevers. The baby's mother did not receive prenatal care and delivered her baby at home without medical assistance. On further questioning, you find out that the patient has had only two wet diapers per day and two loose green stools per day. On exam, the patient is irritable and her anterior fontanelle is bulging. Which of the following possible diagnoses is of most emergent concern at this time?

A. Bacterial sepsis/meningitis B. Down syndrome C. Fetal alcohol syndrome D. Metabolic disorder E. Meconium ileus A. This choice is correct. The patient's mother did not have prenatal care and likely did not have screening for group B strep during pregnancy. She also delivered at home without medical assistance and would not have had access to antibiotics during delivery. Group B strep is a common and serious cause of sepsis and meningitis in newborns. B. This answer choice is incorrect. Although the patient is likely to have missed the newborn screening for Down syndrome, the most emergent concern is the patient's fever and possible sepsis. C. This choice is incorrect. Although the patient is at risk for fetal alcohol syndrome (since her mother did not have prenatal care and may not have had counseling regarding abstinence from alcohol), the most urgent concern is fever in a less than 1-month-old infant. D. This choice is incorrect. While a metabolic disorder may cause irritability in the newborn, it would not explain the tense fontanelle. A newborn screen should be sent now that she has presented for medical care. The newborn screen should detect many of the emergent metabolic deficiencies. E. This choice is incorrect because the patient reportedly passed stool since delivery.

A 6-month-old female presents to the Emergency Department because of decreased appetite and fussiness for four days and fever for one day. She had two episodes of non-bilious, non-bloody emesis today with decreased urine output. She was born at 40 weeks gestation. Pregnancy was uncomplicated and this is her first illness. Her vaccinations are up to date. She is non-toxic appearing but fussy. Temperature is 101 F. Her physical examination is non-focal. Which of the following is the most appropriate to evaluate her risk of urinary tract infection at this time?

A. Basic Metabolic Panel B. Midstream clean catch urine collection C. Renal bladder ultrasound D. Urinary catheterization E. Voiding cystourethrogram (VCUG) A. This answer choice is incorrect. Although a basic metabolic panel would be ordered to evaluate electrolytes in this ill infant, it will not help in the diagnosis of a urinary tract infection. A creatinine would not be elevated unless the infant has significant kidney involvement. B. This answer choice is incorrect. Midstream clean catch specimen is incorrect because the patient has not been toilet-trained and cannot void on demand. C. This answer choice is incorrect. Renal ultrasound may be indicated if the infant is found to have a urinary tract infection but is not indicated as part of the initial work up. D. This answer choice is correct. Urinary catheterization is the best method for obtaining a specimen for culture that has not been contaminated by perineal bacteria, and for this ill child, the first step is to determine the cause of the fever with accuracy, then begin parenteral antibiotics. E. This answer choice is incorrect. A voiding cystourethrogram (VCUG) is not the next appropriate next step in the evaluation of this patient.

A 1-day-old infant is being evaluated in the newborn nursery. The infant was born via NSVD at 40 weeks' gestation to a 38-year-old G1P1 mother who did not have access to prenatal care and did not receive prenatal testing. The infant weighed 7 lbs 12 oz at birth and had Apgar scores of 7 and 8. On exam the infant is sleeping comfortably. She is afebrile with normal vital signs but appears to have low tone on exam. You also notice her ears seem to be lower than her eyes and appreciate mild edema of the hands and feet. Additionally, you note a fluid-filled sac at the base of the neck that does not appear to interfere with breathing. A karyotype performed after birth reveals a chromosomal abnormality. Which of the following is the most likely cause of this infant's condition?

A. Benign neonatal hypotonia B. Cystic hygroma C. Down syndrome D. Fetal alcohol syndrome E. Turner syndrome Turner syndrome is defined by the karyotype 45 XO. Characteristics of females with Turner syndrome include renal abnormalities, lymphedema (causing edema of hands and feet), low-set ears, congenital heart defects, dental abnormalities-such as narrow or high-arched palates-and cystic hygromas. Other possible physical findings include a webbed neck, widely-spaced nipples, and shield-like chest. A. This answer choice is incorrect. Benign neonatal hypotonia is incorrect because the infant demonstrates facial features that suggest an alternate etiology as well as abnormal karyotype. Typically, hypotonia of the newborn, especially at one day after birth, is abnormal and could reflect illness (such as sepsis), or neurologic dysfunction at either the central or peripheral levels. B. This answer choice is incorrect. An isolated cystic hygroma can present with a fluid-filled collection around the neck but is not associated with edema elsewhere and would not be associated with a chromosomal abnormality. C. This answer choice is incorrect. Infants with Down syndrome typically have physical findings that include epicanthic folds, flat facial profile, single palmar creases, redundant neck skin, and heart defects. Other findings may include a gap between the first and second toes. Infants with Down syndrome can be hypotonic as is this infant. However, the patient in this scenario does not have the other physical findings consistent with Down syndrome. D. This answer choice is incorrect. FAS can cause hypotonia, but this infant does not demonstrate the smooth philtrum, thin upper lip, or small palpebral fissures commonly seen in infants with FAS. FAS would not demonstrate a chromosomal abnormality on karyotyping

A 4-day-old infant presents for his first pediatric well-child visit. His birth history consists of an uncomplicated normal spontaneous vaginal delivery after 7 hours of uncomplicated labor (no vacuum or forceps assistance). The patient is the first child to a 30-year-old mother of Mediterranean descent. Mom is very concerned that her baby has started to look "yellow" since leaving the hospital. She has been breastfeeding every 2-3 hours and says that the baby latches on for 1-5 minutes for each feed. He has had few wet diapers, and mom is concerned he is not getting enough to eat. Which of the following would most aid in narrowing the differential diagnoses?

A. Blood smear B. Fractionated bilirubin C. Newborn screen results D. No further workup is needed, as this is likely physiologic jaundice E. WBC A. A blood smear would be helpful to diagnose hemolysis or RBC membranopathies. However, a blood smear would not be the most useful in narrowing the wide range of possibilities in the current differential, because it would only help with identifying hemolytic causes. At this juncture, a fractionated bilirubin would be the most useful test to order first. B. The test that will give you the most information at this juncture is a fractionated bilirubin. With the knowledge of the total serum bilirubin (TSB) and direct serum bilirubin, one will be able to narrow the differential (hemolysis vs. obstruction), and also to guide treatment (i.e., indirect serum bilirubin may be above phototherapy level). TSB can also indicate if the situation requires more drastic measures, such as a transfusion exchange. C. The newborn screen result normally takes a few weeks to process. However, even if the results were available, they might suggest two possibilities in the differential diagnosis of jaundice: hypothyroidism or G6PD deficiency. Even though these diagnoses are important to rule out, the test would not provide more information about the infant's current state of health or the next step in treatment. D. Rule out a more serious cause. Because the mother is of Mediterranean descent, it is very possible that the cause of jaundice is G6PD deficiency, so further workup may be necessary. At the very least, a fractionated bilirubin is needed, and then likely a CBC. Although physiologic jaundice is the most likely cause of jaundice at this age for an infant, there is not yet enough information to rule out a more serious cause. Because the mother is of Mediterranean descent, it is very possible that the cause of jaundice is G6PD deficiency, so further workup is definitely necessary. At the very least, a fractionated bilirubin is needed first. E. A WBC could be helpful if the infant is at risk or showing signs of infection or sepsis. This infant did not have a complicated birth history, and does not have signs of fever or distress. Although it may be helpful to check a CBC for the hemoglobin level, in case of hemolysis, a WBC will not help narrow the differential or guide the management as much as a fractionated bilirubin given the above presentation.

An 8-year-old girl is brought to clinic because of intermittent, crampy abdominal pain that has persisted over the last three months. The pain is nonspecific, nonfocal, and not associated with any other systemic symptoms such as fever, chills, weight loss, nausea, vomiting or diarrhea. The pain also seems to occur more frequently during the week and not as often on weekends. The abdominal exam is normal. The patient is given a diagnosis of functional abdominal pain. Six months later, she returns to the clinic complaining of more frequent, more severe abdominal pain that is waking her up at night. She also reports a week of diarrhea containing mucus and blood without associated fever or vomiting. Review of her growth chart demonstrates a slowing of weight gain and a drop in height velocity. What is the most likely diagnosis?

A. Celiac disease B. Crohn's disease C. Giardiasis D. Henoch-Schonlein purpura (HSP) E. Irritable bowel syndrome SUBMIT A. This answer is incorrect. Celiac disease is an autoimmune disorder of the small intestine characterized by diarrhea, failure to thrive, and fatigue. A rash known as dermatitis herpetiformis (an autoimmune cutaneous eruption) can also occur. This is not the most likely cause of this patient's abdominal pain as diarrhea in Celiac disease is not usually grossly bloody. B. Crohn's disease is most consistent with this presentation, as it affects GI tract from mouth to anus, leading to abdominal pain, diarrhea (can be bloody), vomiting, or weight loss. Extraintestinal symptoms include skin rashes, arthritis, and fatigue. Fever, fistula, and perianal complications are also common. Slowed weight and height velocity may be observed. C. This answer is incorrect. Giardiasis is a parasitic disease caused by flagellate protozoan that inhabits the digestive tract leading to decreased appetite, diarrhea, hematuria, loose or watery stool, stomach cramps, bloating, excessive gas, and burping. Many people with giardia infection are asymptomatic. Untreated symptoms may last six weeks or longer. Diagnosis is made via stool microscopy, ELISA, or entero-test with gelatin capsule and thread. D. This answer is incorrect. Irritable bowel syndrome is a symptom-based diagnosis with chronic abdominal pain, discomfort, bloating, and alteration of bowel habits. It is often a diagnosis of exclusion, with no known organic cause. Other symptoms include diarrhea or constipation. E. This answer is incorrect. Henoch-Schonlein purpura (HSP) presents with abdominal pain, palpable purpura on the lower extremities, joint pain, and often kidney involvement. It is an acute systemic vasculitis characterized by deposition of immune complexes containing IgA. Abdominal pain is colicky often with nausea, vomiting, constipation, or diarrhea.

A 16-year-old female presents to clinic complaining of worsening fatigue. Family history is significant for hypothyroidism and heavy periods in the grandmother. Her exam reveals mild tachycardia and oozing around a recent piercing, but is otherwise normal. Labs reveal Hgb 8.5 g/dL, MCV 58, PT 12.5, PTT 44, and low von Willebrand factor activity. Which of the following is the most appropriate treatment for her underlying disorder?

A. Blood transfusion and iron supplementation B. Cryoprecipitate C. Desmopressin D. Factor VIII concentrate E. Vitamin K A. This patient has a microcytic anemia, as indicated by her hemoglobin level and MCV. Iron deficiency anemia is the most common type of microcytic anemia in adolescents. Nutritional deficiency is the most common cause of iron deficiency anemia, and in these cases iron supplementation would be appropriate. However, a blood transfusion and supplemental iron for a presumed nutritional deficiency without investigating potential ongoing losses is inappropriate, especially in a patient with oozing blood on exam. B. Cryoprecipitate is a blood product containing fibrinogen, factor VIII, factor XIII, and von Willebrand factor. It is most commonly used to replace fibrinogen. It may also be used in certain scenarios for patients with von Willebrand disease. However, it is not as good as desmopressin as a first choice in this scenario. C. Von Willebrand disease is the underlying cause of this patient's anemia, as indicated by the low von Willebrand factor activity. This is the most common hereditary bleeding disorder, occurring in roughly 1% of the population. Intranasal or intravenous desmopressin is appropriate treatment for most bleeding problems. Desmopressin works by causing release of von Willebrand factor from vessel endothelial cells. D. Factor VIII concentrate may be used as a treatment for von Willebrand disease in the setting of major trauma or surgery. It also may be used for less common subtypes of the disease that do not respond to desmopressin. However, most cases of von Willebrand disease can be treated with desmopressin, and therefore it is the most appropriate first choice in treating this patient. E. Vitamin K is found in leafy green vegetables and in most multivitamins. It is important in the synthesis of clotting factors. It is given clinically as a one-time IM dose for neonates to assist in clotting factor formation until the gut microbiome is sufficient to provide Vitamin K. It is also used for patients who become supratherapeutic on warfarin therapy. It would not help in this case.

A two-week-old infant presents to the clinic to follow-up on abnormal newborn screening results. The routine newborn screen showed increased levels of immunoreactive trypsinogen which is concerning for cystic fibrosis. Confirmatory testing is sent. What signs and symptoms should be monitored in this infant?

A. Bloody stools B. Diaphoresis during feeds C. Increased stooling D. Precordial thrill E. Reflux CF causes exocrine pancreatic insufficiency and ultimately results in fat malabsorption. This causes frequent loose stools in infancy and progresses to foul-smelling and greasy stools in childhood. A. This answer choice is incorrect. Bloody stools would be expected in a child who had gastroenteritis or mild protein allergy, not CF. B. This answer choice is incorrect. Diaphoresis during feeds would raise concern for congestive heart failure. The classic associated signs include tachypnea, tachycardia, and hepatomegaly. This is not typical for cystic fibrosis. D. This answer choice is incorrect. A precordial thrill would be indicative of a cardiac problem, and cardiac issues are not typically associated with CF. E. This answer choice is incorrect. There is no association between gastroesophageal reflux disease and CF.

A 3-year-old male presents with fever to 39.4 C for the past week, injected eyes, and a refusal to walk for the past two days. On physical exam, you note conjunctival injection without exudates bilaterally, prominent papillae of his tongue with redness as well as redness of his hands and feet. He also has a new diffuse maculopapular rash on his torso that becomes more prominent with fever. On examination of the swollen extremities, you are unable to elicit any tenderness or effusions in any joints. Which of the following is the most likely diagnosis?

A. Bone or joint infection B. Kawasaki disease (KD) C. Rocky Mountain Spotted Fever (RMSF) D. Scarlet fever E. Systemic onset juvenile idiopathic arthritis Kawasaki disease (KD) is one of the most common vasculitides of childhood. For diagnosis, in addition to fever of ≥ 5 days, the patient must meet four of the following criteria: rash, conjunctivitis, unilateral cervical lymphadenopathy, changes in oral mucosa, or extremity changes (redness/swelling). Our patient does not have lymphadenopathy, but often this is the least common finding in KD. If children have fever with fewer than four of the five clinical findings, they can have "incomplete KD" if they meet certain supplemental laboratory criteria. A. This answer choice is incorrect. Bone or joint infection should be on the differential given the patient's refusal to walk, as up to 80% of these infections are in the lower extremities. In addition, the lack of localized symptoms of warmth and tenderness associated with the lower extremity erythema and swelling make this diagnosis less likely in this patient. C. This answer choice is incorrect. Rocky Mountain Spotted Fever (RMSF) is a tick-borne disease caused by Rickettsia rickettsii. This tick is commonly found in southeastern parts of the U.S., and patients will often come from or have a history of travel to that region. The disease is characterized by headache, fever, myalgia, and a centrally progressing petechial rash originating on the wrists and ankles. The maculopapular rash, and constellation of other symptoms, makes this diagnosis less likely in this patient. D. This answer choice is incorrect. Scarlet fever is caused by erythrogenic toxin produced by Streptococcus pyogenes. Symptoms can include sore throat, fever, "strawberry tongue" and a blanching, erythematous rash. The areas affected by the rash may desquamate after six to seven days of infection. While our patient does have a "strawberry tongue," fever, and rash, the description of the rash and other physical exam findings are more consistent with KD than with scarlet fever. E. This answer choice is incorrect. Systemic onset juvenile idiopathic arthritis, also known as Still's disease, is a subset of JIA describing patients with intermittent rash, fever and arthritis. Even though this patient presents with rash and fever, as well as refusal to walk (potentially a sign of arthritis), systemic onset JIA tends to present with a history of spiking fevers and an evanescent, macular, salmon-pink rash occurring when the child is febrile, and disappearing as the fever fades. This is inconsistent with the description of the findings seen in the patient above, who does not demonstrate tenderness or effusion in any joint.

A 12-year-old female presents to her pediatrician complaining of a headache of gradual onset x 3 hours, non-provoked and described as a "big rubber band around my whole head" and a 5 out of 10 on the pain scale. The pain is not throbbing, and there is no associated photophobia, nausea or vomiting. The patient is afebrile, and there are no neurologic deficits during physical exam. Her mother states her pain is typically relieved with ibuprofen, but her mother is concerned that the patient may have migraines because she has a few headaches every month after school. The child is otherwise healthy. What is the most likely cause of her headaches?

A. Brain tumor B. Idiopathic intracranial hypertension C. Migraine D. Sinusitis E. Tension-type headache Tension-type headaches are often bilateral and involve the forehead, temporal areas, or back of the head. Tenderness of the posterior muscles of the neck may also be present. They should be responsive to NSAIDs. Stress can give rise to a tension headache, and this is consistent with this patient developing headaches after school. A. This answer choice is incorrect. The evolution of headache over hours would be atypical for a brain tumor, as is the intermittent nature and response to NSAIDs. In addition, the absence of focal neurological findings is reassuring.Headaches of new onset that do not go away and are worse in the mornings and associated with vomiting, personality changes or focal neurological finding would be most concerning for a brain tumor etiology. B. This answer choice is incorrect. Idiopathic intracranial hypertension presents as symptoms of increased intracranial pressure—headache, nausea, blurred vision, diplopia, photophobia, and tinnitus—that occur in the absence of a intracranial mass. CT/MRI will be negative, but opening pressure is elevated on lumbar puncture. In this patient, tension headache is more likely given the lack of symptoms of increased intracranial pressure. C. Migraines are typically described as having "throbbing" pain and may also be associated with nausea and/or vomiting with photophobia and phonophobia. Pain is often severe and may be debilitating. D. This answer choice is incorrect. The patient is otherwise healthy and did not complain of any nasal stuffiness, rhinorrhea, or sore throat with postnasal drip, making sinusitis unlikely.

A concerned mother brings her 7-day-old infant to your office after noticing yellowing of his skin for two days. She has also noticed he has not been gaining weight since she brought him home from the hospital five days ago. Mother reports she is feeding the baby six times per day for 10 minutes each time. Mother admits her breasts often feel full and are not relieved by nursing. The baby was born full term by spontaneous vaginal delivery but has a hard time sucking with breastfeeding. Upon exam, he looks dehydrated and appears to have jaundice of the face and chest. He has also lost > 10% of his birth weight. What is the most likely cause of his jaundice?

A. Breastfeeding jaundice B. Breast-milk jaundice C. Crigler-Najjar syndrome D. Physiologic jaundice E. Sepsis A. Breastfeeding jaundice is the correct answer because it usually appears early in the first week of life and is caused by various factors, including poor breast milk intake. A decreased milk supply leads to limited enteral intake and can lead to increased enterohepatic circulation. Increased enterohepatic circulation describes the process where unconjugated bilirubin is reabsorbed into the bloodstream and is recirculated - causing a rise in serum levels of bilirubin. B. Breast-milk jaundice is incorrect, because although it is a cause of jaundice associated with breastfeeding, it is NOT a result of low milk volume. In the above vignette, the infant does not appear to be breastfeeding well, due to poor suck and low number of feeds for the infant. Breast-milk jaundice is more likely to be present in a 1-2 week old baby who is breastfeeding well. C. Crigler-Najjar syndrome is an incorrect choice because it appears within the first days of life, persists thereafter, and is a relatively rare disease. This type of jaundice is caused by decreased bilirubin clearance from deficient or completely absent UDPGT (the enzyme that conjugates bilirubin with glucuronide to make it water-soluble and able to undergo excretion into bile). D. Physiologic jaundice is incorrect because this jaundice usually peaks at three to four days of life in full-term, healthy infants. This type of jaundice is not associated with breastfeeding but could be from numerous factors such as increased bilirubin production from short-lived RBCs or lack of intestinal flora to metabolize bile. No treatment is required for this type and it typically resolves on its own. E. Sepsis is incorrect because the infant does not clinically appear sick or toxic, and jaundice would be just one sign of a serious infection. Other symptoms may include respiratory distress, lethargy, poor feeding, vomiting, apnea, and temperature instability. Sepsis causes an elevated total and direct bilirubin and is highly unlikely when jaundice is the only symptom. Breastfeeding can have some protection against infection.

A 30-minute-old infant is evaluated in the nursery for tachypnea. He was born at 30 5/7 weeks gestation by vaginal delivery. Labor was induced due to severe maternal pre-eclampsia. Mother received no prenatal care. Apgars were 6 and 7 at 1 and 5 minutes, respectively. One dose of betamethasone was given prior to delivery. A chest x-ray reveals decreased lung expansion. Which of the following diagnoses is most consistent with this patient's presentation?

A. Bronchopulmonary dysplasia (BPD) B. Meconium aspiration syndrome (MAS) C. Persistent pulmonary hypertension (PPHN) D. Respiratory distress syndrome (RDS) E. Transient tachypnea of the newborn (TTN) The baby boy is preterm, and his mother received only one dose of betamethasone, which puts him at increased risk for developing infant RDS, which is caused by insufficient surfactant. His physical exam and chest x-ray findings are consistent with RDS. A reticulogranular or "ground-glass" appearance and bronchograms may be present shortly after birth, but take 12-24 hours to maximally develop if surfactant is not administered. A. This choice is incorrect, because BPD is the result of prolonged mechanical ventilation. Our patient is at risk for developing this syndrome if he requires intubation. Chest x-ray may show atelectasis, inflammation, or pulmonary edema. With severe disease, the chest x-ray may reveal fibrosis and hyperinflation. B. This choice is incorrect. Although the presence or absence of meconium was not noted in the case, it is known that from 20 to 34 weeks' gestation, the fetus will pass meconium infrequently. Most cases of MAS are in term or post-term infants. On chest x-ray, we might see overdistention of the lung or other sequelae, such as pneumothorax. C. This choice is incorrect. PPHN generally occurs in babies born after 34 weeks. There are several causes for PPHN: underdevelopment, maldevelopment, and maladaptation of the lungs. Underdevelopment of the lungs can be secondary to congenital diaphragmatic hernia, oligohydramnios in utero, IUGR, or renal agenesis. The underdevelopment causes increased pulmonary vascular resistance and has a poor prognosis. Maldevelopment involves remodeling of pulmonary vasculature and is associated with post-term delivery and meconium aspiration syndrome. Maladaptation can be caused by infection with GBS. Vasoactive mediators are activated by bacterial phospholipids, causing an increase in pulmonary vascular resistance. E. This choice is incorrect. TTN is a disorder of delayed reabsorption of fluid in the newborn's lungs. Prematurity, delivery by C-section, being large or small for gestational age, or having a diabetic mother are all risks. In order to be diagnosed with TTN, the baby would need to show improvement within several hours. Although this is on the differential for the newborn baby's condition based on clinical presentation, a chest x-ray should have shown perihilar streaking and other evidence of interstitial fluid.

A 6-month-old infant is referred to the pulmonology clinic because he was recently diagnosed with cystic fibrosis. His parents have several questions about this diagnosis. Which of the statements about cystic fibrosis is most correct?

A. CF is a disease that exclusively involves the respiratory system B. CF is an autosomal dominant disorder C. CF is caused by a mutation in CFTR, resulting in defective salt balance D. Gene therapy is now the primary source of CF therapy E. It is important to provide calories at a lower level than recommended dietary allowance for a given age in order to prevent GI upset The mutation in the CFTR gene results in defective salt balance. CF is caused by mutations in a single large gene on chromosome 7 that encodes the cystic fibrosis transmembrane conductance regulator (CFTR) protein. Clinical disease requires disease-causing mutations in both copies of the CFTR gene. A. This answer choice is incorrect. CF involves the respiratory tract-leading to chronic cough and recurrent infections-but may also lead to pancreatic dysfunction leading to malabsorption and diabetes, infertility, and malnutrition. B. This answer choice is incorrect. CF is autosomal recessive, NOT dominant. Most people with CF do not have a positive family history. D. This answer choice is incorrect. Gene therapy is NOT a current treatment option for CF patients. Gene therapy is currently being investigated as a possible therapy. Current treatment includes a multidisciplinary approach, using treatments such as pancreatic enzyme and vitamin replacement, mucus clearing, diabetes management and lastly, lung transplant. E. This answer choice is incorrect. It is necessary to provide calories at a HIGHER level than the recommended dietary allowance for age to maintain appropriate nutritional status, as CF patients usually have malabsorption.

A 2-year-old male presents to the ED with a 2-day history of vomiting and diarrhea. The patient's father relays a history of abrupt onset of vomiting that started yesterday around 1 pm. He has had 6 episodes of emesis since yesterday and 3 episodes of diarrhea. The emesis is non-bilious and the diarrhea is described as watery. There are no sick contacts in the home. Vital signs: T 37.1, P 102, R 20, BP 90/60. Abdomen is soft, non tender, normoactive bowel sounds, and no masses. Pulses are normal and capillary refill is 2 seconds. He has still been tolerating some PO feeds without instant vomiting. What is the most immediate intervention for this patient?

A. CT scan and surgical consult B. IV bolus with 0.9% saline C. IV bolus with D5W D. No immediate intervention is necessary E. Random glucose test > The correct answer is D. A. This answer choice is incorrect. A CT scan and surgical consult would be indicated in a patient with suspected surgical causes of vomiting and diarrhea- such as appendicitis or obstruction. This patient has a normal abdominal exam, which makes surgical causes less likely. B. This answer choice is incorrect. An IV bolus of 0.9% saline is not indicated in this patient with normal vital signs and no obvious clinical signs of dehydration. An IV bolus would be indicated for an inability to tolerate oral fluids or severe dehydration as evidenced by tachycardia and physical exam signs such as lethargy, weak pulses, or absent tears. C. This answer choice is incorrect. An IV bolus with D5W is indicated in cases of confirmed hypoglycemia. A bolus with D5W is not indicated in this patient. D. This answer choice is correct. This patient most likely has viral gastroenteritis. No immediate intervention is needed as the patient is still tolerating some PO and has a normal physical exam. He should be advised to continue oral hydration and to return if symptoms worsen and the patient can no longer tolerate oral hydration. E. This answer choice is incorrect. A random glucose test would be indicated if the vomiting and diarrhea in this patient is suspected to have caused hypoglycemia. Hypoglycemia is less likely in this patient with normal vital signs and a normal physical examination.

An 18-month-old girl is brought to her pediatrician by her mother who notes that she has been fussy for the past three days and has been pulling on her ears. The child is up to date with her vaccines. Her temperature is 39 C (102.2 F). Otoscopic exam of her left ear shows a yellow, opaque, and bulging tympanic membrane. Which of the following organisms is the most likely cause of the child's condition?

A. Candida albicans B. Haemophilus influenzae C. Moraxella catarrhalis D. Rhinovirus E. Streptococcus pyogenes H. influenzae is the most frequent cause of AOM (50-60%% of cases). Although the child has been vaccinated against H. influenzae type B, this does not cover the unencapsulated strains of H. influenzae that cause AOM. A. This answer choice is incorrect. Candida albicans is not a frequent cause of AOM. An infection involving this organism in the middle ear should immediately raise suspicion for an immunocompromised state. C. This answer choice is incorrect. M. catarrhalis is another common cause of AOM. However, this organism is responsible for only between 12% and 15% of cases, making it a less frequent cause than non-typeable H. influenzae. D. This answer choice is incorrect. Rhinovirus is a potential cause of AOM, but bacterial AOM occurs with a much higher frequency. E. This answer choice is incorrect. The child is suffering from acute otitis media (AOM). S. pyogenes is a rare cause of this condition (< 5% of cases). One should not confuse this species of strep with S. pneumoniae, which used to be the most common cause of AOM and now represents 15-25% % of cases in the pneumococcal vaccine era.

A 3-year-old female comes to the clinic with a chief complaint of fever (40 C) for over a week. Her mother reports that she has been fussy and inconsolable since she became febrile. She has a red tongue with large papillae, conjunctivitis, a palmar rash, unilateral cervical adenopathy, and swollen feet. Given the most likely diagnosis, what is the most important follow-up for this girl over the next few weeks?

A. Cardiology follow-up to rule out presence of rheumatic fever B. Echocardiogram to look for coronary artery aneurysm C. Neurology follow-up to evaluate partial paralysis of lower extremities D. Ophthalmology follow-up to determine extent of eye damage E. Physical therapy follow-up to help prevent long-term joint deformities Children with Kawasaki disease are at high risk for coronary artery aneurysm formation and should receive an echocardiogram as part of their initial evaluation/treatment as well as follow up echocardiograms after discharge. Use of IVIG for the treatment of Kawasaki disease has decreased the risk of coronary artery aneurysms significantly. Kawasaki disease is diagnosed when there is a fever plus four of the following: changes in oral mucosa (e.g., strawberry tongue), extremity swelling or redness, unilateral cervical adenopathy, conjunctivitis, and rash. A. This answer choice is incorrect. The patient in this case has Kawasaki disease. Patients who have scarlet fever could develop rheumatic fever or post-streptococcal glomerulonephritis, among other complications. Scarlet fever is characterized by an "erythematous, blanching, sandpaper-like rash" with very fine papules secondary to infection with Group A streptococcus. It may start in the groin, axilla, or neck, before spreading rapidly over the trunk and extremities. Fever can be high, but generally resolves within five days. C. This answer choice is incorrect. The patient in this case has Kawasaki disease. Neurologic issues such as paralysis of a lower limb can occur in severe cases of Rocky Mountain Spotted Fever, which is characterized by fever, myalgias, headache, and petechial rash classically starting on wrists and ankles and progressing centrally. D. This answer choice is incorrect. The patient above likely has Kawasaki disease. Early referral to an ophthalmologist is important for patients diagnosed with other conditions, including Stevens Johnson Syndrome (SJS) to determine the degree of eye involvement and if treatment with topical steroids is needed. SJS is a mucocutaneous disorder defined by fever, severe stomatitis (inflammation of the mucous lining of any of the structures in the mouth), conjunctivitis, and a blistering rash. It is typically precipitated by medications or infections. Early referral to an ophthalmologist would also be important for patients diagnosed with juvenile idiopathic arthritis because they can suffer from uveitis, which if left untreated, can lead to long-term problems such as cataracts, glaucoma, or blindness. E. This answer choice is incorrect. The patient in this case has Kawasaki disease. Joint involvement in Kawasaki disease resolves without sequelae. Early referral to physical therapy is important for patients diagnosed with systemic juvenile idiopathic arthritis, which is characterized by prolonged or spiking fever, rash, and arthritis. Early treatment and physical therapy can help prevent joint deformities and improve long-term functionality.

A 7-year-old male presents to the clinic with reports of headaches and episodes of feeling sweaty and flushed. He also reports that at times he feels as if his heart is racing. He was full-term, had an uncomplicated birth, and has been otherwise healthy until now. On exam, his BP is 120/80 mmHg and is the same in his upper and lower extremities. His weight and height are in the 50th percentile for his age. What is the most likely cause of this patient's hypertension?

A. Catecholamine excess B. Coarctation of the aorta C. Primary hypertension D. Renal insufficiency due to renal scarring E. Renal vascular disease Catecholamine excess (pheochromocytoma or neuroblastoma) should be suspected in a child who is hypertensive and has episodes of sudden sweating, flushing, or feels that his heart is racing. This patient is exhibiting these signs and urine catecholamine testing would be appropriate in this case. B. This answer choice is incorrect. Coarctation of the aorta should be suspected in a child with elevated BP (usually > 99th percentile), and a discrepancy between upper and lower extremity BPs. Some children with coarctation of the aorta may go undetected until presenting with hypertension at a school-age visit. It is important to pay special attention to the femoral pulses and to document BP measurement in a lower extremity. This patient's BP in his upper and lower extremities are the same, making this diagnosis less likely. C. This answer choice is incorrect. Most hypertension in children over 6 years of age, and in adolescents, is due to primary HTN. Obesity is often an important correlate. However, this patient's flushing and racing heart rate would not be fully explained by primary hypertension. Also, this patient is an appropriate weight, making this diagnosis less likely. D. This answer choice is incorrect. Recurrent urinary tract infections in childhood are one of the leading causes of hypertension and renal insufficiency later in life due to renal scarring following infections. UTIs are more common in girls. This patient does not have a stated history of urinary tract infections, and his palpitations and flushing are not consistent with hypertension secondary to UTIs. E. This answer choice is incorrect. Umbilical arterial or venous lines as neonate (most often in premature infants) can predispose a child to renal vascular disease. This patient had an uncomplicated birth and did not go to the neonatal ICU, making it less likely that his hypertension is due to renal vascular disease secondary to an umbilical arterial or venous line as a neonate.

A 4-year-old female with a history of type 1 diabetes mellitus was admitted to a local hospital for treatment of DKA. A few hours after the treatment, she develops grunting, irregular respirations, and has vomited twice. On exam, when asked to gaze forward, her left eye is pointing downward and out. Her diastolic blood pressure is 90 mmHg. What is a likely diagnosis?

A. Cerebral edema B. Hypoglycemia C. Hypokalemia D. Hyponatremia E. Pneumonia with possible sepsis A. This choice is correct. Administration of bicarbonate during DKA treatment increases the risk of cerebral edema. Although symptomatic cerebral edema is rare (less than 1%), it is associated with a high mortality rate (over 20%). The signs of cerebral edema are described in the vignette, and include irregular respirations, headache, vomiting, cranial nerve palsies (III or VI), and high blood pressure. B. This choice is incorrect because although hypoglycemia can be a complication of DKA treatment, and can cause mental status changes, in isolation, it would be unlikely to cause a cranial nerve palsy (abducens [IV]). C. This choice is incorrect. Hypokalemia can be a known complication of DKA treatment. Symptoms affecting muscle, cardiac and renal tissue become apparent at serum levels less than 3 mEq/L. Skeletal muscle weakness and myalgias, cardiac arrhythmias, gastro-intestinal ileus, and renal concentrating difficulties are the most common clinical manifestations. The symptoms listed in the vignette above are not specifically due to hypokalemia. D. This choice is incorrect because children in DKA can have hyponatremia due to the dilutional effect caused by water moving from the intracellular to the extracellular compartment drawn by the increased plasma osmolarity from hyperglycemia. This hyponatremia is corrected with the DKA treatment and would not present with signs of raised intracranial pressure, such as a cranial nerve palsy. E. This answer is incorrect. In children with DKA, it is always necessary to investigate children for signs of an intercurrent illness that could have precipitated the current state. The child's irregular respirations are deep and irregular (Kussmaul respirations), which is unlike the increased work of breathing and rapid breathing which is more suggestive of pneumonia. Pneumonia does not present with a cranial nerve palsy.

An 18-month-old boy comes to the clinic for a well-child check. His mother says he is a happy boy and endorses no complaints. She notes that he was born a few weeks early via emergency C-section and spent two months in the NICU. She says he is feeding well and gaining weight. When you ask about developmental milestones, you find out that that he pulls himself up to stand, but is not walking yet. He has several words and is interactive. There is no family history of disease during infancy. He is up to date on his immunizations. He is one of five children and his parents are currently experiencing difficult financial times. Exam reveals increased tone and hyperreflexia in his lower extremities. What is the most likely diagnosis of his developmental delay (if any)?

A. Cerebral palsy B. Genetic abnormality C. Normal variant D. Metabolic disorder E. Psychosocial stress reaction Children with cerebral palsy often present with defects in motor development and are often found to have abnormal neuromuscular exams, including increased tone and reflexes. Other areas of development are less consistently impacted. The cerebral palsy most likely was the result of an hypoxic injury during the perinatal period. B. This choice is incorrect because of a lack any characteristic syndromic phenotype and negative family history, and the delay appears to be limited to his gross motor exam. C. This choice is incorrect because this child has profound motor delay and abnormal neurologic findings (hypertonia). D. This choice is incorrect because this child's newborn screening exam was normal, he has no family history of metabolic disease, and has not had any indication of any metabolic abnormalities. E. This choice is incorrect because of his persistence of developmental delay and lack of behavioral changes (temper tantrum, sleep disturbance, etc.). Temporary loss of developmental achievements can be seen in times of stress.

Case 6-- John is a 17-year-old presenting today for a preparticipation physical exam. During the interview, he reports a low-grade fever, malaise, and headache for one week. In the past few days, his fever has gotten worse and he complains of a sore throat. He denies cough or chest pain. On physical examination, he is found to have a temperature of 101.3° F, and cervical lymphadenopathy and oropharyngeal erythema with exudate are noted. His participation would be most likely affected by which of the following tests?

A. Chest x-ray B. CT head/neck C. EBV serologies D. No further workup E. Throat culture The patient's symptoms are suggestive of infectious mononucleosis. These include complaints of low-grade fever and malaise and findings of cervical lymphadenopathy and pharyngeal exudate. If testing is positive, the patient should be restricted from strenuous activity or contact sports during his illness due to the risk of splenic rupture. A. This answer choice is incorrect. There is no evidence of lower respiratory tract signs or symptoms and therefore CXR would not be indicated, nor affect his participation in sports. B. This answer choice is incorrect. A CT scan of the head and neck is not indicated for this patient's presentation, and also would be unnecessary radiation. D. This answer choice is incorrect. The patient's symptoms and findings warrant further evaluation. E. This answer choice is incorrect. An antigen test (and culture, if negative) should be performed for streptococcal pharyngitis, but this clinical picture is more consistent with EBV, and mononucleosis is more likely to preclude participation in sports. After 24 hours of antimicrobials, the patient with streptococcal pharyngitis is no longer contagious.

Claire is a 16-year-old girl who presents for birth control management. Her review of symptoms is unremarkable except for chest pain.Upon further questioning, she reveals the pains are intermittent, on and off for the past couple months. It is not associated with exertion, and is described as sharp, and well localized at the left sternal border. It is very brief, lasting only a few seconds, during which she says she sometimes notices it gets worse when she breathes in. She denies recent URI or viral illness. The family history is negative for early cardiac disease. Her vital signs and physical exam are normal. Which is the next best step in management?

A. Chest x-ray B. ECG C. Fast ultrasound of pericardial window D. Reassurance E. Referral to a cardiologist The history and normal physical exam in this patient is most suggestive of precordial catch syndrome, the most common cause of chest pain in an adolescent. No further workup is needed. A. This answer choice is incorrect. A chest x-ray would be indicated if you suspected pneumonia as a cause of this patient's chest pain. However, the patient does not have cough and is afebrile, making this less likely. B. This answer choice is incorrect. An ECG would be the next step if you suspected a cardiac abnormality. This would be more likely if the patient described something more like angina, a crushing chest pain or pressure, for longer periods of time rather than a few seconds, and aggravated by exercise. C. This answer choice is incorrect. Fast ultrasound would be the management if you suspected pericarditis or a pericardial effusion, but the patient denies recent URI or viral infection, and your PE is not notable for a pericardial friction rub suggestive of pericarditis. E. This answer choice is incorrect. Associated symptoms of syncope or palpitations in this patient would suggest a more severe cardiac abnormality and would prompt a referral to a cardiologist. The vignette states the rest of the ROS is negative, therefore would not warrant further evaluation by a cardiologist.

A 16-month-old boy is brought to the ED by ambulance. His father called 911 after the patient's eyes deviated to the left as his arms and legs were twitching. During this time he was unresponsive. He has had a tactile fever for three days, and parents mention that he has not been as playful as usual during this time as well. His parents have not had him vaccinated due to personal beliefs. In the ED his vital signs are T 39.1°C, HR 155 bpm, RR 28 bpm, BP 100/65 mmHg, O2 100% (on RA). He does not cry or pull away from you but whimpers during most of your physical exam (including when you look in his ears). You order a CBC and metabolic panel, which are significant for a leukocytosis with a left shift and mild acidosis. Urinalysis and blood/urine cultures are pending. Which of the following additional studies would you obtain after stabilization?

A. Chest x-ray B. Electroencephalogram (EEG) C. Lumbar puncture D. Toxicology screen A young child with fever and altered level of consciousness should lead to a high suspicion for meningitis. This patient's parents expressed concerns about his behavior at home before his seizure, and his mental status during your examination is not normal. While very few patients presenting with febrile seizure actually have meningitis, this patient's lack of immunizations put him at increased risk. Furthermore, clinical signs of meningitis in young children can be very subtle, and so a high level of suspicion is important. A lumbar puncture will help rule in or out meningitis and guide treatment. Note that in some cases the clinician will request a head CT prior to performing a lumbar puncture if there are concerns about increased intracranial pressure. A head CT in itself may not be helpful in the evaluation of a patient with a seizure, although it may be useful in cases where trauma is suspected, or to look for calcifications (such as with cytomegalovirus infection or tuberous sclerosis). A. This answer choice is incorrect. The history given is not suggestive of any pathologic lung process (no cough, respiratory distress, or desaturation). While it is possible that pneumonia could be causing his fever, a chest x-ray is not indicated at this time. B. This answer choice is incorrect. For children with focal, recurrent, or complex seizures, an EEG may be helpful to establish a diagnosis of epilepsy. Sixty percent of children and infants may have an interictal epileptiform abnormality. In this case it will not help to guide management acutely. D. This answer choice is incorrect. It is important to consider toxin ingestion in a patient with altered level of consciousness and seizure. A detailed history about possible exposures should be obtained from the family. However, this patient's fever and leukocytosis make infection the more likely diagnosis.

Case 13-- A 4-year-old boy who recently emigrated from eastern Europe presents with his mother to your general pediatrics clinic. His mother reports that he has a chronic nonproductive cough during the day and night, mild wheezing for one month and failure to gain weight (his weight has dropped from the 50th to the 10th percentile for his age). He does not have high fevers, rhinorrhea, congestion or night sweats. Which of the following are the next best diagnostic tests?

A. Chest x-ray and tuberculin skin test B. CT of nasal sinuses C. Spirometry, before and after bronchodilator therapy D. Chest x-ray and methacholine challenge E. None needed, patient likely has habit cough CXR and tuberculin skin test (TST) is the best choice. Signs and symptoms of primary pulmonary tuberculosis are few to none. Toddlers may present with nonproductive cough, mild dyspnea, wheezing, and/or failure to thrive (defined as weight < 5th percentile or drop in two percentile curves for weight). In children, TB can present without systemic complaints (fever, night sweats, and anorexia), severe cough, and sputum production. Regarding diagnostic tests, the TST is a practical tool for diagnosing TB infections. All children with chronic cough (more than four weeks) should be evaluated with a chest x-ray, as other pathology—such as lung abscess or malignancy—can also be detected on CXR. B. This answer choice is incorrect. Sinusitis is often preceded by a URI, with nasal congestion as a prominent feature, leading to nocturnal cough due to post-nasal drip. These symptoms are not seen in our patient. Furthermore, a diagnosis of sinusitis is made clinically, with CT scan obtained only in complicated cases or cases resistant to treatment. Rare complications include cavernous sinus thrombosis, meningitis, and epidural abscess. C. This answer choice is incorrect. Spirometry (pulmonary function testing) before and after bronchodilator therapy is the most specific means of determining whether or not a child has asthma. Asthma is a very common diagnosis in children, and may present with cough that is worse at night and exacerbated by exercise and cold air. Patients with cough-variant asthma present with only cough, typically nonproductive. However, spirometry is difficult to perform in patients under 5 years old. In addition, given this patient's failure to thrive, a more serious diagnosis such as TB must be considered. Also, a chest x-ray is needed in all children with chronic cough (more than four weeks). D. This answer choice is incorrect. Although a chest x-ray is appropriate in all children with chronic cough, a methacholine challenge (for asthma) would be inappropriate in this scenario. Although asthma is a common diagnosis, given the patient's failure to thrive, a more serious diagnosis must be considered. Further, like spirometry, a methacholine challenge would be difficult to perform in a child less than 5 years of age. E. This answer choice is incorrect. Habit cough is typically very loud, short, dry, brassy, and spasmodic. This cough is unchanged by exercise or cold air, and classically resolves during sleep. The diagnosis is usually made by exclusion of other serious causes of chronic cough.Although the patient in this case has a dry cough, his failure to thrive points to a more serious diagnosis (e.g., TB). All children with chronic cough (persisting longer than four weeks) need a CXR.

A 5-year-old boy is noted to have a grade II systolic murmur and a widely split S2 murmur on cardiac exam. His vital signs are stable and he has been asymptomatic. Which of the following statement is accurate regarding this child's presentation and likely condition?

A. Chest x-ray, ECG, and echocardiogram would be indicated as next steps to work up a presumed ventricular septal defect B. No further work-up for a presumed venous hum C. This patient's murmur is caused by excessive flow through the pulmonary outflow tract and should be evaluated D. This patient should be scheduled for cardiac catheterization A. This answer choice is incorrect. The first steps in the evaluation of presumed structural heart disease would include a chest x-ray, ECG, and echocardiogram. However, the clinical presentation is not consistent with a ventricular septal defect, which usually presents with a holosystolic murmur. B. This answer choice is incorrect. A benign venous hum would not be expected to present with a widely split second heart sound. C. This answer choice is correct. This patient's murmur is likely caused by an atrial septal defect, which causes additional blood flow through the pulmonary outflow tract (representing the combined normal systemic venous return to the right heart plus additional blood coming across the ASD) and should be evaluated. D. This answer choice is incorrect. Cardiac catheterization would not be indicated as first line evaluation. Noninvasive testing such as an ECG and echocardiogram would be indicated before considering a cardiac catheterization for diagnostic/therapeutic intervention.

An asymptomatic, healthy 9-month-old is found to have a palpable RUQ mass on exam. After further imaging and lab studies, the mass is diagnosed as a neuroblastoma that has involvement in the bone marrow as well. The mother is worried about the prognosis. Which of the following is true about the prognosis of neuroblastoma in this child?

A. Children who are older than 12 months have a better prognosis than younger children B. Favorable histology does not play a role in prognosis C. Lymph node involvement is a poor prognostic factor D. Non-amplification of the n-myc gene is a favorable prognostic factor E. Prognosis of neuroblastoma is predictable A. This answer choice is incorrect. In infants less than one year of age, neuroblastoma tumors may spontaneously regress. Stage 4S neuroblastoma is a special category that is reserved for infants less than 12 months who have resectable primary tumors and metastases to the liver, skin, and bone marrow. Overall survival is over 85 percent for babies over 6 weeks of age with Stage 4S. B. This answer choice is incorrect. Favorable histology is a good prognostic factor in neuroblastoma, and is based on the differentiation of the cells involved. C. This answer choice is incorrect. Due to the effectiveness of chemotherapy, neuroblastomas with lymph node involvement are still considered favorable, especially in the setting of other favorable factors, such as young age and differentiating histology. Though distant metastasis is a significant poor prognostic factor, regional lymph nodes do not significantly affect the outcome. D. This answer choice is correct. Non-amplification of the n-myc gene is one of the favorable genetics in neuroblastoma. E. This answer choice is incorrect. Neuroblastoma has a broad spectrum of clinical courses. Some tumors may spontaneously regress, some may mature to a benign type, and yet other tumors can be very aggressive with metastases. Age plays a role in the prognosis, as most infants have a good prognosis even with disseminated disease, while infants over 18 months of age do not do as well.

A 1-month-old infant who is < 3rd percentile for weight presents to the clinic. He is breastfed every 2 hours and latches on well. However, he has frequent non-bilious episodes of vomiting that have been increasing over the past week despite his mother instituting "reflux precautions." He does not have mucus or blood in his stool. Physical exam reveals a small, olive-sized mass in his abdomen. What is the most likely diagnosis?

A. Cleft palate B. Cystic fibrosis C. Munchausen syndrome by proxy D. Non-organic failure to thrive E. Pyloric stenosis A. This answer choice is incorrect. Children with structural anomalies of the palate typically present with difficulty feeding. This child appears to be latching on well, but has difficulty retaining food once ingested. B. This answer choice is incorrect. Children with cystic fibrosis typically present with failure to thrive secondary to chronic malabsorption, with characteristic loose and malodorous stool. Lab testing reveals elevated sweat chloride. There may be a known family history. C. This answer choice is incorrect. Although maltreatment of a child should always be considered, particularly in the youngest children, this child has sufficient findings by history and physical exam to suspect true organic etiology. D. This answer choice is incorrect. Several findings in this patient point toward an organic rather than an inorganic cause of failure to thrive, and organic causes such as pyloric stenosis should be ruled out before an inorganic cause is assumed. E. This answer choice is correct. The history of frequent vomiting, poor weight gain, and the finding of an abdominal mass are consistent with pyloric stenosis. Children with pyloric stenosis often present between 3-5 weeks of age.

A 7-year-old male with a past medical history of headaches presents with increased frequency and severity of headaches along with new onset vomiting. When the boy was walking into the room, he had a wide stance and nearly tripped twice. Which of the following is the most appropriate next step?

A. Computed tomography B. Daily headache diary C. Intraventricular pressure monitoring D. Lumbar puncture E. Magnetic resonance imaging MRI is more expensive and less readily available than CT imaging. It also frequently requires sedation in pediatric patients. However, it provides the best detail of the posterior fossa, which is the most common location of pediatric brain tumors. A. This answer choice is incorrect. CT imaging is faster and more convenient than an MRI and is very valuable in ruling out intracranial hemorrhage. However, it does not visualize the posterior fossa as well as MRI and would not be the modality of choice if MRI is available. B. This answer choice is incorrect. A daily headache diary is a valuable clinical tool. It should include: characterization, duration, location, associated symptoms, recent activity, aggravating, and alleviating factors. This particular case includes multiple concerning symptoms and requires a more in-depth workup. C. This answer choice is incorrect. Intraventricular pressure monitoring involves the placement of an intraventricular catheter, which measures pressure inside the skull and sends measurements to a recording device. This option may reveal increased intracranial pressure, but would not be helpful in the diagnosis. A brain MRI would be more appropriate. D. This answer choice is incorrect. Lumbar puncture can be very helpful in the diagnosis of meningitis, but would not be appropriate in this case. This patient may have increased intracranial pressure, and lumbar puncture is contraindicated in patients with signs of increased intracranial pressure because it may lead to brain herniation.

You see a 6-year-old male in the ED who presents with a history of a 10-second episode of jerking movements of his extremities with unresponsiveness, observed by both of his parents. His parents claim he has had abdominal pain and small quantities of bloody diarrhea for two days. The child has no significant past medical history, has taken no medications recently, has no pets, and has not traveled outside of California in the past year. He attends kindergarten. Which organism is the most likely cause of the child's symptoms?

A. Clostridium difficile B. Enterotoxigenic E. coli (ETEC) C. Rotavirus D. Shigella sonnei E. Vibrio cholerae A. Clostridium difficile typically causes diarrhea after antibiotic use, which allows the organism to overgrow the normal intestinal flora. It would not be expected to cause diarrhea in an otherwise healthy child who is not taking any medications. Infection can be diagnosed by the presence of C. difficile toxin in the stool. B. ETEC is the strain of E. coli that commonly causes traveler's diarrhea, a noninvasive infection without WBCs in the stool. Diarrhea is typically watery and contains no blood. C. Rotavirus is less likely to be associated with seizures. D. Shigella sonnei causes bloody diarrhea and WBCs in the stool on Wright stain. Children infected with Shigella can suffer from seizures due to neurotoxin release. E. Vibrio cholerae causes massive quantities of watery diarrhea, and patients may present with severe dehydration. Bloody diarrhea would not be expected. It is typically seen in endemic or epidemic form in developing countries, and would not be expected in a child living in the U.S. with no recent travel history.

You see a 6-year-old male in the ED who presents with a history of a 10-second episode of jerking movements of his extremities with unresponsiveness, observed by both of his parents. His parents claim he has had abdominal pain and small quantities of bloody diarrhea for two days. The child has no significant past medical history, has taken no medications recently, has no pets, and has not traveled outside of California in the past year. He attends kindergarten. Which organism is the most likely cause of the child's symptoms?

A. Clostridium difficile B. Enterotoxigenic E. coli (ETEC) C. Rotavirus D. Shigella sonnei E. Vibrio cholerae Shigella sonnei causes bloody diarrhea and WBCs in the stool on Wright stain. Children infected with Shigella can suffer from seizures due to neurotoxin release. A. Clostridium difficile typically causes diarrhea after antibiotic use, which allows the organism to overgrow the normal intestinal flora. It would not be expected to cause diarrhea in an otherwise healthy child who is not taking any medications. Infection can be diagnosed by the presence of C. difficile toxin in the stool. B. ETEC is the strain of E. coli that commonly causes traveler's diarrhea, a noninvasive infection without WBCs in the stool. Diarrhea is typically watery and contains no blood. C. Rotavirus is less likely to be associated with seizures. E. Vibrio cholerae causes massive quantities of watery diarrhea, and patients may present with severe dehydration. Bloody diarrhea would not be expected. It is typically seen in endemic or epidemic form in developing countries, and would not be expected in a child living in the U.S. with no recent travel history.

A previously healthy 8-year-old male is found to have a mildly elevated blood pressure during a routine health maintenance visit. Review of systems is negative. Vital signs are otherwise within normal limits. His physical exam is significant for obesity and is otherwise within normal limits. What is the most likely diagnosis?

A. Coarctation of the aorta B. Hyperthyroidism C. Pheochromocytoma D. Primary hypertension E. Renal artery stenosis The most likely diagnosis is primary hypertension because the only physical findings are hypertension and obesity. Given the mild hypertension and the patient's age, symptoms are unlikely to be present. Other etiologies should be ruled out, but review of systems and physical examination do not currently suggest other etiologies. A. This answer choice is incorrect. Assuming sufficient severity of the coarctation, symptoms include chest pain, cold lower extremities, dizziness, syncope, exercise intolerance, failure to thrive, poor growth, headache, and dyspnea, among others. Distal to the coarctation, pulses will be diminished. Blood pressure will be lower in the lower extremities compared to the upper extremities. Murmurs may also be heard on auscultation. Coarctation of the aorta may also be associated with other congenital heart defects (bicuspid aortic valve in 50% of patients) and chromosomal abnormalities (Turner syndrome). B. This answer choice is incorrect. Symptoms of hyperthyroidism include heat intolerance, frequent bowel movements, increased appetite, diaphoresis, nervousness, restlessness, weight loss, tremor, hair loss, and palpitations, among others. On examination, patients typically present with tachycardia, hypertension, hyperreflexia, and sometimes goiter. The patient will have elevated serum T3 and T4. C. This answer choice is incorrect. Symptoms of pheochromocytoma include headache, diaphoresis, palpitations, tremor, nausea, weakness, anxiety, nervousness, irritability, and weight loss, among other symptoms. On physical examination, patients typically present with tachycardia and severely elevated blood pressure. For laboratory studies, the patient will have elevated urinary VMA, urinary HVA, urinary metanephrines, and serum glucose. Pheochromocytoma may be associated with various syndromes, such as multiple endocrine neoplasia and Von Hippel-Lindau disease. E. This answer choice is incorrect. Patients with renal artery stenosis are largely asymptomatic, but as the kidney function deteriorates, they may experience edema and dyspnea. On physical examination, patients will have hypertension (as a consequence of elevated angiotensin II and aldosterone) and possibly abdominal bruits. For laboratory studies, the patient will have elevated serum angiotensin II and serum aldosterone, as the kidneys attempt to compensate for a perceived decrease in glomerular filtration rate.

A 12-year-old boy comes to your clinic with a chief complaint of crampy abdominal pain. His mother tells you that sometimes he wakes up from sleep due to the pain. He also has diarrhea that sometimes has blood in it. When asked about stressors in his life, his mother sighs and tells you that she is recently divorced. On physical exam, he appears small for his age. Abdomen is soft, non-distended, but tender to palpation at the RUQ. On rectal exam, you note anal skin tags and an anal fistula. Skin exam shows red tender nodules on his shins. What is the next best step in management?

A. Colonoscopy B. Obtain IgA anti-tissue transglutaminase antibody C. Reassure and refer to psychiatry D. Start omeprazole and antibiotics E. Start mesalamine A. Colonoscopy is the best answer. This patient likely has IBD (Crohn's disease or ulcerative colitis [UC]). He has crampy abdominal pain and intermittently bloody diarrhea (that wakes him up at night), perianal disease, and erythema nodosum. He also has microcytic anemia, likely from chronic blood loss. Colonoscopy with biopsies will allow you to diagnose Crohn's disease (or UC) prior to treating it. The diagnosis begins with a colonoscopy to obtain tissue biopsies as well as blood tests. B. This answer is incorrect. IgA anti-tissue transglutaminase antibody is incorrect. This is the right diagnostic test for celiac disease. However, celiac disease would not present with blood in the stool or other extraintestinal signs. Celiac can be associated with dermatitis herpetiformis, an itchy, papulovesicular rash. C. This answer is incorrect. Reassurance and a referral to psychiatry is the right management for functional abdominal pain, which is the most common cause of abdominal pain in this age group. However, bloody diarrhea cannot be explained by functional abdominal pain and suggests a more serious diagnosis. D. This answer is incorrect. Starting omeprazole and antibiotics is the correct management of peptic ulcer disease (PUD) caused by H. pylori. PUD does not account for the perianal disease, diarrhea, and erythema nodosum found in this patient. E. This answer is incorrect. Steroids can be first-line treatment for Crohn's disease, but a diagnosis must be established prior to treatment.

A 17-year-old girl presents for a sports preparticipation physical. She reports that she occasionally gets short of breath and feels light-headed with exercise, and sometimes she experiences chest pain as well. She lost consciousness once last season during a playoff basketball game, but attributed it to feeling sick at the time. Her grandfather died suddenly at age 35 of unknown etiology. Which of the following is the most likely diagnosis?

A. Congenital heart block B. Hypertrophic cardiomyopathy C. Hypoglycemia D. Postural hypotension E. Ventricular septal defect Hypertrophic cardiomyopathy is the most common cause of sudden cardiac death, with a reported incidence of 1 in 500 people in the United States. This is commonly due to a genetic mutation in one of several sarcomeric proteins (which can be tested for on genetic testing). Patients are at risk for ventricular arrhythmias, especially during exercise; they may also experience chest pain during exercise due to "relative" myocardial ischemia (greater myocardial mass to perfuse). Symptoms of chest pain, palpitations, or syncope with exercise need to be evaluated thoroughly for this diagnosis. The exam may be normal if there is no subaortic obstruction (subaortic obstruction will create a systolic murmur that is exaggerated with standing or Valsalva maneuvers). The ECG in a patient with hypertrophic cardiomyopathy may show left ventricular hypertrophy or repolarization abnormalities (abnormal ST segments and T waves), but may also be normal; ECG is not a good screening test for this diagnosis. Patients often require an echocardiogram to visualize the myocardial wall thickness and mass to assess for hypertrophic cardiomyopathy. A. This answer choice is incorrect. Congenital heart block can be caused by congenital heart defects and autoimmune disease in pregnant women (lupus). This condition is a rare cause of arrhythmia and is often diagnosed early in life. Many patients require a pacemaker. C. This answer choice is incorrect. Hypoglycemia is very uncommon in healthy children. It is usually associated with diaphoresis, anxiety, tremulousness, and a feeling of hunger. Chest pain and shortness of breath are less likely. D. This answer choice is incorrect. Postural hypotension is a very common cause of dizziness and visual changes in the pediatric population. It is usually triggered by volume depletion and skipping meals. Hypotension is not usually associated with shortness of breath, chest pain, and prolonged (> 5 min) loss of consciousness. E. This answer choice is incorrect. Ventricular septal defects often have negligible cardiovascular sequelae, or will close up on their own after a number of years. If they are not monitored, however, shunting can cause medial hypertrophy of the pulmonary vasculature with resultant hypertrophy of the right heart, called Eisenmeiger syndrome. This syndrome is defined by pulmonary hypertension and often exercise intolerance with eventual heart failure. It would be unlikely, however, to cause chest pain or syncope. The family history of sudden cardiac death also points to hypertrophic cardiomyopathy rather than a VSD.

An 8-week-old infant is brought to the clinic for a health maintenance visit. She was a twin born at 40 weeks gestation. Pregnancy and delivery were uncomplicated. At birth her weight, height, and head circumference were at the 50th percentile; she is now at the 5th, 10th, and 25th percentiles, respectively. She and her twin sister are exclusively breastfed every four hours scheduled. She does not have any vomiting and has 3 normal stools per day. Physical exam reveals a thin but otherwise healthy infant. What is the most likely cause of this infant's growth pattern?

A. Congenital heart disease B. Cystic fibrosis C. Inadequate caloric intake D. Inborn error of metabolism E. Milk protein allergy Inadequate caloric intake is correct because it is the most common cause of FTT. Furthermore, we have reason to believe that this particular infant is not receiving adequate calories. Breastfed infants should eat every two to three hours until 3 months of age, whereas this infant eats every four hours and shares the milk supply with her twin. Twins can thrive on exclusive breastfeeding, but it requires that the mother consume extra fluids and calories and ensure that both twins receive adequate calories. A. This answer choice is incorrect. Congenital heart disease (CHD) is incorrect for two reasons. CHD severe enough to cause FTT would present with associated signs (such as murmur, cyanosis, or hepatomegaly) and symptoms (such as easy fatigability). B. This answer choice is incorrect. Malabsorption due to cystic fibrosis (CF) usually results in diarrhea, steatorrhea, and constant hunger. Respiratory conditions such as chronic cough or recurrent pneumonias may be present as well. D. An inborn error of metabolism is incorrect.Infants with FTT due to metabolic disorders would likely have other symptoms (such as lethargy, vomiting, etc.). E. Milk protein allergy is incorrect. This infant has no symptoms consistent with milk protein allergy, such as vomiting or colitis (diarrhea; mucus or blood in stools).

Case 26-- A 4-week-old female infant presents to the clinic for a well child check. This infant had an uneventful delivery by normal spontaneous vaginal delivery at full term. She had a normal neonatal metabolic screen. Her growth chart demonstrates a weight at 3.0 kg (< 3rd percentile) and weight for height at < 3rd percentile. The mother reports no drinking or drugs since before this pregnancy. She started breastfeeding at birth, but changed to formula after a week because she was too tired to nurse the baby frequently. She offers the baby a bottle every 4-6 hours. The baby appears hungry and drinks the full volume offered of 3 ounces very well. The baby feed mostly during the daytime, with minimal feeding at night. There has been no diarrhea, hematochezia, vomiting, difficulty breathing, or fevers. The vital signs and exam (apart from a thin infant) are normal. The mother's affect is flat, and she seems anxious when you ask her about her infant. What is the most likely diagnosis for this infant's failure to thrive?

A. Congestive heart failure B. Failure to thrive due to inadequate caloric intake C. Gastroenteritis D. Malabsorption E. Milk protein allergy This infant is likely not consuming adequate calories to grow. The history includes inadequate feeding of a newborn infant, with only 3 ounces every 4-6 hours during the daytime. There is concern that the mother may have post-partum depression based on her reported fatigue and flat affect. Standardized screening tools, such as the Edinburgh, are used routinely in pediatric office settings for this purpose. A. This answer choice is incorrect. Congestive heart failure is important to consider in any child with failure to thrive, but this child has no signs of CHF such as tiring with feeds, sweating, or tachypnea, tachycardia, or hepatomegaly. C. This answer choice is incorrect. Gastroenteritis typically presents with acute onset of loose stools, emesis, and fever. This infant has none of these symptoms. D. This answer choice is incorrect. Malabsorption typically presents with poor weight gain and loose stools despite good caloric intake. This infant has poor caloric intake with constipation, making this diagnosis unlikely. E. This answer choice is incorrect. Milk protein allergy typically causes bloody stools rather than isolated poor weight gain. The patient does not have bloody stools or other signs of allergy or formula intolerance.

Case 7 Questions-- A 2-hour-old infant is evaluated in the nursery for progressively worsening tachypnea. He was born at 32 weeks gestational age via spontaneous vaginal delivery to a mother with diabetes and negative group B streptococcus status. His Apgar scores were 8 at one minute and 9 at five minutes. On physical examination he is large for gestational age. His vital signs are respiratory rate 75 breaths/minute, temperature 36.5 C (97.7 F), and pulse is 130 beats per minute. His lung exam is remarkable for intercostal and subcostal retractions, grunting, and equal breath sounds. His heart exam reveals normal rhythm, normal S1 and S2, no murmurs, and normal peripheral pulses and capillary refill. Which of the following is the most likely cause of the patient's condition?

A. Congestive heart failure B. Pneumothorax C. Respiratory distress syndrome D. Sepsis E. Transient tachypnea of the newborn (TTN) Respiratory distress syndrome (RDS) causes tachypnea and is therefore an important consideration in this case. RDS is more common in premature infants. Given his prematurity, an NSVD birth, and few risk factors for sepsis other than prematurity, this infant is likely to have RDS. A. Congestive heart failure is an important cause of tachypnea. This patient has a normal cardiovascular exam, with no murmur, normal pulses, and good capillary refill. Infants with congestive heart failure usually present with the triad of tachypnea, tachycardia, and hepatomegaly. B. Pneumothoraces are uncommon, but should always be considered in an infant with respiratory distress. Good bilateral air entry argues against this diagnosis. D. Infants may present with respiratory distress from sepsis or from pneumonia. In this case, the infant has a normal temperature, good blood pressure, and normal perfusion. While less likely in this infant, sepsis should always be considered in infants with respiratory distress. E. Transient tachypnea of the newborn (TTN) is much more common in term infants born via C-section.. Although maternal diabetes is a risk factor, TTN is unlikely because he is premature (32 weeks gestational age) and was delivered vaginally. RDS is much more likely due to his prematurity and mode of delivery, although TTN is still a possibility and would need to be evaluated with a CXR.

A 2-week-old infant is brought by her mother to the clinic because of concerns for jaundice, constipation, sleepiness, and poor feeding. She has not had any vomiting. The patient was born at home and received no medications or lab studies. Physical exam reveals enlarged anterior fontanelle, jaundice, hypotonia, and an umbilical hernia. The remainder of the examination is normal. While otaining confirmation of the diagnosis, which of the following is the most appropriate pharmacotherapy in this patient?

A. Consult with pediatric endocrinologist and start treatment with 10 to 15 mcg/kg/day of crushed levothyroxine in liquid, and follow up labs in 2 weeks B. Empiric antibiotics after collection of blood, urine, and CSF cultures C. Glucose and electrolyte supplementation D. Glucocorticoid and mineralocorticoid supplementation E. No treatment needed The American Academy of Pediatrics recommends levothyroxine for the treatment of hypothyroidism for infants 0-6 months of age. Dosing is typically calculated based on age and weight. Consultation with a pediatric endocrinologist on initiation and maintenance of therapy is recommended. B. This answer choice is incorrect. Although sepsis must always be considered in a neonate with jaundice, there is no indication of bacterial infection in this infant with many signs and symptoms of congenital hypothyroidism. C. This answer choice is incorrect. Glucose and electrolyte abnormalities can result from a broad range of illnesses, including congenital adrenal hyperplasia. This infant most likely has congenital hypothyroidism which does not generally present with hypoglycemia and electrolyte disturbances. D. This answer choice is incorrect. Glucocorticoid and mineralocorticoid supplementation would be needed in infants with congenital adrenal hyperplasia. Clinical evaluationof this infant does not show virilization, which is commonly seen in female infants with congenital adrenal hyperplasia. It would still be important for this infant to undergo newborn screening, however, which includes screening for this disorder. E. This answer choice is incorrect. Congenital hypothyroidism requires treatment within 2 weeks of onset of symptoms to mitigate severe brain damage and cognitive impairment. No treatment is needed for children born with hypothyroxemia of prematurity without TSH elevation.

Case 11-- A 2-year-old female presents to the urgent care clinic with a 7-day history of high fever to 38.5 C, a maculopapular rash that began on the palms and soles of her feet, red eyes without discharge, and unilateral cervical adenopathy. What other symptom/sign might you discover on further history and exam?

A. Costovertebral angle tenderness B. Erythematous and edematous feet C. Rhinorrhea D. Tonsillar exudates E. White spots on buccal mucosa The constellation of this patient's symptoms suggests Kawasaki disease. The other two classic signs not mentioned are erythematous tongue ("strawberry tongue"), and erythema/edema of the extremities, which is the best answer here. A. This answer choice is incorrect. Although children with Kawasaki disease can have urethritis, with associated pyuria and dysuria, it is not associated with CVA tenderness , a symptom of a pyelonephritis, which would be highly unlikely given the other signs. C. This answer choice is incorrect. Rhinorrhea is present in multiple disease processes ranging from simple URI to measles. Rhinorrhea would not be seen in Kawasaki disease. Similarly, the conjunctivitis that is seen in Kawasaki disease is a nonpurulent conjunctivitis, without any discharge. D. This answer choice is incorrect. Tonsillar exudates would be present in strep pharyngitis or tonsillitis. Given the prolonged fever, rash, lymph node involvement, and conjunctivitis, the disease process is more widespread than a simple tonsillitis. This constellation of symptoms should be recognized as Kawasaki disease. E. This answer choice is incorrect. White spots on the buccal mucosa are also known as Koplik spots, which are pathognomonic for measles. This patient's fever and conjunctivitis could be seen with measles, but there is no cough or coryza. In addition, the rash typically starts at the head and moves downward, rather than starting on the hands and feet.

A 3-year-old girl is brought into the clinic by her mother because she has a gradually worsening cough and she has been having trouble breathing. Her mother says the patient sounds like she is barking when she coughs. She is up to date with her vaccinations. Her mom always watches her when she's playing. Physical examination of the child reveals inspiratory stridor. She does not have wheezing, there are no retractions, and she has symmetrical breath sounds. No pseudomembranes are appreciated. What is the most likely diagnosis?

A. Croup (laryngotracheobronchitis) B. Epiglottis C. Foreign body aspiration D. Pertussis E. Pneumonia Croup or laryngotracheobronchitis is due to a viral infection (primarily parainfluenza type 1). It is most common in the winter, and often occurs in children age 2 to 5 years. Croup can lead to nonspecific URI symptoms with some degree of airway obstruction. A barky or seal-like cough and inspiratory stridor (which should be differentiated from expiratory wheezes) is common in croup. B. This answer choice is incorrect. Epiglottitis is a life-threatening emergency caused by an infection with H. influenzae type B. It is less common now with the advent of Hib vaccine, but in rare cases can occur due to staphylococcal or streptococcal infections. It most often occurs in children ages 2 to 5 years. Children with epiglottitis present with fever, stridor, drooling, dysphonia, dysphagia, and respiratory distress. They frequently appear toxic and sit in the "sniffing position" (sitting, leaning forward, neck hyperextended, chin protruding). A "thumb sign" (thickened epiglottis and aryepiglottic folds) appears on films. This patient is not exhibiting any of these characteristic symptoms and she is up to date with vaccines, making epiglottitis a less likely diagnosis for her cough. C. This answer choice is incorrect. Symmetrical breath sounds and the gradual onset of this patient's cough make foreign body aspiration less likely. Focal wheezing and tachypnea are also expected with foreign body aspiration. D. This answer choice is incorrect. Pertussis occurs in three phases. The catarrhal stage lasts one to two weeks when children present with upper respiratory symptoms including cough, congestion and rhinorrhea. A paroxysmal stage follows and lasts four to six weeks. During this phase, children have repetitive forceful coughing with massive inspiratory effort ("whoops"). Finally, during the convalescent stage, children present with a continued cough that may last up to three months. The acellular pertussis vaccine protects against pertussis, and, as this patient is up to date with her immunizations, this diagnosis is less likely. Her barking cough is more suggestive of croup than the "whooping" cough of pertussis. E. This answer choice is incorrect. While retractions and cough are present with pneumonia, asymmetric breath sounds and tachypnea would be more specific for a diagnosis of pneumonia.

A 5-week-old infant is brought to the pediatrician for failure to thrive with a history of prolonged feeding time and tachypnea. What other features does this infant most likely have?

A. Cyanosis from a right-to-left shunt B. Early systolic click without a murmur C. Hyperdynamic precordium with a holosystolic murmur D. Systolic murmur with a widely split second heart sound A. This answer choice is incorrect. Cyanosis from a right-to-left shunt would occur in the setting of either cyanotic congenital heart disease (such as tetralogy of Fallot) or severe pulmonary hypertension, such as an unprepaired VSD. Children with cyanotic heart defects typically present with cyanosis, not tachypnea and poor weight gain. An unrepaired left to right shunt such as a VSD can eventually lead to an increase in pulmonary vascular resistance, which is referred to as Eisenmenger's syndrome. When this occurs, the pulmonary vascular resistance may ultimately exceed that of the systemic vascular resistance, leading to reversal of the direction of blood flow through the VSD to become a right-to-left shunt. However, this generally occurs only if the VSD persists for months to years, which is less likely in this 5-week-old infant. B. This answer choice is incorrect. An early systolic click occurs when an abnormal semilunar valve opens at the end of isovolumic contraction early in systole. This occurs when the semilunar valve (aortic or pulmonic) is bicuspid. If the bicuspid valve is also stenotic, the click will be accompanied by a systolic murmur; if the bicuspid valve is regurgitant, the click will be accompanied by a diastolic murmur. A click without a murmur implies that although the valve is anatomically abnormal, its function is normal. Thus a bicuspid semilunar valve that is not accompanied by a murmur has normal function and would not be expected to contribute to symptoms in infancy. C. This is the correct answer choice. A heart murmur from a VSD is typically not appreciated in the immediate newborn period, as the pulmonary vascular resistance is still quite elevated. During this time, since the pulmonary vascular resistance equals the systemic vascular resistance, there is no shunting of blood through the VSD. However, after a few days to weeks after birth, the pulmonary vascular resistance decreases, and the murmur appears, reflecting the shunted flow of blood through the VSD (from left to right). As flow through the VSD increases and becomes audible, the child is at risk for developing symptoms related to excessive pulmonary flow: tachypnea and poor weight gain. D. This answer choice is incorrect. A widely split, fixed S2 indicates an atrial septal defect (ASD). These are often detected in children between the ages of 3 and 5 years. The systolic murmur is due to the increased blood flow across the pulmonic valve. The widely split fixed second heart sound indicates an ASD is the cause of the murmur rather than an innocent heart murmur.

Case 4-- An 8-year-old boy is brought to clinic by his parents because they are concerned that he has not been doing his homework. His teacher recently called the parents to say that their son seems distracted in class, constantly interrupts other children when they are speaking, and is very fidgety. When you speak with the boy, he tells you that he did not know about the homework assignments and that he tries hard to pay attention in class. What is the next best step in management?

A. Do nothing, as this child's behavior is normal B. Find out more about his behavior at home and at school C. Prescribe a stimulant medication for ADHD D. Recommend group therapy for the child E. Suggest behavior modification for the child and parenting classes Contacting the teacher to find out more about the child's behavior at school and learning more about his behavior at home are the best ways to determine if at least six of the symptoms are present in two or more settings, which is required to make the diagnosis of ADHD. It also will be important to learn more about other aspects of this child's life, as there are several factors that can lead to acting out (including learning disability, hearing disability, family stress, and abuse). A. This answer choice is incorrect. It is true that many school-aged children are easily distractible, impatient, and hyperactive. It is important to distinguish those who truly have ADHD from those who do not. To have a diagnosis of ADHD, symptoms must be present in two or more settings. This vignette only describes symptoms at school. C. This answer choice is incorrect. Pharmacotherapy is often used in combination with behavioral modification/group therapy for children diagnosed with ADHD. However, to be diagnosed with ADHD, one must have six or more symptoms in two or more settings for at least six months, and several of these symptoms must be present before the age of 12. The symptoms fall within the three categories of inattention, hyperactivity, and impulsivity. A stimulant medication may be prescribed once a diagnosis of ADHD is made. D. This answer choice is incorrect. Children with ADHD often learn best in group therapy, particularly social skills and self-esteem. Again, this child first requires further evaluation. E. This answer choice is incorrect. Behavior modification for the child and parenting classes for the parents are both used as treatment modalities in ADHD. Parents should be counseled on positive reinforcement, firm non-punitive limit setting, and how to reduce external stimuli. However, this child first requires further evaluation.

A 10-year-old female comes to the clinic for a well child exam. Her mom asks about puberty and wants to know in what order she should expect to see normal pubertal changes in her daughter. Which of the following sequences is correct?

A. breast bud -> pubic hair -> growth spurt -> menarche B. breast bud -> pubic hair -> menarche -> growth spurt C. pubic hair -> breast bud -> growth spurt -> menarche D. pubic hair -> breast bud -> menarche -> growth spurt E. pubic hair -> menarche -> breast bud -> growth spurt A. This choice is correct because breast buds are the first sign (10-11 years), followed by pubic hair (10-11 years), then a growth spurt (12 years), and then menarche (defined as the first occurrence of menstruation, 12-13 years). Most girls reach adult height by approximately 15 years.

A 4-year-old patient presents with several months of cough. Mom also reports a history of red skin patches, which are pruritic, and allergies to peanuts, eggs, and mangoes. Which of the following would most likely be characteristic of the cough that this patient would present with?

A. Does not awaken patient from sleep B. Paroxysmal C. Barking cough D. Worse at night Asthma frequently presents with cough that worsens at night. The cough is usually a dry cough and may be associated with wheezing. It is also often associated with other atopic conditions such as atopic dermatitis and food allergies. A. This choice is incorrect. This patient likely has asthma, which commonly presents with symptoms awakening the patient from sleep. Habit cough disappears at night but is a diagnosis of exclusion. This patient's history is suggestive of atopy, which makes habit cough less likely. B.This choice is incorrect. Paroxysmal coughs are associated with bacterial infections such as pertussis, chlamydia, or mycoplasma.. His chronic cough, in combination with a history suggestive of atopy, makes infection and paroxysmal cough less likely. C. This choice is incorrect. A barking cough is associated with croup or other forms of subglottic disease. Foreign body aspiration can also produce this type of cough. His history of atopy makes barking cough less likely.

A 39-year-old woman who is 22 weeks pregnant comes to the clinic for information on prenatal genetic counseling. Family history is significant for a 10-year-old boy with intellectual disability. A photograph of her son reveals large ears and long face but no other congenital malformations. The mother is worried that she will have a second child with similar problems. Her prenatal serum screening and fetal ultrasound were both normal. If she were to have a second male child with developmental impairment, what would be the most likely etiology?

A. Down syndrome B. Fragile X syndrome C. Klinefelter syndrome D. Trisomy 13 E. Turner syndrome Fragile X syndrome is the most common familial etiology of intellectual disability and is due to an abnormal number of trinucleotide repeats. Children with Fragile X syndrome present with large ears, long face and mandible and, after puberty, large testicles. A. This answer choice is incorrect. Risk of Down syndrome is increased with advanced maternal age, but prenatal screening would show abnormal maternal serum values with or without abnormal fetal ultrasound findings. C. This answer choice is incorrect. Prepubertal children with Klinefelter syndrome have IQs that vary around the low-normal range, but typically have no prepubertal stereotypical physical features. D. This answer choice is incorrect. Patients with Trisomy 13 would not have large ears and long face, but would present with severe impairments at birth. E. This answer choice is incorrect. The child's male sex rules out a diagnosis of Turner syndrome. Also, individuals with Turner syndrome have normal IQ.

A 16-year-old boy presents to your office requesting clearance to play football. You begin by taking his medical history. He says that he feels very well, but admits that he recently experienced one episode of syncope that occurred when he trained really hard for football tryouts with his friends. He denies any shortness of breath, or chest pain currently. Family history is significant for an uncle who died of heat stroke at the age of 30 while playing basketball. Physical examination reveals no abnormalities. What is the next best step in management?

A. ECG now, and if normal, reassurance B. Medically clear him to play C. Observe and follow up in six months D. Referral to Cardiology E. Stress test Referral to cardiology is the absolute next best step! The combination of syncope with exertion and a family history of a young death is concerning for a diagnosis such as hypertrophic cardiomyopathy. An uncle passing away from heat stroke is considered positive family history for sudden death in a young person. This patient must be evaluated by cardiology, even there is no cardiac murmur on exam. A. This answer choice is incorrect. An ECG is indicated in all cases of syncope when the history and exam are not clearly consistent with neurocardiogenic events. Neurocardiogenic syncope, also known as vasovagal syncope, occurs when the brain is not being adequately perfused. Predisposing factors may include dehydration, hypovolemia, and standing up too quickly. It is not usually dangerous, and no further evaluation is needed if the history is consistent and the exam is normal. However, in cases of atypical syncope, as above, in which syncope occurs with exertion, an ECG would not be a sufficient work up. B. This answer choice is incorrect. All medical issues must be resolved prior to clearing a patient to participate in a sport. Syncope with exertion, during exercise, is very concerning, and this patient should be worked up appropriately prior to clearance. C. This answer choice is incorrect. As mentioned above, the syncopal event and positive family history are concerning. It would be inappropriate to solely observe especially if this patient has hypertrophic cardiomyopathy, a very significant risk factor for sudden death. E. This answer choice is incorrect. A stress test is appropriate for evaluating chest pain, particularly in those at risk for atherosclerosis. However, in a patient with known syncope with exertion, a stress test might actually be dangerous. In addition, it only considers vessel disease as a source of pain, and does not take into account structural issues like an outlet obstruction.

A 3-year-old female comes to the clinic with a limp and a slightly externally rotated right hip. Which of the following signs/symptoms would you expect in the history or exam if a diagnosis of transient synovitis were made?

A. ESR of 110 mm/hr B. High-grade fever C. History of a recent upper respiratory tract infection D. Iridocyclitis E. Knee pain A. A significantly elevated ESR would be concerning for septic arthritis. In fact, in one study, an ESR ≥ 40 mm/hr was one of the criteria found to be more associated with septic arthritis than with transient synovitis. Other criteria associated more with septic arthritis than with transient synovitis include: refusal to bear weight, erythema/warmth/swelling of a joint, elevated CRP, and an elevated WBC count. B. High-grade fever would be more concerning for osteomyelitis or septic arthritis, and would not be expected in transient synovitis (although low-grade fever is possible). However, fever occurs in only half of cases of osteomyelitis. Other symptoms associated with osteomyelitis include localized bony tenderness, and refusal to bear weight. Osteomyelitis cannot be ruled out due to absence of fever. C. Transient synovitis of the hip is associated with a low-grade fever and frequently occurs during or after a URI. Between 32% and 50% of children who present with transient synovitis had a recent upper respiratory tract infection. It is also important to remember that transient synovitis is a diagnosis of exclusion, and it is important to rule out other causes of hip pain that may require urgent intervention, such as septic arthritis. D. Iridocyclitis is associated with juvenile idiopathic arthritis (JIA). Other findings associated with JIA include fever and rash. Diagnostic criteria for JIA include children less than 16 years old and at least 6 weeks of arthritis in at least one joint. E. Hip pathology can present as knee pain. However, it is not something that you would use to rule in or rule out transient synovitis (it is not specific for transient synovitis).

A 6-week-old is brought to clinic by his mother for poor weight gain. He was born at 40 weeks' gestation. Pregnancy and delivery were uncomplicated. Family history reveals a cousin with trouble gaining weight and frequent "lung infections." The baby drinks four ounces of formula every two to three hours. He does not cough or gag during feeds. He has frequent, loose stools. His weight today is the same as at his 2-week visit. His weight is < 5th percentile, and height and head circumference are at the 25th percentile. Vital signs are normal. Physical exam reveals decreased adipose tissue, but is otherwise normal. Which of the following is the best next step in management of this patient's growth?

A. Echocardiogram B. Increase formula to a higher calorie mixture C. Stool culture and Wright stain D. Swallow study E. Sweat chloride test The patient's loose stool is concerning for malabsorption. This occurs in patients with cystic fibrosis due to their impaired pancreatic exocrine function. Given the stool history in the face of failure to thrive, a sweat chloride test would be indicated at this time. Cystic fibrosis should still be suspected in the child who has a normal newborn screen. States differ in the type of test offered to screen for cystic fibrosis, and although great strides have been made in newborn detection, methods are not 100% sensitive. A. This answer choice is incorrect. An echocardiogram may be useful if you suspect congestive heart failure as the cause of failure to thrive. In this case, however, the cardiac exam was normal and there were no other signs of heart failure, such as tachypnea, tachycardia, hepatomegaly, or crackles on lung exam. B. This answer choice is incorrect. Caloric intake is appropriate for this infant. This child may need a higher calorie formula in the future, but not before a diagnosis has been made, as there is likely an organic cause of failure to thrive. C. This answer choice is incorrect. Stool culture with Wright stain for WBC's could be useful in a patient with diarrhea if you suspect an infectious cause of the diarrhea. This patient however, presents with no fever and no blood in the stool, and a good appetite, all of which are less consistent with an infectious etiology. D. This answer choice is incorrect. A swallow study would be used to evaluate this child's ability to safely drink fluids, but since he has not been coughing or gagging and appears to take in milk without difficulty, this test would be less useful at this time.

A 3-day-old infant is evaluated in the nursery for poor feeding since birth. The infant takes 1 ounce of formula every 4 hours. Each feed takes about 40 minutes because the patient falls asleep during the feed. The patient has vomited twice today and stooled once yesterday. Physical exam reveals a lethargic infant who is difficult to arouse by exam. The liver is enlarged and muscle tone is decreased. Serum ammonia level is elevated. Which of the following laboratory results would be expected in this patient?

A. Elevated 17-OH progesterone B. Elevated thyroid stimulating hormone C. Elevated urine orotic acid D. Hyponatremia E. Polycythemia Elevated urine orotic acid is diagnostic of OTC deficiency. This is the most common urea cycle disorder and is an x-linked condition. This diagnosis is most likely based on the enlarged liver, mental status changes, and hyperammonemia. A. This answer choice is incorrect. Elevated 17-OH progesterone would be expected and diagnostic of a patient with congenital adrenal hyperplasia (CAH) and would likely be associated with virilization in a female infant. Ammonia should not be elevated in CAH. B. This answer choice is incorrect. Elevated TSH is diagnostic of congenital hypothyroidism. This clinical scenario is not typical of congenital hypothyroidism. D. This answer choice is incorrect. Infants with congenital adrenal hyperplasia may have low sodium, but not patients with OTC deficiency. E. This answer choice is incorrect. Although polycythemia can cause poor feeding and decreased alertness, this patient's elevated ammonia points to a metabolic disorder.

A 6-year-old boy presents to the clinic with a chief complaint of acute onset of bruising. He is afebrile, and his mother reports that he recently had a URI. He was born full-term and has never been hospitalized. He was circumcised at birth with no problems with bleeding. No one in his family has any chronic medical problems. There have been no serious childhood illnesses or deaths. No one has a history of easy bruising or bleeding. On exam you find that he has a purpuric rash on his buttocks and legs. His urinalysis reveals 15 to 20 RBCs/hpf. Which of the following additional findings would be inconsistent with a diagnosis of IgA vasculitis?

A. Elevated serum IgA B. Blood in the stool C. Colicky abdominal pain D. Pain in his knees and ankles E. Low platelets A. Answer choice A is incorrect. IgA vasculitis is an IgA-mediated vasculitis involving the skin, joints, GI tract, and kidneys. Serum IgA may be elevated, occurring in 50% of patients. B. Answer choice B is incorrect. Deposition of IgA immune complexes in the intestinal vasculature results in intestinal bleeding in approximately two-thirds of patients with IgA vasculitis, either occult blood or grossly bloody stool. C. Answer choice C is incorrect. Abdominal pain occurs in 50 to 75% of patients with IgA vasculitis. D. Answer choice D is incorrect. Arthralgia/arthritis, mainly of the knees and ankles, is seen in about 75% of children with IgA vasculitis. E. Answer choice E is correct. Thrombocytopenia is not characteristic of IgA vasculitis, but is commonly seen in idiopathic thrombocytopenic purpura (ITP). Decreased platelets are often the delineating finding between HSP and ITP.

A 10-month-old asymptomatic infant presents with a RUQ mass. Work-up reveals a normocytic anemia, elevated urinary HVA/VMA, and a large heterogeneous mass with scant calcifications on CT. A bone marrow biopsy is performed. Which of the following histologic findings on bone marrow biopsy is most consistent with your suspected diagnosis?

A. Enlarged cells with intranuclear inclusion bodies B. Hypersegmented neutrophils C. Sheets of lymphocytes with interspersed macrophages D. Small round blue cells with dense nuclei forming small rosettes E. Stacks of RBCs A. This answer choice is incorrect. This describes the classic "owl's eyes" seen in CMV and other viral infections. B. This answer choice is incorrect. Hypersegmented neutrophils are characteristic of megaloblastic anemia, a condition associated with a vitamin B12 and/or folate deficiency, not malignancy. C. This answer choice incorrect. Sheets of lymphocytes with interspersed macrophages are associated with Burkitt lymphoma. D. This is the correct answer choice. In addition to neuroblastoma, other tumors associated with small blue cells include Ewing's sarcoma and medulloblastoma, both of which tumors are seen in children. E. This answer choice is incorrect. Stacks of RBCs suggest rouleaux formation, a phenomenon seen in multiple myeloma, a condition not seen in young infants.

A 5-year-old female, previously healthy, presents with an erythematous, vesicular rash on her palms and soles and a high fever for several days. Upon examination, she is also found to have ulcers in her mouth. A few days later, the fever and rash resolve. What is the most likely pathogen?

A. Enterovirus B. Group A strep C. Herpes simplex virus 1 (HSV-1) D. Human herpesvirus 6 (HHV-6) E. HIV This patient's presentation is consistent with infection by cocksackie A, an enterovirus. This is commonly known as hand, foot, and mouth disease (HFMD). Following an incubation period of three to five days, patients have fever, tender vesicles on their hands and feet, and oral ulcers. Sometimes the rash also occurs on the buttocks and the genitals. The infection resolves spontaneously and is spread from person to person via saliva, fluid from the vesicles, stool, or nasal discharge. B. This answer choice is incorrect. Group A strep infection could cause fever and a rash but this rash is described as "sandpaperlike" with small papules, not vesicular, and is also not confined to the hands and feet. C. This answer choice is incorrect. HSV-1 causes gingivostomatitis and can sometimes be accompanied by fever and malaise. Infection may be complicated herpetic whitlow, appearing as grouped vesicular lesions on the hand/fingers; lesions on the feet would be uncommon. D. This answer choice is incorrect. HHV-6 is the virus that causes roseola, which manifests as fever for 3-5 days followed by eruption of a macular or maculopapular rash as the fever resolves. This rash begins on the trunk, eventually spreading to the extremities, and does not cause oral lesions. E. This answer choice is incorrect. HIV infection can increase the risk of oral lesions secondary to infections by HSV-1 or Candida albicans, but again would be unlikely to cause lesions on the hands and feet. Furthermore, at this patient's age (5 years), she is unlikely to be HIV-positive unless it was vertically transmitted from her mother.

A 6-week-old infant is brought to the emergency room by his mother due to sleepiness, constipation, and yellow skin for the past 3 weeks. The mother and the baby recently immigrated to the United States. No medical records from the delivery are available. The infant has been breast-fed since birth. Vital signs are normal. Physical examination reveals jaundice, large anterior and posterior fontanelles, a large tongue, and abdominal distension. Which of the following is the most appropriate next step in management of this patient?

A. Exchange transfusion B. Head ultrasound C. Phototherapy D. Thyroid function studies The diagnosis of untreated congenital hypothyroidism best explains this baby's constellation of symptoms. This infant must be followed carefully for long-term complications of untreated hypothyroidism. This is seen rarely in the U.S. due to early detection on the newborn screen. A. This answer choice is incorrect. Exchange transfusion is the management of infants with unconjugated hyperbilirubinemia approaching levels of concern for bilirubin encephalopathy. B. This answer choice is incorrect. There is no indication of hydrocephalus or concern for bleeding to warrant a head ultrasound. C. This answer choice is incorrect. Phototherapy is the treatment choice for high levels of unconjugated hyperbilirubinemia. This typically occurs in the first few days of life and rarely presents at 6 weeks. This infant has a constellation of findings most consistent with congenital hypothyroidism. This diagnosis typically does not cause high enough bilirubin levels to warrant phototherapy.

A 5-day-old infant presents with a chief complaint of jaundice. As you obtain a careful history, which of the following would NOT be a risk factor for neonatal jaundice?

A. Families of Mediterranean origin have higher risks of hereditary diseases that can cause jaundice. Glucose-6-phosphate dehydrogenase (G6PD) deficiency and the thalassemias are more common in Mediterranean families. Both cause hemolysis, which results in increased bilirubin. B. ABO incompatibility is a common cause of hemolysis in newborns. Mothers with blood type O make IgG antibodies to A-antigens and to B-antigens which are able to cross the placenta and lead to hemolysis of the B-antigen fetal red blood cells. These antibodies can be detected by a positive direct Coombs test. C. Phenylketonuria (PKU) is an autosomal recessive metabolic disorder caused by a mutation in phenylalanine hydroxylase, which is required to convert phenylalanine to tyrosine. PKU leads to buildup of phenylalanine in the brain, leading to intellectual disability, seizures, and death if not detected and treated early. It is not associated with jaundice. D. Poor breastfeeding during the first few days of life and decreased enteral intake may result in breastfeeding jaundice. This may be caused by inadequate maternal milk production, poor infant feeding, or a combination of the two.. Decreased intake leads to decreased motility of the GI tract and retention of meconium. The meconium contains β-glucuronidase, which hydrolyzes the conjugated bilirubin to an unconjugated form, which is reabsorbed and re-circulated into the blood through enterohepatic circulation, increasing bilirubin levels in the blood. E. Difficult deliveries and birth trauma may result in cephalohematomas, or hemorrhage between the skull and the periosteum. Cephalohematomas are caused by the rupture of blood vessels crossing the periosteum, usually caused by a prolonged second stage of labor or the use of forceps or other instrumentation during delivery. As the cephalohematoma resolves, the breakdown of RBCs from the hematoma can result in increased bilirubin levels.

A 15-month-old is able to stand on his own, walk backward, and throw objects underhand. He is unable to draw/scribble or grasp markers. He can say the one-syllable words "ma" and "da" but his words are unintelligible. He holds a sippy cup with help. He does come when called, plays with a ball and waves bye-bye by imitating his parents. Which of the following is the most likely diagnosis in this patient?

A. Fine motor and language delay B. Gross motor and fine motor delay C. Normal development D. Language delay and gross motor E. Social and language delay . A 15-month-old child should be able to scribble, use a cup, and/or stack 2 blocks. His word count should be three to six words at this stage of development. B. Gross motor delay is incorrect because this child is able to walk backward and throw objects without difficulty. However, he does exhibit some delay with fine motor skills with his inability to scribble or hold a sippy cup on his own. C. A 15-month-old child should be more developed in language and fine motor skills. D. Although this child does have some evidence of language delay, his gross motor skills are at a normal stage of development. E. Socially this child is within normal developmental range since he comes when called and he imitates others.

A 6-month-old infant comes to clinic because of several weeks of vomiting after feedings. The vomiting has become blood-streaked, which is when the mom became concerned and brought him in. The baby's PO intake has decreased and he has been losing weight. Abdominal exam is normal, with no masses palpated. What is the most likely diagnosis?

A. Gastroenteritis B. GERD with esophagitis C. Intussusception D. Pyloric stenosis E. Volvulus A. This answer choice is incorrect. Large watery stools, which this patient did not have, are the hallmark of infectious gastroenteritis. Also, gastroenteritis is a more acute disease (typically less than 10 days), while this patient is having symptoms lasting spanning several weeks. B. This answer choice is correct. Regurgitation/spitting up may be difficult to distinguish from true vomiting. Infants who reflux with overfeeding may sometimes have forceful vomiting. Severe esophagitis may result in blood-streaked emesis. Pain from reflux or esophagitis may lead to feeding aversion when gastroesophageal reflux is severe. C. This answer choice is incorrect. Intussusception is not a chronic condition, and there are usually episodes of severe associated pain. A child with intussusception may present with classic "currant jelly" stools, although this is a very late finding, indicating necrosis from bowel ischemia. The abdominal exam in children with intussusception may show the presence of a sausage-like mass on the right side due to the telescoped bowel. D. This answer choice is incorrect. Pyloric stenosis is characterized by a pattern of forceful, projectile, non-bilious vomiting, usually in younger infants. Infants are usually hungry and nurse or bottle feed avidly. An oval mass, 5-15 mm in longest dimension can be felt on deep palpation in the right upper abdomen, especially after vomiting. Symptoms of pyloric stenosis usually begin between 3-5 weeks of age, and very rarely occur after 12 weeks of age. E. This answer choice is incorrect. In a child with volvulus, blood may be seen in the stool but not typically in the vomitus. Bowel ischemia from volvulus can cause significant abdominal pain.

Case 23-- A 6-year-old male presents to the ED with three days of diffuse muscle aches and occasional chills. Today, he has headache and abdominal pain. He reports that he does not feel hungry because he feels sick to his stomach. He reports no recent cough, congestion, sore throat, joint pains, or sick contacts. His vitals are: T 101.3 F, BP 108/71 mmHg, P 110 bpm, R 28 bpm, O2 sat 100% on RA. On physical exam, you notice several petechiae on his wrists and ankles. Upon questioning, his mother says that the spots began on his wrists and then spread to his ankles. His neck is supple and there is no hepatosplenomegaly or lymphadenopathy. He reports no sick contacts but recently visited his cousins in North Carolina. What is the best next step in management?

A. Give acetaminophen and obtain CBC, UA, and BUN/Cr B. Give acetaminophen, obtain a Monospot, write a note for activity restriction, and advise his mother to bring him back if he is unable to tolerate fluids C. Obtain CBC, blood and CSF cultures, and await culture results to guide antibiotic therapy D. Obtain CBC, blood and CSF cultures, then give loading doses of doxycycline and ceftriaxone E. Perform skin scraping of macules and examine under microscope with KOH prep This patient's constellation of symptoms—abdominal pain, headaches, myalgias, fever, and nausea, followed by petechiae on the wrists and ankles—is classic for RMSF. His recent travel to North Carolina also fits with the geographical distribution of RMSF. The treatment of choice is doxycycline. N. meningitidis coverage with ceftriaxone is also necessary given his rash, headaches, and fevers. A. This choice is incorrect. Although Henoch-Schonlein Purpura can present with abdominal pain and a rash on the lower extremities, (consistent with petechiae and purpura), HSP does not usually present with meningeal signs such as headache, fever AND petechial rash. B. This choice is incorrect. Although infectious mononucleosis is a possibility, the above presentation is concerning for Rocky Mountain Spotted Fever (RMSF). Mononucleosis presents most often with sore throat, lymphadenopathy, and hepatosplenomegaly, which are not present in the above patient. C. This choice is incorrect. The history of fever, headache, and petechial rash, raises suspicion for meningitis and this patient should receive empiric antibiotics immediately. Treatment with doxycycline is also indicated with suspicion of RMSF and continued until further testing can be performed. E. This choice is incorrect. Scabies or a fungal infection such as tinea versicolor would not explain his fever, headache, abdominal pain, petechiae, and myalgias. For scabies, his chief concern might be intense itching, and he might have an exposure history.

A 6-week-old infant is brought by her parents to the clinic for increased sleepiness for the past 2 weeks. The patient is not easily aroused for feedings and is not as alert as she was previously. The infant was born abroad and the family recently immigrated to the United States. Neonatal screening was not performed. Vital signs are normal. Physical exam reveals jaundice, an enlarged fontanelle and an umbilical hernia. Which of the following laboratory abnormalities would most likely in this patient?

A. Glucose < 40 mg/dL B. High TSH, low T4 C. High WBC with bandemia D. Low sodium, high potassium Congenital hypothyroidism may present with poor feeding, constipation, jaundice (longer and more persistent than physiologic jaundice of newborn), mottled skin, large fontanelles, hypotonia, hypothermia and an umbilical hernia. Later findings include a hoarse cry, macroglossia, and myxedematous facies. Patients usually remain asymptomatic until 6 weeks of age, as maternal thyroid hormones may still circulate in younger infants. Patients with primary hypothyroidism will have high TSH and low T4 levels. The most common cause of primary hypothyroidism is aplasia or hypoplasia of the thyroid gland, and - less commonly - dyshormonogenesis. Secondary or tertiary hypothyroidism (HPA dysfunction) will have both low or inappropriately normal TSH and low T4, and are relatively rare causes of hypothyroidism in infants. A. This answer choice is incorrect. Symptoms of hypoglycemia in the neonate may include jitteriness, tremors, hypotonia, poor feeding and seizures. Management includes STAT glucose levels and intervention with parenteral or oral glucose. The work up should include laboratory studies to rule out hyperinsulinism (IDM, insulinoma, prematurity), increased metabolic demand (polycythemia, sepsis, asphyxia), and inborn errors of metabolism (galactosemia, glycogen storage diseases, maple syrup urine disease). This patient did not present with jitteriness, tremors, or seizures. C. This answer choice is incorrect. Sepsis should always be considered in lethargic neonates. While septic infants may present with fever or hypothermia, they may also be afebrile. This child did not appear to be acutely ill or toxic in appearance. D. This answer choice is incorrect. Low sodium and high potassium in a lethargic infant would suggest the diagnosis of congenital adrenal hyperplasia (CAH), characterized by a decreased production of cortisol and aldosterone. Low aldosterone results in decreased stimulation of the H/K exchange in the collecting duct, hence loss of sodium, retention of potassium, and dehydration. In combination with low cortisol levels, patients in adrenal crisis may progress to shock, and death if not treated. The usual age of presentation is 1 to 2 weeks of age. Initial laboratory studies would include serum electrolytes, renin, cortisol and cortisol precursors, androgens, and glucose levels. Note that low cortisol will also impact gluconeogenesis and glycogenolysis. This diagnosis would be less likely in a non-viralized female.

Case 14-- An 18-month-old girl presents to the clinic with nasal congestion and fever for three days. She is previously healthy. She is receiving acetaminophen for fever. Temperature is 101.2 F (38.4 C), pulse is 100 beats/minute, respirations are 24 breaths per minute. Oxygen saturations are 98% on room air. She is alert and well perfused with clear mucus coming from both nostrils and no increased work of breathing. Both turbinates show erythema. Her oropharynx is erythematous. No crackles or wheezing are heard. Her immunizations are up to date. Which of the following infectious organisms is most likely cause of this patient's presentation?

A. Group A Strep B. Haemophilus Influenzae type B C. Pertussis D. Rhinovirus E. Strep pneumoniae Rhinovirus causes the common cold and is the most reasonable diagnosis for this patient. Rhinovirus is a very common cause of congestion and other cold-like symptoms. She presents with slightly elevated temperature, slight tachypnea, and inflamed turbinates, and oral mucosa. Her symptoms all correlate with the common cold. A. This answer choice is incorrect. Group A Strep is rarely a cause of pharyngitis in young children and typically does not cause the feared sequelae (rheumatic fever) in those < 3 years. B. This answer choice is incorrect. Haemophilus Influenzae type B causes pneumonia and epiglottitis. This patient does not have the typical symptoms of epiglottitis that include difficulty breathing, high fever, and drooling. With the advent of vaccinations, Hib infections have decreased significantly. C. This answer choice is incorrect. The catarrhal phase of pertussis can be indistinguishable from the common cold but quickly develops into the paroxysmal phase. The paroxysmal phase is characterized by coughing fits and post-tussive emesis, not seen in this patient. It typically does not have associated fever. E. This answer choice is incorrect. This patient's symptoms do not fit a diagnosis of pneumonia, as she does not have significant fever, increased respiratory rate, crackles, or rales.

The mother of a 5-year-old boy calls your office asking if she should take her son to the emergency department or wait another day. She states that her son suddenly developed a "high fever" and is extremely tired. When you ask about her son's behavior, she states that he also seems very confused. She also noticed he had developed reddish-purplish spots on his extremities. What is the next best step in management of this patient?

A. Have the patient hydrate well over the weekend and follow up with you in a few days B. Have the patient make an appointment to come to your office today C. Recommend ipecac to induce emesis and call 911 D. Tell the mother to take her son to an ED immediately or call 911 if they are unable to drive there immediately This patient's symptoms represent meningococcemia. Although it is important to rehydrate and control his fever, it should not be done in an outpatient setting. This is a medical emergency! Sepsis can lead to altered mental status. Signs and symptoms of sepsis include: fever, nausea, vomiting, diarrhea, apnea/dyspnea, oliguria, pallor, tachypnea, tachycardia, lethargy, irritability, petechiae, purpura, tremors, and seizures. A. This choice is incorrect. This patient is too ill to remain at home over the weekend with just supportive care. If he is bacteremic and septic he has a high mortality rate over the next 12 hours. B. This choice is incorrect. Any acute change in mental status should raise serious concern, and therefore an office visit would not be adequate. C. This choice is incorrect. There is no indication for inducing emesis, and this can present a risk of aspiration in a child with altered mental status.

A 2-year-old male is brought into the ED by his mother because of vomiting and altered mental status. He has pinpoint pupils and seems to be drooling and sweating uncontrollably. His heart rate is 60 bpm, his respiratory rate is 45 bpm, and he seems to have difficulty breathing. Which ingestion is the most likely cause of his symptoms?

A. Organophosphates B. Tricyclic antidepressant C. Barbiturates D. Codeine E. Pseudoephedrine Organophosphates cause cholinergic effects, such as miosis, sweating, lacrimation, salivation, urination, increased gastric mobility (vomiting, diarrhea), muscle twitching, bronchospasm, bradycardia, and seizures. A good mnemonic is SLUDGE (salivation, lacrimation, urination, defecation, GI mobility, emesis). B. Tricyclic antidepressants are part of the group of anticholinergics. Overdose can cause mydriasis, dry and red skin, fever, delirium, seizures, tachycardia, urinary retention, and ileus. C. Barbiturates are part of the group of sedative-hypnotics. Excessive ingestion presents as blurred vision, hypotension, apnea, bradycardia, hypothermia, sedation, delirium, and coma. D. Codeine and other opioids can cause miosis, respiratory depression, bradycardia, hypotension, hypothermia, and depressed mental status. E. Pseudoephedrine is a sympathomimetic, and overdose results in mydriasis, fever, diaphoresis, tachycardia, agitation, and seizures.

A 2-month-old infant is brought to the ED after his mother found him in his crib not breathing. She says he had no color and was still when she found him, but quickly regained his color. While you are examining him he starts having a tonic-clonic seizure and subsequently is found to have a temperature of 96 F, HR 200 bpm, and RR 18 bpm. On exam he cries intermittently, does not track you with his eyes, has a tense, full fontanelle, and decreased tone throughout. You also notice a healing bruise on his left arm. After assessing circulation, airway, and breathing you obtain IV access. What is the next step in your diagnostic workup?

A. Head CT B. Lumbar puncture C. Skeletal survey D. Social work consult A. This answer choice is correct. A head bleed due to non- accidental trauma should be the first diagnostic consideration in this case due to the bulging fontanelle, low tone, seizure activity and bruise on the arm. Head CT is highly sensitive for an intracranial bleed, such as a subdural hematoma and can be quickly carried out in the emergency setting. B. This answer choice is incorrect. Meningitis is on the differential; however, this child is afebrile making this diagnosis less likely. Other causes of altered mental status and seizures should be ruled out first. C. This answer choice is incorrect. In cases of suspected non-accidental trauma skeletal survey should be included in the diagnostic work up, but it is not the initial test of choice in this unstable child. D. This answer choice is incorrect. The infant is not yet clinically stable. However, a consult would be appropriate after the infant is stabilized due to suspicion for non-accidental trauma.

A 5-month-old male is brought to the urgent care clinic with a 3 day history of rhinorrhea and non-productive cough. At birth the baby was large for gestational age and exam at the time was notable for macrocephaly, macroglossia, and hypospadias. Vital signs are stable on physical exam at this time. There is copious nasal discharge, but lungs are clear to auscultation. On abdominal exam, you palpate an abdominal mass on the right side just below the subcostal margin. It is 7 cm in diameter and does not cross the midline. The abdomen is soft and non-tender with active bowel sounds. What is the most likely cause of his mass?

A. Hepatoblastoma B. Renal cell carcinoma C. Teratoma D. Wilms' tumor E. Benign mass A. This answer choice is incorrect. While children with Beckwith-Wiedemann syndrome can have hepatoblastoma (in addition to other types of tumors), this is not the most common tumor in this genetic condition. Note that hepatoblastoma may also be associated with familial adenomatous polyposis. B. This answer choice is incorrect. Renal cell carcinomas are much more common in adulthood. Risk factors include cigarette smoking and obesity. C. This answer choice is incorrect. Teratomas are congenital tumors that are present at birth. These benign tumors are often identified incidentally, or may become symptomatic due to mass effect of the lesion within the abdominal cavity. The aggressiveness of the tumor depends on the degree of differentiation. D. This answer choice is correct. Wilms' tumor is commonly associated with Beckwith-Wiedemann syndrome, a genetic overgrowth syndrome. Other features that may be seen in children with this syndrome include omphalocele, hemihypertrophy, hypoglycemia, large for gestational age, and other dysmorphic features. E. This answer choice is incorrect. In patients who have a mass with underlying Beckwith-Wiedemann syndrome, the mass is usually not benign.

A 3-year-old female presents to the ED with sudden onset difficulty walking. She does not have a fever, headache, nausea, or vision changes, but two weeks ago she had a runny nose, a fever, and a rash. Musculoskeletal exam reveals no abnormalities of lower extremities. Neurologic exam reveals bilateral horizontal nystagmus, wide based stance and swaying, and bilateral overreaching on finger to nose test. An LP is performed which reveals a normal CSF. Which of the following is the most likely diagnosis?

A. Hydrocephalus B. Infectious cerebellitis C. Migraine headache D. Opsoclonus-myoclonus syndrome E. Post-infectious cerebellitis Post-infectious cerebellitis (often called acute cerebellar ataxia) typically presents in a younger child with ataxia, nystagmus, vomiting and sometimes dysarthria. It is believed to be an autoimmune response leading to demyelination of the cerebellum occurring several weeks after a viral infection such as varicella or coxsackie virus. A. This answer choice is incorrect. The presentation of hydrocephalus tends to be more gradual or insidious in onset and it is usually associated with headache and vomiting. B. This answer choice is incorrect. Infectious cerebellitis presents with fever and sometimes mental status changes. Pathogens that cause this may include EBV, mumps, and enterovirus. C. This answer choice is incorrect. Migraine headache would present with a headache. Although basilar artery migraines and hemiplegic migraines may present with acute ataxia, headache is an accompanying symptom. D. This answer choice is incorrect. Opsoclonus-myoclonus is a paraneoplastic syndrome that occurs most often with neuroblastoma in a young child who presents with ataxia and jerking or erratic movements as well as jerky conjugate movements of the eyes. This child's preceding URI points to the more probable diagnosis of post-infectious cerebellitis.

A 3-hour-old infant, born by C-section at 36 weeks to a 30-year-old G1P1 with Apgars of 8 and 9 at 1 and 5 minutes, respectively, is found to be tachypneic in the newborn nursery. His mother has a history of type II diabetes that was poorly controlled during her pregnancy. She took prenatal vitamins and no other medications during her pregnancy. Prenatal labs, including GBS, were negative. The mother's membranes ruptured 9 hours prior to delivery, she was afebrile, and the amniotic fluid had no meconium. On physical exam, the infant is large for gestational age. He has good air movement through the lungs bilaterally, without retractions or nasal flaring. He appears well perfused with a normal cardiac exam. He has decreased muscle tone and a weak suck reflex. Cutaneous blood glucose measurement is 39 mg/dL. What is the most likely diagnosis?

A. Hypoglycemia B. Neonatal sepsis C. Pneumothorax D. Transposition of the great arteries E. Transient tachypnea of the newborn Hypoglycemia is a common presentation in an infant born to mothers with diabetes with poor glucose control during her pregnancy. The increase in maternal serum glucose stimulates fetal pancreatic beta cells to increase insulin production, and this hyperinsulinemic state leads to hypoglycemia when the placental glucose supply is discontinued after delivery. At < 4 hours of life, a glucometer reading of < 25 mg/dL without symptoms or < 40 mg/dL with symptoms would require intervention to correct the hypoglycemic state. This infant has signs of hypotonia, with absence of flexed posture and weak suck, and a blood glucose reading of 39 mg/dL, making hypoglycemia the most likely diagnosis. B. Neonatal sepsis is a consideration for any ill newborn. It is most commonly caused by GBS and E. coli, transmitted from mother to baby. Risk factors include prolonged rupture of membranes (> 18 hours prior to delivery), preterm delivery, and chorioamnionitis. Infants may present with fever (or hypothermia), trouble breathing, jaundice, and lethargy. Our infant is premature and tachypneic, but he is afebrile with normal Apgars and no evidence of altered level of alertness. Furthermore, mother was GBS negative, afebrile (no chorioamnionitis), with no prolonged rupture of membranes, making this diagnosis less likely. If his symptoms do not resolve with correction of his hypoglycemia, however, evaluation for sepsis must be considered. C. Pneumothorax is collapse of lung tissue secondary to air accumulation in the pleural space. Risk factors for pneumothorax in an infant include previous intubation or underlying lung disease (such as severe respiratory distress syndrome). Characteristic physical exam findings include asymmetric breath sounds or decrease in breath sounds on one side. This infant has good air movement in bilateral lung fields, making this diagnosis less likely. D. Transposition of the great arteries is a congenital heart defect in which the aorta and pulmonary artery are switched, resulting in poorly oxygenated blood pumped into the systemic circulation. The infant with transposition is generally cyanotic and will be in respiratory distress. This congenital defect is usually accompanied by a VSD, and maternal diabetes is a risk factor. This infant appears well perfused on exam, and has no murmurs on cardiac exam, making transposition a less likely diagnosis. E. Transient tachypnea of the newborn (TTN) is a condition characterized by delayed clearance of amniotic fluid from the infant's lung following birth (persistent postnatal pulmonary edema) resulting in respiratory distress. Infants born by C-section and to mothers with diabetes are at an increased risk of TTN. X-ray findings include "wet" appearing lungs with significant perihilar streaking, interstitial and alveolar fluid, and fluid in the pleural space and along the fissures. Infants with TTN would not have hypotonia and a weak suck. TTN generally resolves within 24-48 hours and is treated symptomatically.

A one-hour-old newborn male is evaluated in the nursery. He was born to a G2P1 mother at 36 weeks gestation via spontaneous vaginal delivery. The mother did not receive prenatal care because she did not have insurance. She thinks that her water broke about two days ago, but she did not have any contractions after that, so she decided not to come to the hospital. She did not start having contractions until 19 hours before she delivered. No meconium was noted at delivery. The infant did not cry vigorously and was tachypneic and cyanotic. His temperature is 38.1 C (100.5 F). His chest x-ray is normal. Which of the following is the most likely cause of this patient' symptoms?

A. Hypothermia B. Meconium aspiration syndrome C. Pneumothorax D. Sepsis secondary to prolonged rupture of membranes E. Transient tachypnea of the newborn (TTN) Prolonged rupture of membranes is when the chorioamniotic membrane ruptures before the onset of labor. The main risks associated with PROM are preterm labor and delivery and neonatal sepsis. The infant's mother reported that her "water broke" two days ago, which indicates that she had PROM. His mother also did not receive prenatal care; therefore, she did not receive any of the prenatal screening tests that she should have, which increases the likelihood that she has an infection that could have potentially been transferred to the infant after the rupture of her membranes. The history of PROM along with his fever and respiratory distress make this answer choice the best choice. A. Hypothermia can cause tachypnea of the newborn, especially premature newborns. However, this patient does not demonstrate hypothermia given his temperature of 100.5. B. Meconium aspiration syndrome occurs when the baby passes meconium in utero and aspirates the meconium either in utero or at the time of delivery. Since it was noted that the amniotic fluid did not contain meconium, it is less likely that he is suffering from meconium aspiration syndrome, even though he has symptoms of respiratory distress that can often be seen in meconium aspiration syndrome. C. Pneumothorax is less likely in this case because of his normal chest x-ray, but is important to consider in a tachypneic newborn. The presence of fever also makes pneumothorax less likely to be the sole cause of his symptoms. E. Transient tachypnea of the newborn (TTN) is a benign, self-limited condition caused by delayed clearance of lung fluid after birth. Patients with TTN usually have a classic chest x-ray that shows coarse fluffy densities that represent fluid-filled alveoli and/or fluid in the pleural space and a small amount of fluid in the fissures on the lateral view. Given this infant's normal chest x-ray and fever, it is unlikely that he has TTN.

A 5-month-old is brought by her parents to the clinic because of decreasing oral intake over the past 4 days. The patient has been sleeping more than previously and seems to tire out when feeding. The patient breastfeeds and eats homemade pureed vegetables. The patient has not had a bowel movement in three days. She has no fever or respiratory symptoms. Physical examination reveals a weak cry and decreased strength. What additional physical examination findings would be expected with this patient's presumptive diagnosis?

A. III/VI systolic murmur B. Absent deep tendon reflexes C. Cataracts and hepatosplenomegaly D. Large tongue and umbilical hernia E. Vesicular rash on her scalp This infant likely has infant botulism which usually presents in the first year of life with hypotonia, lethargy, constipation, weak cry and can eventually lead to respiratory failure. These infants will have absent DTRs. A. This answer choice is incorrect. A grade III/VI systolic murmur may be a sign of congenital heart disease. Congenital heart disease may present with poor feeding, but a large VSD would likely present earlier, have associated respiratory symptoms and would not be associated with constipation and hypotonia. C. This answer choice is correct. An inborn error of metabolism such as galactosemia can present with lethargy and poor feeding in addition to the characteristic hepatosplenomegaly and cataracts, but this infant presented more acutely and at an older age than would be expected for a metabolic disorder. D. This answer choice is incorrect. Although congenital hypothyroidism can present with lethargy, constipation, and poor feeding, the infant would be less likely to present with these symptoms as late as 5 months of age. E. This answer choice is incorrect. A vesicular rash may be seen in neonatal HSV infection, which can be a cause of encephalitis. It is less likely in this older age group and would most likely present with fever and possibly seizure.

A 6-month-old vaccinated infant arrives in the ED with a 12-hour history of poor feeding, emesis, and irritability. On exam, she is ill-appearing with T 39.2 C, P 160 bpm, R 40 bpm, BP 80/50 mmHg. CBC shows WBC 11.2, Hgb 13.5, Plt 250. Urinalysis shows > 100 WBC per hpf, positive leukocyte esterase, and positive nitrites. She has no history of prior urinary tract infection. Chest x-ray is negative. Urine and blood cultures are pending. After bringing her fever down, she is still uninterested in drinking, but her exam improved, and you are confident she does not have meningitis, so an LP is not performed. Which of the following is the best next step in management?

A. Intravenous ceftriaxone B. Intravenous ciprofloxacin C. Intravenous piperacillin + tazobactam D. Oral ampicillin E. Oral doxycycline > The correct answer is A. This answer choice is correct. The patient's presentation is suggestive of a UTI. Given the ill appearance, vital signs, and white count, upper tract disease (pyelonephritis) should be strongly considered. A parenteral (IV/IM) third-generation cephalosporin such as ceftriaxone is the best choice of those listed for pyelonephritis, given its excellent gram negative coverage (except for Pseudomonas). B. This answer choice is incorrect. Ciprofloxacin could be used for complicated UTIs, but it has the potential for adverse reactions in young children so is reserved for patients > 1 year with complications such as resistant organisms or urinary tract anomalies C. This answer choice is incorrect. IV piperacillin + tazobactam has excellent gram negative coverage with added Pseudomonas coverage, but it is highly expensive and Pseudomonas is unlikely to be the cause of a UTI in a child who is not regularly catheterized. D. This answer choice is incorrect. Oral amoxicillin is not the ideal choice for a patient who is ill appearing and uninterested in oral intake. There is rising resistance of E. Coli to amoxicillin, so if it is used, secondary coverage with gentamicin or another aminoglycoside would be needed unless cultures proved the organism was sensitive to ampicillin alone. E. This answer choice is incorrect. Although parenteral and oral treatment produce similar outcomes in high quality RCTs, this patient is ill and refuses to drink, requiring parenteral antibiotics. Oral doxycycline is not an appropriate choice for empiric coverage.

A 3-month-old male presents to the ED with a fever that started the previous day. Mother reports that he is fussy and has decreased oral intake. He has had five fewer diaper changes than usual. He has no vomiting, diarrhea, or respiratory difficulty. On physical exam his temperature is 101.6 F, pulse 110 bpm, RR 24 bpm, and BP 95/67 mmHg. The baby seems irritable and is not consolable by the parent. HEENT exam is significant for dry mucous membranes. Other than his irritability, the rest of the physical exam is unremarkable. CBC shows WBC 3.5, but is otherwise normal. BMP is within normal limits. Urinalysis shows positive leukocyte esterase, positive nitrite, and WBCs > 10/hpf. An LP is performed, and urine and CSF culture results are pending. The patient is placed on IV fluids and is started on cefotaxime. What is the next best step in evaluation?

A. Intravenous pyelogram B. Kidney-ureter-bladder (KUB) x-ray C. Change antibiotic to oral ampicillin D. Renal bladder ultrasound E. VCUG A. This answer choice is incorrect. Intravenous pyelogram would expose the patient to radiation and would not be recommended to screen for renal abnormalities. B. This answer choice is incorrect. KUB is not recommended for UTI, as it does not assess structural abnormalities. C. This answer choice is incorrect. The patient is already on appropriate empiric parenteral antibiotics, so oral antibiotics would not be necessary. Also, ampicillin would not provide appropriate empiric coverage. D. This answer choice is correct. This infant has a fever without other respiratory symptoms. Meningitis and UTI must be considered in patients with fever and unclear source. The only way to rule out meningitis is by lumbar puncture. This patient has a low WBC, suspicious for sepsis, and a UA that is highly suggestive of UTI. Empiric therapy should be started to cover common organisms including E.coli, P. mirabilis, and Klebsiella. Cefotaxime is reasonable empiric therapy. Renal ultrasound is recommended for all infants with pyelonephritis to assess for renal structural abnormalities or signs of obstructive uropathy (hydronephrosis). E. This answer choice is incorrect. VCUG screening is recommended only for recurrent UTI or when there is abnormal renal ultrasound.

A 2-year-old female with normal birth and developmental history presents with increased agitation and decreased arousability. Her father suffers from chronic pain secondary to a back injury, and her mother found an open container of pills on the bed. Vitals reflect bradycardia, bradypnea, hypotension, and slight hypothermia. On physical exam, she exhibits somnolence, constricted pupils, hypoactive bowel sounds, and hyporeflexia. What substance was most likely ingested?

A. Iron B. Amitriptyline C. Insecticides D. Hydromorphone A. Iron is incorrect. Iron toxicity tends to cause severe abdominal symptoms followed by signs of shock. The child would be at risk for coagulopathies, gastrointestinal hemorrhage, and/or metabolic acidosis. B. Incorrect. Amitriptyline is an anticholinergic medication that belongs to the tricyclic antidepressant family. Anticholinergic toxicity can cause fever, dry, and flushed skin, urinary retention, hypertension, tachycardia, mydriasis, and/or decreased gastric motility. C. Incorrect. Ingestion of insecticides, which typically are organophosphates, can induce increased lacrimation, salivation, sweating, urination, bronchorrhea, bronchospasm, muscle twitching, muscle weakness, bradycardia, miosis and blurred vision, and/or increased gastric motility. D. Hydromorphone is correct. Opioids such as hydromorphone can cause respiratory depression, bradycardia, hypotension, hypothermia, constipation, nausea, vomiting, sedation, confusion, and/or miosis.

CASE 17-- A 6-year-old female comes to the clinic because of worsening right knee pain over the past month. On exam, you note generalized lymphadenopathy and splenomegaly. She coughs intermittently throughout the visit, and her mother explains that she is just getting over a cold. You note absence of tenderness, erythema, effusion or warmth over the hip, knee, and ankle joints. Her vitals are unremarkable except for a low-grade fever (100.8 F). Reviewing her chart, you note that she has lost 5 lbs since her visit 2 months ago. She sits with her right leg externally rotated but appears to be in pain despite trying several different positions, refusing to bear weight on that side. What is the most likely diagnosis?

A. Leukemia B. Osteomyelitis C. Reactive arthritis D. Septic arthritis E. Transient synovitis A. Choice A is correct. Leukemia can present as bone pain due to replacement of bone marrow by leukemic cells. Patients may present with a limp or refusal to walk. Leukemia is associated with systemic symptoms such as low-grade fever, chronic/insidious joint pain, generalized LAD, weight loss, and/or hepatosplenomegaly. B. Choice B is incorrect because osteomyelitis typically presents with point tenderness over the bone affected and signs of joint inflammation. It would not be associated with splenomegaly. Osteomyelitis is most commonly associated with infection by Staph aureus or Strep pyogenes. The pain is worse upon weight-bearing, and fever is seen in about 50% of cases. C. Choice C is incorrect because this patient is showing systemic symptoms such as weight loss, hepatosplenomegaly, and generalized LAD, not typically seen in reactive arthritis. Reactive arthritis typically follows a viral infection, presenting two to four weeks following the infection. Children are commonly afebrile, and pain may involve multiple joints. D. Choice D is incorrect because the history and physical are more suggestive of leukemia; however, septic arthritis may present similarly. Erythema, warmth, and swelling of a deep joint may not be readily apparent on exam. While fever is strongly associated with septic arthritis, it would likely be higher (> 38.5 °C) and the patient would have a more acute presentation. A CBC with differential would be useful in this situation. An ultrasound of the joint would also be able to identify an effusion and/or guide joint aspiration if septic arthritis was suspected. Most common organisms responsible for septic arthritis by age include: Staph aureus, GBS, E coli (neonates < 2 months); Staph aureus, Strep pyogenes, Kingella kingae, Strep pneumo (older children); Staph aureus, Neisseria gonorrhea (adolescents). E. Choice E is incorrect because this patient is showing systemic symptoms such as weight loss, hepatosplenomegaly, and generalized LAD. Transient synovitis would be high on the differential if this child were otherwise well-appearing with isolated involvement of the joint, most commonly the hip. Transient synovitis may follow an upper respiratory infection and usually resolves on its own within three to seven days.

A male infant weighing 3200 grams is born to a G1P1 female at 39 weeks' gestational age via planned C-section. Maternal PMH is unremarkable, and GBS status is negative. Apgars are 7 and 8 at 1 and 5 minutes of life, respectively. The delivery was uncomplicated, and the infant initially appeared in good condition. However, one hour following delivery the infant develops increasing respiratory distress. Respiratory rate is assessed as 90 breaths/min. All other vital signs are within normal limits. On exam, the infant is acyanotic with rapid respirations and robust capillary refill. Chest x-ray shows bilateral lung fields with the appearance of "a radio-opaque line of fluid in the horizontal fissure of the right lung." No air bronchograms are noted. What is the most likely etiology of the infant's respiratory distress?

A. Meconium aspiration B.Neonatal sepsis C. Respiratory distress syndrome (RDS) D. Transient tachypnea of the newborn (TTN) Transient tachypnea of the newborn (TTN) is the most likely underlying etiology. This condition is caused by residual fluid in the infant's lungs following delivery, and usually resolves within 24-48 hours. It is more common in babies delivered via C-section, as the normal mechanical force of labor that helps expel fluid from the lungs is lacking. Babies with TTN and other forms of respiratory distress may be unable to nurse and require feeding via NG tube until respiratory status stabilizes. A. Meconium aspiration can lead to respiratory distress, but seems less likely in this case given the infant's delivery via C-section. Additionally, meconium aspiration is more common when meconium is found in the amniotic fluid and/or products of conception. No mention of this was made in the above case description. B. Neonatal sepsis is possible, but less likely given the mother's negative GBS status and the mode of delivery. Sepsis can certainly cause respiratory distress and, if suspected, should be promptly evaluated with screening labs and blood cultures. Neonatal sepsis is also more common with prolonged rupture of membranes (PROM) > 18 hours prior to delivery. C. Respiratory distress syndrome (RDS) is less likely than TTN in this case. RDS is more common in premature infants and infants born to mothers with diabetes. This infant was delivered at term gestation, and mother has no history of diabetes. On chest x-ray, RDS is characterized by a ground-glass appearance and air bronchograms.

An 8-year-old female presents to your clinic for follow-up after being hospitalized for status asthmaticus. She is completing a five-day course of systemic steroids. Given her history of moderate persistent asthma, her outpatient regimen includes daily use of inhaled fluticasone/salmeterol. She was also diagnosed with ADHD one year ago and was started on Adderall XR daily. Her BMI today is at the 83rd percentile for her age, and her blood pressure is at the 98th percentile for her age. What is the most likely cause of her stage I hypertension? A. Medications

A. Medications B. Neurofibromatosis 1 C. Obesity D. Renal insufficiency E. The blood pressure cuff is too big Both steroids and CNS stimulants can cause an increase in blood pressure, especially when used in combination. Steroids increase blood pressure by mimicking endogenous cortisol and the sympathetic fight or flight response. Stimulants mimic norepinephrine, stimulating alpha and beta adrenergic receptors, causing an overall increase in blood pressure. B. This answer choice is incorrect. NF-1 can be associated with hypertension as a result of vascular malformations that affect blood supply to the kidneys; however, this child does not present with any signs of NF-1 (café au lait macules, neurofibromas, optic gliomas, lisch nodules). C. This answer choice is incorrect. While obesity is a risk factor for hypertension, this patient is not overweight. A child is considered overweight when his or her BMI is between the 85th and 95th percentile for age. Obesity is considered > 95th percentile. D. This answer choice is incorrect. Renal insufficiency can be a cause of secondary hypertension, but is unlikely in this patient. Risk factors that warrant investigation of renal causes for hypertension include recurrent UTIs, arterial lines placed while a child was in the NICU, and a family history of renal disease. E. This answer choice is incorrect. Blood pressure would be falsely decreased if the cuff was too big, and, inversely, falsely elevated if the BP cuff was too small.

Case 20-- A 7-year-old male presents with a five-year history of intermittent vomiting, vertigo, and throbbing unilateral headaches that seem to be induced by emotional stress and when his teacher wears perfume. The headaches occur 1-2 times a month and interfere with his school attendance and lifestyle. He reports that the pain is not worsened by long naps or coughing. His mother reports that she has a history of headaches that started as a child and wonders if her son inherited this from her. His neurological exam shows no focal deficits. What is the next step in diagnosis or treatment?

A. NSAIDs B. MRI C. Referral to an ENT surgeon D. Trial of prophylactic medication for migraine headaches This child is presenting with signs of both typical and atypical migraines and could be started on a trial of prophylactic medication. Tricyclic antidepressants (TCAs) , topiramate, propranolol and cognitive behavioral therapy are often used in children for migraine prophylaxis, which is the most likely diagnosis in a child with this constellation of symptoms. A. This answer choice is incorrect. NSAIDS do help in aborting migraine headaches, but in this instance, due to the frequency and severity of episodes, a prophylactic approach should be strongly considered with the family. B. This answer choice is incorrect. An MRI would be indicated if an intracranial mass is suspected. An infratentorial mass, which could increase intracranial pressure, might cause headaches in the morning after laying down, or also with valsalva during a cough. An infratentorial mass can also compress the cerebellum and may cause ataxia, dysarthria, and nystagmus. A supratentorial mass is more likely to present with focal motor and sensory deficit. C. This answer choice is incorrect. While ENT doctors specialize in vertigo that is caused by inner ear pathology, it is likely that this patient's vertigo is due to an atypical migraine.

A mother brings her 8-year-old son to his primary care physician for pain in his knees and ankles that has been present for the past three days. She also notes that he has had a rash since yesterday, but otherwise feels well. The patient has no chronic illnesses, but he was brought in three weeks ago for an upper respiratory infection. Exam is significant for pain elicited on passive movement of the ankles and knees. Additionally, the patient is found to have an erythematous, slightly raised, non-blanching rash over his legs, buttocks, and posterior portion of his elbows. CBC shows WBC 8.9, Hgb 12.5, Hct 36.1, and Plt 327. Urinalysis is unremarkable. Which of the following is the best next step in management?

A. Observation B. Corticosteroids C. Intravenous immunoglobulin (IVIG) D. Intravenous hydration E. Ceftriaxone A. Answer choice A is correct. The patient's presentation is consistent with IgA vasculitis. Most cases of IgA vasculitis resolve within approximately one month and do not require treatment. However, symptomatic treatment for joint pain, e.g. acetaminophen, may be indicated. B. Answer choice B is incorrect. Corticosteroids can be useful for treating severe abdominal pain in patients with IgA vasculitis, but this patient did not present with abdominal pain. Additionally, corticosteroids can be used as part of an immunosuppressive regimen in patients who have IgA vasculitis complicated by acute kidney failure. There is no suggestion of kidney involvement in this case, as the urinalysis was found to be unremarkable. C. Answer choice C is incorrect. IVIG is not used as a mainstay of treatment for IgA vasculitis. D. Answer choice D is incorrect. There is no evidence of significant dehydration or inability to tolerate enteral nutrition in this patient. Thus, intravenous hydration is not warranted. E. Answer choice E is incorrect. Sepsis, including meningococcemia, is an important diagnostic consideration for patients with petechiae/purpura. However, this patient is well-appearing and has a normal CBC, findings that are inconsistent with sepsis.

A previously healthy 14-year-old female presents to the ED with a one-day history of fever and altered mental status. Vital signs on presentation include: BP 115/70 mmHg, Pulse 145 bpm, RR 42 bpm, temp 39.7 C, oxygen sat 93%. Physical exam reveals nuchal rigidity, cool extremities, 1+ distal pulses, diffuse petechial rash, and capillary refill > 2 seconds. What is the important first step in management?

A. Obtain a head CT B. Order blood cultures C. Order CBC, CMP, PT, and INR D. Place IV and start NS bolus E. Perform a lumbar puncture. This patient is in septic shock due to meningococcal infection and should immediately be started on IV fluids in order to maintain perfusion to vital organ systems. Although this patient has a normal blood pressure, other vital signs and physical examination point to shock (HR and RR are both significantly elevated), which first and foremost requires fluid resuscitation. A. This answer choice is incorrect. A head CT is not indicated in the initial evaluation of a patient presenting in septic shock, regardless of the source of infection. If the patient were stable, then a head CT may be required to rule out any masses prior to LP. B. This answer choice is incorrect. Blood cultures would be the step just prior to antibiotics, so as not to contaminate the results. However, cultures still come after fluid resuscitation when a patient is in shock. C. This answer choice is incorrect. Although these labs will need to be obtained, the patient must first be managed in order to ensure hemodynamic stability. Drawing blood for lab tests is not the first priority. E. This answer choice is incorrect. While a lumbar puncture is warranted, medical stabilization is required first. This includes addressing shock with fluid resuscitation as a first step.

An 11-year old boy presents to the clinic with wheezing. Mom states that in the past he has used inhaled albuterol and it has helped with wheezing and shortness of breath. Mother also reports that the patient experiences shortness of breath three times a week and is awakened at night by these symptoms once a week. What is the most appropriate outpatient therapy at this time?

A. Only albuterol rescue inhaler as needed B. Low dose inhaled corticosteroids daily and albuterol rescue inhaler as needed C. Medium dose inhaled corticosteroids daily and albuterol rescue inhaler as needed D. High dose inhaled corticosteroids, LABA (long-acting beta agonist), and oral corticosteroids daily and albuterol rescue inhaler as needed E. A 5-day course of oral corticosteroids Low-dose inhaled corticosteroid daily is correct because this patient has mild persistent asthma. His symptoms occur 3-6 days/week and 3-4 nights/month. A.This answer choice is incorrect. A short-acting beta agonist rescue inhaler is indicated in patients with intermittent asthma. Intermittent asthma is classified as symptoms fewer than two days a week or two nights a month. This patient has more frequent symptoms. C. This choice is incorrect. Medium-dose inhaled corticosteroids would be indicated in a patient with moderate persistent asthma (symptoms occur daily and more than one night per week). D. This answer choice is incorrect. High-dose inhaled corticosteroids, LABA, and oral corticosteroids would be indicated in a patient with severe persistent asthma (symptoms occur continuously throughout the day and every night). E. This answer choice is incorrect. A brief course of oral corticosteroids may be used to treat an acute asthma exacerbation. This patient has poorly controlled symptoms but is not having an acute exacerbation. Long term prevention of exacerbations is better accomplished with a daily inhaled corticosteroid .

A 9-year-old boy presents to your clinic with discoloration under his eyes, persistent cough, and skin rashes. He has struggled with these complaints over the past three years but recently his symptoms have gotten worse, affecting him every other day. He is afebrile. He is found to have wheezing on physical exam and increased lung volume bilaterally on chest x-ray. What would be the most appropriate treatment for him?

A. Oral antibiotics B. Short-acting inhaled beta agonist PRN C. Short-acting inhaled beta agonist PRN with daily low-dose inhaled corticosteroid D. Short-acting inhaled beta agonist PRN with daily medium-dose inhaled corticosteroid E. Long-acting inhaled beta agonist alone Persistent cough and wheezing that affect the patient every other day (3-4 days with symptoms/week) are consistent with mild persistent asthma, which is appropriately treated with short-acting beta agonist PRN and low dose inhaled corticosteroid. The swelling under the eyes (allergic "shiners") and skin rash are other signs of atopy, as mentioned above. A. This answer choice is incorrect. The patient's presentation is more consistent with asthma than an infection. The patient has had these complaints for the last few years. His skin rashes and lower eyelid darkening are consistent with allergic processes (atopy), which are associated with asthma. Furthermore, increased lung volumes bilaterally and persistent cough without fever also suggest asthma, thus antibiotics would not be appropriate. B. This answer choice is incorrect. Cough and wheezing that occur intermittently (< 2 days/week) are consistent with intermittent asthma, which is treated with short-acting beta agonist PRN. His more frequent symptoms suggest persistent asthma. D. This answer choice is incorrect. Short-acting beta agonist PRN with medium dose inhaled corticosteroid is the preferred treatment for moderate or severe persistent asthma, which corresponds to daily symptoms or continuous symptoms throughout the day, respectively. E. This answer choice is incorrect. The use of a long-acting beta agonist is reserved for severe persistent asthma, which corresponds to symptoms throughout the day. Long-acting beta agonists are never used alone as they can induce tachyphylaxis and are associated with an increased risk of death.

A 2-year-old male is brought into the ED by his mother because of vomiting and altered mental status. He has pinpoint pupils and seems to be drooling and sweating uncontrollably. His heart rate is 60 bpm, his respiratory rate is 45 bpm, and he seems to have difficulty breathing. Which ingestion is the most likely cause of his symptoms?

A. Organophosphates B. Tricyclic antidepressant C. Barbiturates D. Codeine E. Pseudoephedrine A. Organophosphates cause cholinergic effects, such as miosis, sweating, lacrimation, salivation, urination, increased gastric mobility (vomiting, diarrhea), muscle twitching, bronchospasm, bradycardia, and seizures. A good mnemonic is SLUDGE (salivation, lacrimation, urination, defecation, GI mobility, emesis). B. Tricyclic antidepressants are part of the group of anticholinergics. Overdose can cause mydriasis, dry and red skin, fever, delirium, seizures, tachycardia, urinary retention, and ileus. C. Barbiturates are part of the group of sedative-hypnotics. Excessive ingestion presents as blurred vision, hypotension, apnea, bradycardia, hypothermia, sedation, delirium, and coma. D. Codeine and other opioids can cause miosis, respiratory depression, bradycardia, hypotension, hypothermia, and depressed mental status. E. Pseudoephedrine is a sympathomimetic, and overdose results in mydriasis, fever, diaphoresis, tachycardia, agitation, and seizures.

A 4-year-old child is refusing to walk over the course of a week. Her mother recalls that she fell off her bike yesterday. On exam, she is afebrile, but has decreased range of motion of her hip. You review her file and note that she is up-to-date on her immunizations and she was last seen three weeks ago for a self-limited episode of diarrhea that she developed while visiting family in rural Mexico. Aspiration of her affected hip joint reveals slight increase in inflammatory cells but normal chemistries and a negative gram stain. Culture is pending. Which of the following is the most likely diagnosis?

A. Osteomyelitis B. Reactive arthritis C. Septic arthritis D. Transient synovitis E. Trauma A. This answer choice is incorrect. The gradual onset of refusing to walk is consistent with osteomyelitis, and only half of patients present with fever. However, this patient is not acutely ill and does not have bony point tenderness as one would expect with hematogenously acquired osteomyelitis. While this is a possible diagnosis, it is further down on the differential diagnosis. B. Correct. The patient likely had a recent case of mild to moderate gastroenteritis in Mexico, which may have been secondary to a bacterial enteritis such as shigella, or campylobacter. In reactive arthritis, joint inflammation occurs a few weeks later because antibodies made during the illness are attacking the joint. While several inflammatory cells would be seen in the aspirate, importantly, the cultures will turn out to be negative. C. This answer choice is incorrect. Although on exam it may be difficult to distinguish septic from reactive arthritis, in septic arthritis the culture is often positive. These patients usually have acute onset of pain, fever, and limp, and the hip or knee is commonly affected. It is caused by an infection, typically bacterial, in the joint space. Septic arthritis is high on the differential for this patient but is not the best answer choice given the history of diarrhea and the benign finding on joint aspiration. D. This answer choice is incorrect. Transient synovitis could certainly present with pain, and is more likely to present acutely in the absence of other complaints. A recent URI, not gastroenteritis, is most consistent with a transient synovitis. E. This answer choice is incorrect. The timing of this patient's pain is not consistent with trauma, since her pain started one week ago, but she fell off her bike yesterday. Of note, children who present with limp secondary to trauma may not have any findings visible on x-ray for a few weeks.

Case 2-- A 36-month-old presents for a well child visit. The parents would like to know if the child's development is progressing appropriately. The child passed a hearing test at birth, and other than a few URIs, has been generally healthy. The child has not had any hospitalizations or serious illnesses. The child is able to run well, walk up stairs, walk slowly down stairs, uses more words than the parents are able to count, but can only use them in short, two or three-word sentences. The child's speech is understandable and the child can draw a circle, but not a cross. Neurological examination reveals normal cranial nerves, normal sensitivity, normal motor reflexes, and no Babinski sign. Which of the following is the next best step in management of this child?

A. Perform a brain-stem auditory evoked potential hearing screen B. Perform a screening exam for autism C. Reassure the parents that the child's development appears normal D. Refer the child to a developmental specialist for comprehensive evaluation E. Refer the child to a specialist for evaluation of his delayed motor development > The correct answer is C. A. A hearing test is an important part of the evaluation for a child failing to meet language milestones, but this child has no evidence of language delay and does not require referral at this point. If he did, a 36-month-old should be able to cooperate with behavioral audiometry.A brain-stem auditory evoked potential hearing test (BAER) would only be indicated in infants who fail to meet language milestones and cannot cooperate with other, more comprehensive testing. B. This answer choice is incorrect. Autism is an increasingly diagnosed cause of developmental delay, but this child is not delayed and no mention is given of any autistic features, such as a lack of symbolic play, repetitive movements, or poor sociability. C. TThis answer choice is correct. he developmental milestones mentioned in the vignette are within the range of normal for a 36-month-old child. In the absence of any other evidence of significant impairment, there is no indication for referral at this point. D. This answer choice is incorrect. If there are reasons for concern on developmental screening tests, a referral may be indicated. However, the developmental milestones mentioned in the vignette are within the range of normal for a 36-month-old child. E. This answer choice is incorrect. This child's motor milestones are not delayed, and no referral is indicated.

A 2-year-old male presents to the ED with a 5-hour history of hyperactivity, fever, and sweating. His BP is 160/90 mmHg, HR 130 bpm, RR 30 bpm. On exam, he has dilated pupils, cool skin, and hyperreflexia. What is his most likely accidental medication ingestion?

A. Pseudoephedrine B. Codeine C. Iron pill D. Acetaminophen E. Propranolol A. Choice A is correct. Ingestion of a sympathomimetic like pseudoephedrine stimulates the beta and alpha adrenergic receptors, causing elevated HR, RR, BP and hypothermia along with diaphoresis, dilated pupils, hyperreflexia, and hyperactivity. B. Choice B is incorrect because constricted pupils are seen in ingestions of opioids. Slowed breathing, rather than tachypnea, may also be seen. C. Choice C is incorrect. Iron toxicity would present with severe abdominal symptoms followed by signs of shock. D. Choice D is incorrect. Acetaminophen toxicity initially presents with minimal symptoms, followed by symptoms of liver toxicity such as jaundice and RUQ pain. E. Choice E is incorrect. Beta-blocker toxicity would cause bradycardia, not tachycardia and hypertension.

A 2-year-old male presents to the ED with a 5-hour history of hyperactivity, fever, and sweating. His BP is 160/90 mmHg, HR 130 bpm, RR 30 bpm. On exam, he has dilated pupils, cool skin, and hyperreflexia. What is his most likely accidental medication ingestion?

A. Pseudoephedrine B. Codeine C. Iron pill D. Acetaminophen E. Propranolol Ingestion of a sympathomimetic like pseudoephedrine stimulates the beta and alpha adrenergic receptors, causing elevated HR, RR, BP and hypothermia along with diaphoresis, dilated pupils, hyperreflexia, and hyperactivity. B. Choice B is incorrect because constricted pupils are seen in ingestions of opioids. Slowed breathing, rather than tachypnea, may also be seen. C. Choice C is incorrect. Iron toxicity would present with severe abdominal symptoms followed by signs of shock. D. Choice D is incorrect. Acetaminophen toxicity initially presents with minimal symptoms, followed by symptoms of liver toxicity such as jaundice and RUQ pain. E. Choice E is incorrect. Beta-blocker toxicity would cause bradycardia, not tachycardia and hypertension.

A 16-year-old male comes to clinic because of concerns about being one of the smallest students in his class. He says that people think that he looks like he should be in middle school rather than high school. His mother says that until about four years ago, she did not notice much difference between the patient and his friends. However, in the past two years, he has become the shortest person in his class. When assessing for pubertal development, what is the first physical exam finding expected in this patient if puberty has started?

A. Pubic hair appearance occurs around 12 years of age, and is usually the second sign of puberty following testicular enlargement. It precedes growth of the penis and scrotum. B. First ejaculations usually occur around 13-14 years of age. These follow growth of the penis and scrotum, and precede a growth spurt. C. Growth of the penis occurs with growth of the scrotum, and usually occurs around 13-14 years of age. It normally follows pubic hair appearance and precedes first ejaculations. D. A growth spurt is usually not appreciated until at least 14 years of age for most boys. It is one of the last secondary sexual characteristics to be noted, following first ejaculations. E. The first sign of puberty in a boy is testicular enlargement. The onset of puberty is quite variable, but usually occurs between 10 and 15 years for boys. It is rare for boys not to have begun puberty by the age of 16. To assess whether or not a male has entered puberty, one must know the order of the appearance of secondary sexual characteristics.

You have accepted a part-time tutoring job for first-year medical students. One of your students asks if you would please clarify the details of normal fetal circulation. Which of the following best describes the path oxygenated blood takes to reach the fetal brain?

A. RA > foramen ovale > LA > LV > systemic circulation B. RA > RV > VSD > LV > systemic circulation C. RA > RV > pulmonary circulation > LA > LV > systemic circulation D. RA > RV > ductus arteriosus > LV > systemic circulatio E. RA > RV > ductus arteriosus > systemic circulation SUBMIT A. This answer choice is correct. In fetal circulation, the foramen ovale connects the RA to the LA, allowing a portion of the blood to bypass the RV and the lungs. Approximately a third of the blood that enters the RA passes through this route (preferentially the most oxygenated via the unbilical veins which joined the IVC to reach the RA, which is then delivered to the brain and heart), leaving the majority of the blood to travel into the RV. Closure of the foramen ovale is a normal transition from fetal to extrauterine circulation. B. This answer choice is incorrect. VSDs are common congenital heart defects, and are not considered a part of normal fetal circulation. C. This answer choice is incorrect. In utero, without ventilation, the pulmonary vasculature is a high-resistance system. As such, only 8% to 10% of the blood that enters the RV flows through the circulation. D. This answer choice is incorrect. The ductus arteriosus does not empty into the LV, but rather into the descending aorta. E. This answer choice is incorrect. The majority of the fetal circulation to the lower body travels this route. Approximately 90% to 92% of the blood that enters the RV (two-thirds of the blood that enters the RA) travels out and through the ductus arteriosus, bypassing the pulmonary circulation and the left heart, ending up in the descending aorta (below where the head and neck vessels arise.) Blood taking this pathway would not go to the fetal brain. This blood is preferentially less oxygenated than that which flows through the foramen ovale. Like the foramen ovale, closure of this bypass is a normal transition from intra to extrauterine life.

A previously healthy 10-year-old boy comes to the clinic with a chief complaint of progressive cough for three days that began gradually. His cough is described as productive with whitish sputum. His mother reports that he has been febrile up to 101.5 F daily. She thinks he is fatigued and has not eaten well in the past several days. He has no throat pain, vomiting, or diarrhea on review of systems. On exam, there is air passage throughout all lung fields, with crackles in the lower right lung field, but no other abnormal sounds. Which of the following would you likely find in your evaluation?

A. Response to inhaled beta-agonist B. Hyperinflation in one lung field on chest radiograph C.Alveolar consolidation in the right lower lobe on chest radiograph D. Positive PCR for pertussis E.Fluffy bilateral infiltrates and a large heart on chest radiograph Alveolar consolidation in the right lower lobe would be suggestive of a pneumonia. The prolonged fever, productive cough, fatigue and focal findings on examination are all suggestive of pneumonia. A. This answer choice is incorrect. Response to an inhaled beta agonist might be suggestive of asthma. Asthma usually presents with a dry, nonproductive cough and may be associated with wheezing. A wet, productive cough with fever would not be a typical presentation of asthma. B. This answer choice is incorrect. Hyperinflation in one lung field would be consistent with a foreign body aspiration. Foreign body aspirations might present with fever and cough but would be unusual in a school-aged child unless they had other neurologic impairments. In addition, a detailed history would usually uncover an acute precipitating event rather than the gradual onset described in the case. D. This answer choice is incorrect. A diagnosis of pertussis can be confirmed by nasopharyngeal PCR. Pertussis can produce a prolonged cough, but the cough is typically paroxysmal and is preceded by upper respiratory symptoms. It is not typically associated with fever, a productive cough, or abnormal lung findings. E. This answer choice is incorrect. Fluffy bilateral infiltrates and a large heart on chest x-ray would be suggestive of congestive heart failure. Children with acute heart failure (due to, for example, myocarditis) may present with fever and cough. However, pulmonary edema due to CHF is a symmetrical process and is unlikely to present with crackles on one side.

A 14-year-old girl presents to your office wondering why she has not had her period yet. Her mother states that she and the patient's grandmother reached menarche at 13 years of age. The patient is concerned she is behind her friends in terms of development. She is doing well in school and has not had developmental problems in the past. On physical examination, her breasts are elevated without a secondary mound, and curly, coarse pubic hair is present on the labia majora in a triangular shape but does not reach the mons pubis. What Tanner stage would you assign this girl?

A. Tanner Stage I B. Tanner Stage II C. Tanner Stage III D. Tanner Stage IV E. Tanner Stage V A. Tanner Stage I breast development consists of no glandular tissue and is prepubertal. Tanner Stage I consists of no pubic hair at all. This is usually around age 10 or younger. B. Tanner Stage II breast development occurs when breast buds form and the areola begins to widen. A small amount of long, downy hair with slight pigmentation appears on the labia majora. This patient's elevated breast buds and pubic hair distribution puts her beyond Tanner Stage II. C. The patient in the vignette is at Tanner Stage III of development. Her breast buds are elevated but do not have the secondary mound characteristic of Tanner Stage IV. Her pubic hair distribution extends more laterally than Stage II but is not adult-like in hair quality and does not extend onto the mons pubis. D. In Tanner Stage IV, breasts are increased in size and elevation and the areola and papilla form a secondary mound that projects from the contour of the rest of the breast, and the pubic hair extends across the mons pubis and spares the medial thighs. E. In Tanner Stage V, breasts reach their adult size and the areola returns to the contour of the surrounding breast while the central papilla remains projecting and the pubic hair extends to the medial surface of the thighs.

A 15-month-old male presents to the ED with a 3-day history of vomiting and diarrhea. His current weight is 11 kg. He was born at 39 weeks, without any perinatal complications. There is no significant history of travel, sick contacts, or recent changes in diet. The mother notes that he has had only 2 urine diapers over the last day. Physical exam is remarkable for an irritable but consolable infant with tachycardia and normal blood pressure. He is crying with minimal tears and his mucous membranes are dry. His abdominal exam is benign. There is no tenting, and capillary refill is 2 seconds. He is diagnosed with gastroenteritis and started on rehydration therapy. Which of the following statements is true?

A. The patient is moderately dehydrated and should be bolused with 220 ccs of D5 ½ normal saline for emergency phase correction, to ensure hemodynamic stability B. The patient is moderately dehydrated and should be managed with 50-100ml/kg of oral rehydrating solution over 2-4 hours. C. The patient is minimally dehydrated and should be managed with adequate fluid and age appropriate diet D. The patient should be rehydrated with clear liquids and then transitioned to a lactose-free diet until his diarrhea resolves. This answer choice is correct. It is appropriate to trial a bolus of oral rehydration solution in the mild-moderately dehydrated patient. The recommended treatment is 50-100ml/kg of oral rehydration administered over 2-4 hours in small aliquots. Oral rehydration would not be appropriate if the child was severely dehydrated.

You are working in the pediatric ED when a 3-year-old female, Jenny, presents with altered mental status for the past six hours. Her mother reports that the babysitter called her at work today after Jenny started acting agitated and "looking very sick." The mother reports "she feels so warm, I think she has a fever and has become dehydrated." On exam, the patient is agitated and anxious with dilated pupils. Her skin is warm and dry. Vitals reveal tachycardia and hypotension. You suspect the child may have accidentally ingested one of her mother's medications. An overdose of which of the following medications could cause Jenny's symptoms?

A. Tricyclic antidepressant B. SSRI C. Decongestant D. Acetaminophen E. ACE inhibitor A. Tricyclic antidepressant (TCA) is correct. TCA toxicity usually presents with sedation. Cardiac toxicity includes conduction delays, arrhythmias, hypotension. Anticholinergic effects cause tachycardia, hypothermia, dilated pupils, flushing and hot, dry skin.. B. SSRI is incorrect, since toxicity causes the serotonin syndrome. Patients usually have mental status changes such as anxiety and restlessness. Autonomic symptoms include fever diaphoresis and hypertension. Hyperreflexia and clonus are common. C. Decongestant is not the correct answer. Decongestant overdose will elicit a sympathomimetic response, including agitation, tachycardia and mydriasis. However, unlike TCA overdose, decongestant overdose will cause hypertension, and fever/sweating. Additionally, severe overdose of sympathomimetics may cause seizures. D. Acetaminophen is not the correct answer. Acetominophen overdose will produce mild and nonspecific symptoms and include right upper quadrant pain, with elevation in liver enzymes, resulting in liver failure at toxic levels. E. ACE inhibitor is not the correct answer. The typical adverse event for ACE inhibitors is cough.

You are working in the pediatric ED when a 3-year-old female, Jenny, presents with altered mental status for the past six hours. Her mother reports that the babysitter called her at work today after Jenny started acting agitated and "looking very sick." The mother reports "she feels so warm, I think she has a fever and has become dehydrated." On exam, the patient is agitated and anxious with dilated pupils. Her skin is warm and dry. Vitals reveal tachycardia and hypotension. You suspect the child may have accidentally ingested one of her mother's medications. An overdose of which of the following medications could cause Jenny's symptoms?

A. Tricyclic antidepressant B. SSRI C. Decongestant D. Acetaminophen E. ACE inhibitor Tricyclic antidepressant (TCA) is correct. TCA toxicity usually presents with sedation. Cardiac toxicity includes conduction delays, arrhythmias, hypotension. Anticholinergic effects cause tachycardia, hypothermia, dilated pupils, flushing and hot, dry skin.. B. SSRI is incorrect, since toxicity causes the serotonin syndrome. Patients usually have mental status changes such as anxiety and restlessness. Autonomic symptoms include fever diaphoresis and hypertension. Hyperreflexia and clonus are common. C. Decongestant is not the correct answer. Decongestant overdose will elicit a sympathomimetic response, including agitation, tachycardia and mydriasis. However, unlike TCA overdose, decongestant overdose will cause hypertension, and fever/sweating. Additionally, severe overdose of sympathomimetics may cause seizures. D. Acetaminophen is not the correct answer. Acetominophen overdose will produce mild and nonspecific symptoms and include right upper quadrant pain, with elevation in liver enzymes, resulting in liver failure at toxic levels. E. ACE inhibitor is not the correct answer. The typical adverse event for ACE inhibitors is cough.

A 5-year-old boy presents to the emergency department with two days of abdominal pain and one day of rash. He has not had any vomiting or diarrhea. Medical history is significant for an upper respiratory tract infection one week ago. He is otherwise healthy and takes no medications. Vital signs are normal. On examination he has petechiae and purpura on his buttocks and lower extremities. Palpation of the abdomen reveals mild tenderness. The remainder of the exam is normal. Laboratory studies are normal. A clinical diagnosis is made. Which of the following is most important to monitor at this patient's follow-up visit in two days?

A. Urinalysis B. Platelet count C. Prothrombin time D. White blood cell count E. Hemoglobin A. Answer choice A is correct. This patient likely has IgA vasculitis. The exact etiology of IgA vasculitis is unknown, but it is believed to involve an IgA-mediated immune response to infection or other triggers. The incidence is 10 cases per 100,000 children with a peak at ages 4 to 6 years (range of 2 to 17 years). IgA vasculitis is characterized by a rash consisting of petechiae and palpable purpura. Other findings include colicky diffuse or periumbilical abdominal pain, arthritis or arthralgia, and renal disease. Given the potential for delayed onset of renal disease, it is important to follow blood pressure closely and check the urine for signs of hematuria or proteinuria; a normal urinalysis does not necessarily exclude subsequent renal involvement Abnormal findings should prompt further evaluation of renal function with serum BUN and creatinine. B. Answer choice B is incorrect. Platelet count is normal in IgA vasculitis and the hallmark rash is a non-thrombocytopenic purpura. Repeating the platelet count at follow-up when the initial value is normal is not indicated. If thrombocytopenia were the cause of this patient's rash, as seen in patients with idiopathic thrombocytopenic purpura (ITP) or leukemia, the platelet count would have been low at time of presentation. C. Answer choice C is incorrect. A coagulation disorder is part of the initial differential for a patient presenting with palpable purpura and petechiae. In this case, our patient with IgA vasculitis has no abnormalities in any coagulation pathway. Instead, the mechanism of petechiae and purpura is secondary to an immune-mediated vasculitis. Therefore, checking the PT/PTT at the follow-up visit is not indicated. D. Answer choice D is incorrect. Leukemia was in the initial differential for this patient's petechial and purpuric rash. Leukemia is characterized by abnormalities in the different cell lines (WBC, RBCs and platelets) resulting in cytopenias. The clinical presentation of leukemia is associated with bone marrow infiltration and cytopenia. Signs and symptoms include petechiae and purpura, bone pain, fever, fatigue, malaise, hepatosplenomegaly, and lymphadenopathy. When leukemia is suspected, it is important to measure the white blood cell count, hemoglobin, and platelets. In IgA vasculitis, there should not be abnormalities in any of these cell lines and, therefore, measuring these values at follow-up is not indicated. E. Answer choice E is incorrect. IgA vasculitis is not typically associated with anemia, and checking the hemoglobin level during follow up is not generally indicated. An exception would be made for patients with IgA vasculitis complicated by intestinal bleeding.


Set pelajaran terkait

EnviroScience: Chapter 19.1: Solid Waste (Part 2)

View Set

Social Studies- Kentucky History- Regions and Places

View Set

Cardiovascular Disease, Stroke, and Framingham Study-Health

View Set

Certified Meeting Professional (CMP) Exam

View Set

Chapter 58: Assessment and Management of Patients With Breast Disorders

View Set

Heimler's History AP World History U4 Review

View Set

Chapter 4 Anatomy and Physiology Review Questions

View Set